Health Assessment Final Mashup

Pataasin ang iyong marka sa homework at exams ngayon gamit ang Quizwiz!

Normal vs Abnormal BMI

Underweight < 18.5 kg/m2 Normal weight 18.5 to 24.9 kg/m2 Overweight 25 to 29.9 kg/m2 Obesity (class 1) 30 to 34.9 kg/m2 Obesity (class 2) 35 to 39.9 kg/m2 Extreme Obesity (class 3) > or equal to 40 kg/m2

hematoma

a localized collection of extravasated blood, usually clotted in an organ, space, or tissue

The TMT accurately measures

core body temperature. The TMT senses the infrared emissions of the tympanic membrane; the tympanic membrane shares the same vascular supply that perfuses the hypothalamus. The TMT is used with unconscious patients or patients in the emergency department, recovery areas, and labor and delivery units. The temperature is displayed in 2 to 3 seconds.

alopecia

hair loss, baldness

religion

refers to an organized system of beliefs concerning the cause, nature, and purpose of the universe, as well as the attendance of regular services

Medications causing sun-sensitivity

sulfonamides thiazide diuretics oral hypoglycemic agents tetracycline

Follow-up database

the status of any identified problems should be evaluated at regular and appropriate intervals. - This type of database is used in all settings to follow up both short-term and chronic health problems.

percussion

is tapping the person's skin with short, sharp stroked to assess underlying structures. The strokes yield an audible vibration and a characteristic sound that depicts the location, size, and density of the underlying organs.

Snellen eye chart

is the most commonly used and accurate measure of visual acuity.

List the different typed of opiates.

morphine, heroin, meperidine

Somatic pain

originates from musculoskeletal tissues or the body surface

erythema

redness of the skin

Holistic care includes assessment factors like

lifestyle behaviors, culture and values, family and social roles, self-care behaviors, job related stress, developmental tasks, and failures and frustrations of life

carbon monoxide poisoning

light skin: bright cherry red in face and upper torso dark skin: cherry-red color in nail beds, lips, and oral mucosa

Otalgia

occurs directly from ear disease or is referred pain from a problem in teeth or oropharynx

Menarche

occurs during the latter half of the sequence of breast and pubic hair development, just after the peak of growth velocity. Coarse curly hairs develop in the pubic area first and then in the axillae.

Nutrition for the aging adult

protein, vitamin, and mineral needs remain the same or increase, nutrient-dense food choices are important to offset lower energy/calorie needs

The jugular venous pressure is an indirect reflection of the

Heart's efficiency of the pump.

Percussion of the chest is

Helpful in identifying surface alterations of the lung tissue

While discussing the treatment plan, the nurse infers that the patient is uncomfortable asking the physician for a different treatment because of fear of the physician's reaction. In this situation, the nurse's verbal interpretation

Helps the patient understand personal feelings in relation to his or her verbal message.

A If pitting edema is present, then the nurse should grade it on a scale of 1+ (mild) to 4+ (severe). Brawny edema appears as nonpitting edema and feels hard to the touch.

How should the nurse document mild, slight pitting edema the ankles of a pregnant patient? a. 1+/0-4+ b. 3+/0-4+ c. 4+/0-4+ d. Brawny edema

Describe the behavior of a person with intoxication of opiates.

Lethargy, somnolence; slurred speech; initial euphoria followed by apathy, dysphoria, and psychomotor retardation; inattention; impaired memory; impaired judgment; impaired social or occupational functioning

The relative proportion of glandular, fibrous, and adipose breast tissue depends on

Nutritional State

PQRST method of pain assessment

P: provocation/palliation Q: quality/quantity R: region/radiation S: severity scale T: timing

Nutritional Status is best determined by

Serum Albumin

Clonus that may be seen when testing deep tendon reflexes is characterized by a(n)

Set of rapid, rhythmic, contractions of the same muscle

A caruncle is a(n)

Small red mass protruding on the urethral meatus.

B An ethnically based variability in the color of the iris and in retinal pigmentation exists, with darker irides having darker retinas behind them.

Which of these assessment findings would the nurse expect to see when examining the eyes of a black patient? a. Increased night vision b. Dark retinal background c. Increased photosensitivity d. Narrowed palpebral fissures

pruritis

severe itching

Genital herpes

sexually transmitted infection caused by the herpes simplex virus; the vesicles erupt on the glans or foreskin.

Sensorineural (or perceptive) hearing loss

signifies pathology of the inner ear cranial nerve VII, or the auditory areas of the cerebral cortex. A simple increase in amplitude may not enable the person to understand words. Sensorineural hearing loss may be caused by presbycusis, a gradual nerve degeneration that occurs with aging, and by ototoxic drugs, which affect the hair cells in the cochlea.

Spirituality

is a broader term focused on a connection to something larger than oneself and a belief in transcendence

What pain is most difficult to assess and treat?

neuropathic pain

sinus arrythmia

normal increase in heart rate that occurs during inspiration

Palpation

often confirms what you noted during inspection. - applies your sense of touch to assess the following factors: texture, temperature, moisture, organ location and size, and any swelling, vibration, or pulsation, rigidity or spasticity, crepitation, presence of lumps or masses, and presence of tenderness or pain.

What is the heart rate of a 10 to 20 year old during exercise/fever?

195-215 bpm

What is the heart rate of a 2 year to 10 year old during exercise/fever?

195-215 bpm

tachycardia

over 95 or 100 beats/min

What is the resting (asleep) heart rate of a 10 to 20 year old?

50-90 bpm

What is the resting (awake) heart rate of a 10 year to 20 year old?

55-90 bpm

The abdomen normally moves with breathing until the age of ____ years.

7

What is the resting (awake) heart rate of a 2 year to 10 year old?

70-100 bpm

What is the resting (asleep) heart rate of a 3 month to 2 year old baby?

80-150 bpm

What is the resting (awake) heart rate of a 3 month to 2 year old baby?

80-150 bpm

What is the resting heart rate (asleep) of a newborn?

80-160 bpm

What is the resting (asleep) heart rate of a 1 week to 3 month old baby?

80-200 bpm

What is the normal range of body temperature?

96.4 - 99.1 F

What is the normal body temperature?

98.6 F

bradycardia

< 50 beats/min

C With an acute infection, tonsils are bright red and swollen and may have exudate or large white spots. Tonsils are enlarged to 2+, 3+, or 4+ with an acute infection.

A 10 year old is at the clinic for a sore throat that has lasted 6 days. Which of these findings would be consistent with an acute infection? a. Tonsils 1+/1-4+ and pink; the same color as the oral mucosa b. Tonsils 2+/1-4+ with small plugs of white debris c. Tonsils 3+/1-4+ with large white spots d. Tonsils 3+/1-4+ with pale coloring

B Successful communication with an adolescent is possible and can be rewarding. The guidelines are simple. The first consideration is ones attitude, which must be one of respect. Second, communication must be totally honest. An adolescents intuition is highly tuned and can detect phoniness or the withholding of information. Always tell him or her the truth.

A 16-year-old boy has just been admitted to the unit for overnight observation after being in an automobile accident. What is the nurses best approach to communicating with him? a. Use periods of silence to communicate respect for him. b. Be totally honest with him, even if the information is unpleasant. c. Tell him that everything that is discussed will be kept totally confidential. d. Use slang language when possible to help him open up.

Palpable inguinal lymph nodes are

Normal if small, movable, and nontender.

A complete database is

Used to perform a thorough or comprehensive health history or physical examination

PQRSTU is

a pain presentation

Dysmenorrhea is

Pain associated with menstruation

The function of the trachea and bronchi is to

Transport gases between the environment and the lungs

A water-hammer or Corrigan pulse is associated with

Aortic Valve Regurgitation

What term is used to describe involuntary muscle movements?

Athetosis

The term rugae refers to

Folds of thin skin of the scrotal wall

Soft, pointed, fleshy papules that occur on the genitalia caused by human papillomavirus (HPV) are known as

Genital Warts

An example of a primary lesion is a(n)

Urticaria

Claudication is caused by

Arterial Insufficiency

The normal color of the optic disc is

Creamy yellow orange to pink

The cervical nodes drain the

Head and Neck

Nonverbal communication is the primary form of communication for which group of individuals?

Infants

An abnormal sensation of burning or tingling is best described as

Parasthesia

A Whenever possible, the nurse should use a trained interpreter, preferably one who knows medical terminology. In general, an older, more mature interpreter is preferred to a younger, less experienced one, and the same gender is preferred when possible.

A female patient does not speak English well, and the nurse needs to choose an interpreter. Which of the following would be the most appropriate choice? a. Trained interpreter b. Male family member c. Female family member d. Volunteer college student from the foreign language studies department

A male patient with possible fertility problems asks the nurse where sperm is produced. The nurse knows that sperm production occurs in the: a. Testes. b. Prostate. c. Epididymis. d. Vas deferens.

ANS: A Sperm production occurs in the testes, not in the other structures listed.

An expected postmenopausal breast change is

Decreased breast size

During a session on substance abuse, the nurse is reviewing statistics with the class. For persons aged 12 years and older, which illicit substance was most commonly used? a. Crack cocaine b. Heroin c. Marijuana d. Hallucinogens

ANS: C In persons age 12 years and older who reported using during the past month, marijuana (hashish) was the most commonly used illicit drug reported.

An example of objective data is

Alert and Oriented, Crepitation in the left knee joint.

To examine a toddler, the nurse should

Allow the child to sit in the parent's lap

Obesity in adults is defined as

BMI greater than 30

The nurse should use which test to check for large amounts of fluid around the patella? a. Ballottement b. Tinel sign c. Phalen test d. McMurray test

Ballottement of the patella is reliable when large amounts of fluid are present. The Tinel sign and the Phalen test are used to check for carpal tunnel syndrome. The McMurray test is used to test the knee for a torn meniscus.

Vaginal lubrication during intercourse is produced by

Bartholin Glands

Automatic associated movements of the body are under the control and regulation of

Basal Ganglia

Hematuria is a term used for

Blood in urine

The first sign of puberty in girls is

Breast and pubic hair development. The first signs of puberty are breast and pubic hair development, beginning between 8½ and 13 years of age. These signs usually occur together, but it is not abnormal if they do not develop together. This development takes about 3 years to complete.

A patient reports consuming approximately 2000 calories per day. For a healthy diet, the patient should

Consume three servings (6-ounce equivalents) or more of whole grain products per day and three servings of other enriched grain products. 2 cups of fruit and 2½ cups of vegetables per day. ess than 300 mg per day of cholesterol. keep total fat intake to 20% to 35% of total calories.

Endogenous obesity is

Caused by excess adrenocorticotropin production by the pituarty gland

Bleeding into the periosteum during birth is known as

Cephalhematoma

The most common sexually transmitted infection in the United States is

Chlamydia

In addition to initiating digestion of food, saliva

Cleans and protects the mucosa

The first heart sound (S1) is produced by the

Closure of the AV Valves S1 occurs with closure of the atrioventricular valves. The second heart sound (S2) occurs with closure of the semilunar valves.

Describe the different colors of bruises.

Color is a light-skinned person is usually (1) red-blue or purple immediately after or within 24 hours of trauma and generally progresses to (2) blue to purple, (3) blue-green, (4) yellow, and (5) brown to disappearing. - A recent bruise in a dark-skinned person is deep, dark purple.

Which of the following is not included in the definition of the thoracic cage?

Costochondral Junction

An area of the body that is supplied mainly from one spinal segment through a particular spinal nerve is identified as a

Dermatome

When taking the health history, the patient complains of pruritus. What is a common cause of this symptom?

Drug Reactions

Assessment of self-esteem and self-concept is part of the functional assessment. Areas covered under self-esteem and self-concept include

Education, financial status, and value belief system.

Viewing the world from another person's inner frame of reference is called

Empathy

The first sign of puberty in boys is

Enlargement of tests. The first sign of puberty in boys is enlargement of the testes. Following the enlargement of the testes, pubic hair appears, then penis size increases. Pubic hair growth extending up the abdomen occurs after puberty.

During assessment of extraocular movements, two back-and-forth oscillations of the eyes in the extreme lateral gaze occurs. This response indicates

Expected

A Intuition is characterized by pattern recognition expert nurses learn to attend to a pattern of assessment data and act without consciously labeling it. The other options are not correct.

Expert nurses learn to attend to a pattern of assessment data and act without consciously labeling it. These responses are referred to as: a. Intuition. b. The nursing process. c. Clinical knowledge. d. Diagnostic reasoning.

Narrow palpebral fissures, epicanthal folds, and midfacial hypoplasia are characteristic of

Fetal Alcohol Syndrome

C Determining the patency of the nares in the immediate newborn period is essential because most newborns are obligate nose breathers. Nares blocked with amniotic fluid are gently suctioned with a bulb syringe. If obstruction is suspected, then a small lumen (5 to 10 Fr) catheter is passed down each naris to confirm patency. The inability to pass a catheter through the nasal cavity indicates choanal atresia, which requires immediate intervention.

Immediately after birth, the nurse is unable to suction the nares of a newborn. An attempt is made to pass a catheter through both nasal cavities with no success. What should the nurse do next? a. Attempt to suction again with a bulb syringe. b. Wait a few minutes, and try again once the infant stops crying. c. Recognize that this situation requires immediate intervention. d. Contact the physician to schedule an appointment for the infant at his or her next hospital visit.

C Unequal pupil size is termed anisocoria. It normally exists in 5% of the population but may also be indicative of central nervous system disease.

In a patient who has anisocoria, the nurse would expect to observe: a. Dilated pupils. b. Excessive tearing. c. Pupils of unequal size. d. Uneven curvature of the lens.

D When reviewing the cardiovascular system, the health care provider should ask whether any activity is limited or whether the child can keep up with her peers. The other items are not appropriate for a child this age.

In obtaining a review of systems on a healthy 7-year-old girl, the health care provider knows that it would be important to include the: a. Last glaucoma examination. b. Frequency of breast self-examinations. c. Date of her last electrocardiogram. d. Limitations related to her involvement in sports activities.

An increase in the transverse diameter of the chest cage in a pregnant woman is due to a(n)

Increase in estrogen

Which of the following pairs correctly expresses the relationship to the lobes of the lungs and their anatomic position?

Lower Lobes—Posterior Chest

Decreased vision in an elderly patient may be due to which of the following conditions?

Macular Degeneration

A severe deficiency of thyroid hormone leading to nonpitting edema, coarse facial features, dry skin, and dry coarse hair is known as

Myxedema

What disease is characterized by a flat, expressionless, or masklike face; a staring gaze; oily skin; and elevated eyebrows?

Parkinson Disease

One of the leg's deep veins is the

Popliteal

Methods to enhance abdominal wall relaxation during examination include

Positioning the patient with the knees bent

C The receiver attaches meaning determined by his or her experiences, culture, self-concept, and current physical and emotional states. The man whose wife has just been diagnosed with lung cancer may be experiencing emotions that affect his receiving.

Receiving is a part of the communication process. Which receiver is most likely to misinterpret a message sent by a health care professional? a. Well-adjusted adolescent who came in for a sports physical b. Recovering alcoholic who came in for a basic physical examination c. Man whose wife has just been diagnosed with lung cancer d. Man with a hearing impairment who uses sign language to communicate and who has an interpreter with him

Craniosynostosis is a severe deformity caused by

Premature closure of the sutures.

The functions of the musculoskeletal system include

Protection and Storage

A flat macular hemorrhage is called a(n)

Purpura

A deep recess formed by the peritoneum between the rectum and the cervix is called

Rectouterine Pouch The rectouterine pouch (or cul-de-sac of Douglas) is the deep recess between the rectum and the cervix. The cervical mucosa during the second month of pregnancy is blue, which is termed the Chadwick sign. The cervix may also turn blue in any condition causing hypoxia or venous congestion. A cystocele is an abnormality of the pelvic musculature in which the bladder prolapses into the vagina. A rectocele is an abnormality of the pelvic musculature in which the rectum prolapses into the vagina.

Testing the deep tendon reflexes gives the examiner information regarding the intactness of the

Reflex arc at specific levels in the spinal cord

At the end of the examination, the examiner should

Review the findings with the patient

Which of the following is an appropriate position to have the patient assume when auscultating for extra heart sounds or murmurs?

Roll toward the left side After auscultation in the supine position, the nurse should have the patient roll onto the left side; the examiner should listen at the apex with the bell for the presence of any diastolic filling sounds (i.e., S3or S4) or murmurs that may be heard only in this position.

Crepitation is an audible sound that is produced by

Rouqhened articular surfaces moving over each other

Which of the following guidelines may be used to identify which heart sound is S1?

S1 Coincides with the carotid artery pulse

_____________ is an emergency requiring surgery.

Testicular Torsion

pattern injury

an injury caused by an object that leaves a distinct pattern on the skin and/or organ (e.g. being whipped with an extension cord) or injury caused by a unique mechanism of injury (e.g. immersion burns to the hand or feet)

A medical diagnosis is used to evaluate

The cause of death

Amplitude is

The intensity (soft or loud) of sound

The ejaculatory duct is

The passage formed by the joining of the vas deferens and the seminal vesicles.

When recording information for the review of systems, the interviewer must document

The presence and absences of all symptoms under the system heading.

A The pericardium is a tough, fibrous double-walled sac that surrounds and protects the heart. It has two layers that contain a few milliliters of serous pericardial fluid.

The sac that surrounds and protects the heart is called the: a. Pericardium. b. Myocardium. c. Endocardium. d. Pleural space.

A The mouth contains three pairs of salivary glands. The largest, the parotid gland, lies within the cheeks in front of the ear extending from the zygomatic arch down to the angle of the jaw. The Stensens duct (not gland) drains the parotid gland onto the buccal mucosa opposite the second molar. The sublingual gland is located within the floor of the mouth under the tongue. The submandibular gland lies beneath the mandible at the angle of the jaw.

The salivary gland that is the largest and located in the cheek in front of the ear is the _________ gland. a. Parotid b. Stensens c. Sublingual d. Submandibular

D The frenulum is a midline fold of tissue that connects the tongue to the floor of the mouth. The uvula is the free projection hanging down from the middle of the soft palate. The palate is the arching roof of the mouth. Papillae are the rough, bumpy elevations on the tongues dorsal surface.

The tissue that connects the tongue to the floor of the mouth is the: a. Uvula. b. Palate. c. Papillae. d. Frenulum.

Arteriosclerosis refers to

Thickening and loss of elasticity of the arterial walls.

What is the major cause of decreased saliva production in older adults?

Use of anticholinergic medications

When addressing a toddler during the interview, the health care provider should

Use short, simple, concrete sentences.

The semilunar valves separate the

Ventricles from the arteries The semilunar valves separate the ventricles from the arteries. The atrioventricular valves separate the atria and ventricles. The atrioventricular valves separate the atria and the ventricles. The septum separates the right atria from the left atria. The vena cava are not separated by a valve from the right atrium; the pulmonary veins are not separated by a valve from the left atrium.

A patient's complaints include bleeding gums, splinter hemorrhages of the nails, and joint pain. These complaints are symptomatic of

Vitamin C Defeciancy

D Prolonged bottle use during the day or when going to sleep places the infant at risk for tooth decay and middle ear infections.

While obtaining a health history from the mother of a 1-year-old child, the nurse notices that the baby has had a bottle in his mouth the entire time. The mother states, It makes a great pacifier. The best response by the nurse would be: a. Youre right. Bottles make very good pacifiers. b. Using a bottle as a pacifier is better for the teeth than thumb-sucking. c. Its okay to use a bottle as long as it contains milk and not juice. d. Prolonged use of a bottle can increase the risk for tooth decay and ear infections.

What type of database is most appropriate for an individual who is admitted to a long-term care facility?

complete

Describe the withdrawal signs and symptoms of a patient that was intoxicated with opiates.

dilated pupils, lacrimation, runny nose, tachycardia, fever, elevated blood pressure, piloerection, sweating, diarrhea, yawning, insomnia, restlessness, irritability, depression, nausea, vomiting, malaise, tremor, muscle and joint pains; symptoms remarkably similar to clinical picture of influenza

B Normal or physiologic splitting of the S2 is associated with inspiration because of the increased blood return to the right side of the heart, delaying closure of the pulmonic valve.

he nurse knows that normal splitting of the S2 is associated with: a. Expiration. b. Inspiration. c. Exercise state. d. Low resting heart rate.

A Reflection of the light on the corneas should be in exactly the same spot on each eye, or symmetric. If asymmetry is noted, then the nurse should administer the cover test.

hen performing the corneal light reflex assessment, the nurse notes that the light is reflected at 2 oclock in each eye. The nurse should: a. Consider this a normal finding. b. Refer the individual for further evaluation. c. Document this finding as an asymmetric light reflex. d. Perform the confrontation test to validate the findings.

Peyronie disease

result of hard, nontender, subcutaneous plaques on the penis that cause a painful bending of the penis during an erection. Over the glans, the skin folds in and back on itself forming a hood or flap called the foreskin or prepuce.

D With volume overload, as in heart failure and cardiomyopathy, cardiac enlargement laterally displaces the apical impulse and is palpable over a wider area when left ventricular hypertrophy and dilation are present.

uring an assessment, the nurse notes that the patients apical impulse is laterally displaced and is palpable over a wide area. This finding indicates: a. Systemic hypertension. b. Pulmonic hypertension. c. Pressure overload, as in aortic stenosis. d. Volume overload, as in heart failure.

S&S of visceral pain

vomiting nausea pallor diaphoresis

Paraphimosis

when the foreskin is retracted and fixed.

Pulse

with every beat the heart pumps an amount of blood the stroke volume into the aorta. The force flares the arterial walls and generates a pressure wave, which is felt in the periphery as the pulse - assess for rate, rhythm, and pulse

Visceral pain

originates from the larger internal organs - often described as dull, deep, squeezing, or cramping - pain can stem from direct injury to the organ or stretching of the organ from tumor, ischemia, distention, or severe contraction - pain impulse is transmitted by ascending nerve fibers along with nerve fibers of the ANS

Conductive hearing loss

involves a mechanical dysfunction of the external or middle ear. It is a partial loss because the person is able to hear if the sound amplitude is increased enough to reach normal nerve elements in the inner ear. Conductive hearing loss may be caused by impacted cerumen, foreign bodies, a perforated tympanic membrane, pus or serum in the middle ear, and otosclerosis (a decrease in mobility of the ossicles).

Neuropathic pain

pain that does not adhere to the typical and predictable phases in nociceptive pain -pain due to a lesion or disease in the somatosensory nervous system -implies an abnormal processing of the pain message from an injury to the nerve fibers

Benefits of breastfeeding

promotes normal infant growth and development natural immunity through IgA antibodies fewer food allergies and intolerances reduced likelihood of overfeeding less cost than commercial infant formulas increased mother-infant interaction time

phobia

strong, persistent, irrational fear of an object or situation; feels driven to avoid it

vertigo

(subjective) is the feeling of spinning of person; (objective) is feeling of spinning of objects around person

Presbycusis

is a sensorineural loss that affects the middle ear structures or causes damage to nerve cells in the inner ear or to cranial nerve VII

Hearing loss

is anything that obstructs the transmission of sound impairs hearing.

Inspection

is concentrated watching. It is close, careful scrutiny, first of the individual as a whole and then of each body system.

auscultation

is listening to sounds produced by the body, such as the heart and blood vessels and the lungs and abdomen.

culturally competent care

implies that the caregivers understand and attend to the total context of the individual's situation, including awareness of immigration status, stress factors, other social factors, and cultural similarities and differences

hyperemia

increased blood in engorged arterioles light skin: red, bright pink dark skin: purplish tongue but difficult to see; palpate for increased warmth with inflammation, taut skin, and hardening of deep tissues

Gigantism

Excessive secretion of growth hormone by the anterior pituitary results in overgrowth of the entire body. When this occurs during childhood before closure of bone epiphyses, it causes increased height (here 2.09 m, or 6 ft 9 in), as well as increased weight and delayed sexual development.

Pulse force

Force shows the strength of the heart's stroke volume. "weak thready" pulse reflects a decreased SV (i.e. occurs with hemorrhagic shock) "full, bounding" pulse denotes increased SV: anxiety, exercise, and some abnormal conditions. Recorded using a three-point scale: 3+ full, bounding 2+ normal 1+ weak, thready 0 absent

Past health recording

Past health events are important because they may have residual effects on the current health state. - The previous experience with illness may also give clues about how the person responds to illness and the significance of illness for him or her: childhood illnesses, accidents or injuries, hospitalizations, operations, obstetric history, last examination date, allergies, and immunizations. Pg. 47

Complete (total health) database

it describes the current and past health state and forms a baseline against which all future changes can be measured. It yields the first diagnosis

What type of pain can turn into neuropathic pain?

nociceptive pain can change into a neuropathic pain pattern over time when pain has been poorly controlled - due to constant irritation and inflammation caused by a pain stimulus, which alters nerve cells, making them more sensitive to any future stimulus

What is the most convenient and accurate site to read temperature?

oral temperature - the sublingual pocket has a rich blood supply from the carotid arteries that quickly responds to changes in inner core temperature

Obese persons with more fat in the upper body, especially the abdomen, have

Android Obesity

Cessation of menses is known as

Menopause

xerosis

dry skin

The normal temperature is influenced by:

- A diurnal cycle of 1 degree to 1.5 degree F, with trough occurring in the early morning hours and peak occurring in the late afternoon to early evening. - The menstrual cycle in woman. Progesterone secretion, occurring with ovulation at mid cycle, causes a 0.5 - 1 degree F rise. - Exercise. -Age. Wider normal variations occur in the infant an young child because less effective heat control mechanisms. In older adults, temp. usually lower than in other age-groups, with a mean of 36.2 degrees C (97.2 degrees F) via the oral route.

What factors can lead to hearing loss?

- Cilia ling the ear canal become coarse and stiff. This may cause cerumen to accumulate and oxidize, which greatly reduces hearing. The cerumen itself is dryer because of atrophy of the apocrine glands. - A life history of frequent ear infections may result in scarring on the drum. - Impacted cerumen is common in aging adults and other at-risk groups who may underreport that associated hearing loss. Cerumen impaction also blocks conduction in those wearing hearing aids.

The rectal temperature measures ________________ higher than an oral measurement.

0.7 to 1 C

A Normal visual acuity is 20/20 in each eye; the larger the denominator, the poorer the vision.

A patients vision is recorded as 20/80 in each eye. The nurse interprets this finding to mean that the patient: a. Has poor vision. b. Has acute vision. c. Has normal vision. d. Is presbyopic.

C Sputum is not diagnostic alone, but some conditions have characteristic sputum production. Tuberculosis often produces rust-colored sputum in addition to other symptoms of night sweats and low-grade afternoon fevers

A 35-year-old recent immigrant is being seen in the clinic for complaints of a cough that is associated with rust-colored sputum, low-grade afternoon fevers, and night sweats for the past 2 months. The nurses preliminary analysis, based on this history, is that this patient may be suffering from: a. Bronchitis. b. Pneumonia. c. Tuberculosis. d. Pulmonary edema.

What is the resting heart rate (awake) of a newborn?

100-180 bpm

What is the resting (awake) heart rate of a 1 week to 3 month baby?

100-220 bpm

Respirations should be counted for

30 seconds (if regular) and multiplied by two. The respirations should be counted after the pulse assessment.

D How the child reacted to previous hospitalizations and any complications should be assessed. If the child reacted poorly, then he or she may be afraid now and will need special preparation for the examination that is to follow. The other items are not significant for the procedure.

A 5-year-old boy is being admitted to the hospital to have his tonsils removed. Which information should the nurse collect before this procedure? a. Childs birth weight b. Age at which he crawled c. Whether the child has had the measles d. Childs reactions to previous hospitalizations

D Xerostomia (dry mouth) is a side effect of many drugs taken by older people, including antidepressants, anticholinergics, antispasmodics, antihypertensives, antipsychotics, and bronchodilators.

A 72-year-old patient has a history of hypertension and chronic lung disease. An important question for the nurse to include in the health history would be: a. Do you use a fluoride supplement? b. Have you had tonsillitis in the last year? c. At what age did you get your first tooth? d. Have you noticed any dryness in your mouth?

A nursing diagnosis is best described as

A concise statement of actual or potential health concerns of wellness.

D Eye contact is perhaps among the most culturally variable nonverbal behaviors. Asian, American Indian, Indochinese, Arabian, and Appalachian people may consider direct eye contact impolite or aggressive, and they may avert their eyes during the interview. American Indians often stare at the floor during the interview, which is a culturally appropriate behavior, indicating that the listener is paying close attention to the speaker.

A female American Indian has come to the clinic for follow-up diabetic teaching. During the interview, the nurse notices that she never makes eye contact and speaks mostly to the floor. Which statement is true regarding this situation? a. The woman is nervous and embarrassed. b. She has something to hide and is ashamed. c. The woman is showing inconsistent verbal and nonverbal behaviors. d. She is showing that she is carefully listening to what the nurse is saying.

D Gum margins are red and swollen and easily bleed with gingivitis. A changing hormonal balance may cause this condition to occur in pregnancy and puberty.

A pregnant woman states that she is concerned about her gums because she has noticed they are swollen and have started bleeding. What would be an appropriate response by the nurse? a. Your condition is probably due to a vitamin C deficiency. b. Im not sure what causes swollen and bleeding gums, but let me know if its not better in a few weeks. c. You need to make an appointment with your dentist as soon as possible to have this checked. d. Swollen and bleeding gums can be caused by the change in hormonal balance in your system during pregnancy.

A retention cyst in the epididymis filled with milky fluid containing sperm is called

A spermatocele A spermatocele is a retention cyst in the epididymis filled with milky fluid containing sperm.

C During pregnancy, the woman may develop an increased awareness of the need to breathe. Some women may interpret this as dyspnea, although structurally nothing is wrong. Increases in estrogen relax the chest cage ligaments, causing an increase in the transverse diameter. Although the growing fetus increases the oxygen demand on the mothers body, this increased demand is easily met by the increasing tidal volume (deeper breathing). Little change occurs in the respiratory rate.

A woman in her 26th week of pregnancy states that she is not really short of breath but feels that she is aware of her breathing and the need to breathe. What is the nurses best reply? a. The diaphragm becomes fixed during pregnancy, making it difficult to take in a deep breath. b. The increase in estrogen levels during pregnancy often causes a decrease in the diameter of the rib cage and makes it difficult to breathe. c. What you are experiencing is normal. Some women may interpret this as shortness of breath, but it is a normal finding and nothing is wrong. d. This increased awareness of the need to breathe is normal as the fetus grows because of the increased oxygen demand on the mothers body, which results in an increased respiratory rate.

Abnormal characteristics of pigmented lesions

ABCDEF A: asymmetry B: border c: color e: elevation or evolution f: funny

The nurse knows that a common assessment finding in a boy younger than 2 years old is: a. Inflamed and tender spermatic cord. b. Presence of a hernia in the scrotum. c. Penis that looks large in relation to the scrotum. d. Presence of a hydrocele, or fluid in the scrotum.

ANS: D A common scrotal finding in boys younger than 2 years of age is a hydrocele, or fluid in the scrotum. The other options are not correct.

When reviewing the musculoskeletal system, the nurse recalls that hematopoiesis takes place in the: a. Liver. b. Spleen. c. Kidneys. d. Bone marrow.

ANS: D The musculoskeletal system functions to encase and protect the inner vital organs, to support the body, to produce red blood cells in the bone marrow (hematopoiesis), and to store minerals. The other options are not correct

When discussing the use of the term subculture, the nurse recognizes that it is best described as: a. Fitting as many people into the majority culture as possible. b. Defining small groups of people who do not want to be identified with the larger culture. c. Singling out groups of people who suffer differential and unequal treatment as a result of cultural variations. d. Identifying fairly large groups of people with shared characteristics that are not common to all members of a culture.

ANS: D Within cultures, groups of people share different beliefs, values, and attitudes. Differences occur because of ethnicity, religion, education, occupation, age, and gender. When such groups function within a large culture, they are referred to as subcultural groups

According to the holistic model, a narrow definition of holistic health includes

Absence from disease

Orchitis is (are)

An acute inflammation of the testes Epispadias is a meatus opening on the dorsal side of the glans or shaft. Peyronie disease is a result of hard, nontender, subcutaneous plaques on the penis that cause a painful bending of the penis during an erection. A hydrocele is a circumscribed collection of serous fluid in the tunica vaginalis surrounding the testes.

A Asthma is allergic hypersensitivity to certain inhaled particles that produces inflammation and a reaction of bronchospasm, which increases airway resistance, especially during expiration. An increased respiratory rate, the use of accessory muscles, a retraction of the intercostal muscles, prolonged expiration, decreased breath sounds, and expiratory wheezing are all characteristics of asthma

An adult patient with a history of allergies comes to the clinic complaining of wheezing and difficulty in breathing when working in his yard. The assessment findings include tachypnea, the use of accessory neck muscles, prolonged expiration, intercostal retractions, decreased breath sounds, and expiratory wheezes. The nurse interprets that these assessment findings are consistent with: a. Asthma. b. Atelectasis. c. Lobar pneumonia. d. Heart failure.

The "review of systems" in the health history is

An evaluation of past and present health state of each body system

The nurse records that the patient's pulse is 3+ or full and bounding. Which of the following could be the cause?

Anxiety A weak, thready pulse may reflect a decreased stroke volume, as with dehydration. A weak, thready pulse may reflect a decreased stroke volume, as with shock. A weak, thready pulse reflects a decreased stroke volume, as with bleeding.

When testing for muscle strength, the examiner should

Apply an opposing force when the individual puts a joint in flexion or extension.

Which of the following tests provides a precise quantitative measure of hearing?

Audiometer test

The ability of the heart to contract independently of any signals or stimulation is due to

Automictic The heart can contract by itself, independent of any signals or stimulation from the body; this property is termed automaticity. Depolarization is the reversal of the resting potential in excitable cardiac muscle cell membranes when stimulated. Conduction is the process by which an electrical impulse is transmitted through the heart. Repolarization is the process by which the membrane potential of a cardiac muscle cell is restored to the cell's resting potential.

The use of euphemisms to avoid reality or to hide feelings is known as

Avoidance language.

A As individuals, nurses lack research skills in evaluating the quality of research studies, are isolated from other colleagues who are knowledgeable in research, and often lack the time to visit the library to read research. The other responses are not considered barriers.

Barriers to incorporating EBP include: a. Nurses lack of research skills in evaluating the quality of research studies. b. Lack of significant research studies. c. Insufficient clinical skills of nurses. d. Inadequate physical assessment skills.

The extrapyramidal system is located in the

Basal Ganglia

In pulsus paradoxus

Beats have weaker amplitude with inspiration and stronger amplitude with expiration.

An example of subjective data is

Left knee has been swollen and hot for 3 days

Inspiration is primarily facilitated by which of the following muscles?

Diaphragm and Intercostals

A Decreased or absent breath sounds occur when the bronchial tree is obstructed, as in emphysema, and when sound transmission is obstructed, as in pleurisy, pneumothorax, or pleural effusion.

During auscultation of the lungs, the nurse expects decreased breath sounds to be heard in which situation? a. When the bronchial tree is obstructed b. When adventitious sounds are present c. In conjunction with whispered pectoriloquy d. In conditions of consolidation, such as pneumonia

D Movement of the extraocular muscles is stimulated by three CNs: III, IV, and VI.

During ocular examinations, the nurse keeps in mind that movement of the extraocular muscles is: a. Decreased in the older adult. b. Impaired in a patient with cataracts. c. Stimulated by cranial nerves (CNs) I and II. d. Stimulated by CNs III, IV, and VI.

The labyrinth of the inner ear is responsible for maintaining the body's

Equilibrium The labyrinth maintains the body's equilibrium. Binaural interaction is controlled by the brainstem and permits locating the direction of a sound. The normal pathway of hearing is by air conduction. The eustachian tube allows equalization of air pressure on each side of the tympanic membrane.

During an assessment, the nurse asks a female patient, How many alcoholic drinks do you have a week? Which answer by the patient would indicate at-risk drinking? a. I may have one or two drinks a week. b. I usually have three or four drinks a week. c. Ill have a glass or two of wine every now and then. d. I have seven or eight drinks a week, but I never get drunk

For women, having seven or more drinks a week or three or more drinks per occasion is considered at-risk drinking, according to the National Institute on Alcohol Abuse and Alcoholism.

Which of the following pairs of sinuses is absent at birth, is fairly well developed between 7 and 8 years of age, and is fully developed after puberty?

Frontal The frontal sinuses are absent at birth, are fairly well developed between 7 and 8 years of age, and reach full size after puberty. The maxillary sinuses are present at birth and reach full size after all permanent teeth have erupted. The sphenoid sinuses are minute at birth and develop after puberty. The ethmoid sinuses are present at birth and grow rapidly between 6 and 8 years of age and after puberty.

Binaural interaction at the level of the brainstem permits

Identification and location of the direction of the sound

Deep palpation is used to

Identify abdominal contents

D When the health care provider uses the response of confrontation, the frame of reference shifts from the patients perspective to the perspective of the health care provider, and the health care provider starts to express his or her own thoughts and feelings. Empathy, reflection, and facilitation responses focus on the patients frame of reference.

In using verbal responses to assist the patients narrative, some responses focus on the patients frame of reference and some focus on the health care providers perspective. An example of a verbal response that focuses on the health care providers perspective would be: a. Empathy. b. Reflection. c. Facilitation. d. Confrontation.

Functional Assessment

It measures a person's self-car ability in the areas of general physical health; ADLs such as bathing, dressing, toileting, eating, walking; instrumental ADLs or those needed for independent living such as housekeeping, shopping, cooking, cooking, doing laundry, using the telephone, managing finances; social relationships and resources; self- concept and coping; and, home environment.

A student nurse has been assigned to teach fourth graders about hygiene. While preparing, the student nurse adds information about the sweat glands. Which of the following should be included while discussing this topic?

Newborn infants do not sweat and have compensatory mechanisms to make up for body temp.

The general survey consists of four distinct areas. These areas include

Physical appearance, body structure, mobility, and behavior

What are infants are risk for with use of a bottle ?

Prolonged bottle use increases risk for tooth decay and middle ear infections.

B, C, A First-level priority problems are immediate priorities, such as trouble breathing (remember the airway, breathing, circulation priorities). Second-level priority problems are next in urgency, but not life-threatening. Third-level priorities (e.g., patient education) are important to a patients health but can be addressed after more urgent health problems are addressed

Put the following patient situations in order according to the level of priority. a. A patient newly diagnosed with type 2 diabetes mellitus does not know how to check his own blood glucose levels with a glucometer. b. A teenager who was stung by a bee during a soccer match is having trouble breathing. c. An older adult with a urinary tract infection is also showing signs of confusion and agitation

Use of the Snellen Eye chart

The Snellen Chart should be in a well-lit spot at eye level. Position the person on a mark exactly 20 feet from the chart. Use an opaque card to shield one eye at a time during the test. If the person wears glasses or contact lenses, leave them on. Remove only reading glasses because they blur distance vision.

B Specialized cells in the SA node near the superior vena cava initiate an electrical impulse. Because the SA node has an intrinsic rhythm, it is called the pacemaker of the heart.

The component of the conduction system referred to as the pacemaker of the heart is the: a. Atrioventricular (AV) node. b. Sinoatrial (SA) node. c. Bundle of His. d. Bundle branches.

Define holistic health.

The essence of considering the person as a whole. It views the mind, body, and spirit as interdependent and functioning as a whole within the environment.

Which of the following statements regarding the results obtained from use of the Snellen chart is true?

The larger the denominator, the poorer the vision.

A With a loud sudden noise, the nurse should notice the infant turning his or her head to localize the sound and to respond to his or her own name. A startle reflex and acoustic blink reflex is expected in newborns; at age 3 to 4 months, the infant stops any movement and appears to listen.

The nurse assesses the hearing of a 7-month-old by clapping hands. What is the expected response? The infant: a. Turns his or her head to localize the sound. b. Shows no obvious response to the noise. c. Shows a startle and acoustic blink reflex. d. Stops any movement, and appears to listen for the sound.

B The correct technique for using an otoscope is to insert the apparatus into the nasal vestibule, avoiding pressure on the sensitive nasal septum. The tip of the nose should be lifted up before inserting the speculum.

The nurse is using an otoscope to assess the nasal cavity. Which of these techniques is correct? a. Inserting the speculum at least 3 cm into the vestibule b. Avoiding touching the nasal septum with the speculum c. Gently displacing the nose to the side that is being examined d. Keeping the speculum tip medial to avoid touching the floor of the nares

B The costal cartilages become calcified with aging, resulting in a less mobile thorax. Chest expansion may be somewhat decreased, and the chest cage commonly shows an increased anteroposterior diameter.

The nurse knows that a normal finding when assessing the respiratory system of an older adult is: a. Increased thoracic expansion. b. Decreased mobility of the thorax. c. Decreased anteroposterior diameter. d. Bronchovesicular breath sounds throughout the lungs.

B An early sign of otitis media is hypomobility of the tympanic membrane. As pressure increases, the tympanic membrane begins to bulge.

The nurse suspects that a patient has otitis media. Early signs of otitis media include which of these findings of the tympanic membrane? a. Red and bulging b. Hypomobility c. Retraction with landmarks clearly visible d. Flat, slightly pulled in at the center, and moves with insufflation

Decreased estrogen levels during menopause cause

The ovaries atrophy. Decreased estrogen levels during menopause cause atrophy of the ovaries. Decreased estrogen levels during menopause cause the uterus to shrink related to a decrease in the myometrium. Decreased estrogen levels during menopause cause the sacral ligaments to relax and the pelvic musculature to weaken, which causes the uterus to drop. Decreased estrogen levels during menopause cause the cervix to shrink and look pale with a thick, glistening epithelium.

Emergency database

This is an urgent, rapid collection of crucial information and often is compiled concurrently with lifesaving measures

Adnexa is (are)

Uterine Accessory Organs

D Data collection, including performing the health history, physical examination, and interview, is the assessment step of the nursing process (

What step of the nursing process includes data collection by health history, physical examination, and interview? a. Planning b. Diagnosis c. Evaluation d. Assessment

C When using the Doppler ultrasonic stethoscope, the pulse site is found when one hears a swishing, whooshing sound.

When using a Doppler ultrasonic stethoscope, the nurse recognizes venous flow when which sound is heard? a. Low humming sound b. Regular lub, dub pattern c. Swishing, whooshing sound d. Steady, even, flowing sound

D The proficient nurse, with more time and experience than the novice nurse, is able to understand a patient situation as a whole rather than as a list of tasks. The proficient nurse is able to see how todays nursing actions can apply to the point the nurse wants the patient to reach at a future time.

Which statement best describes a proficient nurse? A proficient nurse is one who: a. Has little experience with a specified population and uses rules to guide performance. b. Has an intuitive grasp of a clinical situation and quickly identifies the accurate solution. c. Sees actions in the context of daily plans for patients. d. Understands a patient situation as a whole rather than a list of tasks and recognizes the long-term goals for the patient.

D The superficial veins of the arms are in the subcutaneous tissue and are responsible for most of the venous return.

Which vein(s) is(are) responsible for most of the venous return in the arm? a. Deep b. Ulnar c. Subclavian d. Superficial

Risks for ear infections in children

a first episode within 3 months of life increases risk for recurrent OM. (Recurrent OM is 3 episodes in past 3 months or 4 within past year) - Passive and parenteral smoke are risk factors for OM. - Daycare attendance and bottle-feeding are risk factors for OM.

flight of ideas

abrupt change, rapid skipping from topic to topic, practically continuous flow of accelerated speech; topics usually have recognizable associations or are plays on words

CVA

an upper motor neuron lesion (central). A stroke is an acute neurologic deficit caused by blood clot of a cerebral vessel, as in atherosclerosis (ischemic stroke), or a rupture in a cerebral vessel (hemorrhagic stroke). If you suspect a stroke, ask if the person can smile. Note paralysis of the lower facial muscles but also note that the upper half of face is not affected because of the intact nerve from the unaffected hemisphere. The person is still able to wrinkle the forehead and close the eyes. However, stroke requires emergency 911 treatment.

Rectal temperatures

are the most accurate route and the result is as close to core temperature as possible without using more invasive measures reserved for the operating room, and critical care environments

clubbing of nails

finding in the nails that indicates chronic hypoxia nail angle is > 160 degrees

delusion

firm, fixed, false beliefs; irrational; person clings to delusion despite objective evidence to contrary

Focused or Problem-Centered Database

here you collect a "mini" database that is smaller in scope and more targeted than the complete database. It concerns mainly one problem, one cue complex, or one body system. - This is for a limited or short-term problem

How is temperature regulated within the body?

hypothalamus of the brain through a feedback mechanism

polycythemia

increased red blood cells, capillary stasis light skin: ruddy blue in face, oral mucosa, conjunctiva, hands, and feet dark skin: well concealed by pigment; check for redness in lips

What are the 4 different types of patient databases

complete (total) database focused or problem-centered database follow-up database emergency database

Hypothermia

is usually caused by a accidental, prolonged exposure to cold - it may be purposefully induced to lower the body's oxygen requirements during heart or peripheral vascular surgery, neurosurgery, amputation, postcardiac arrest or GI hemorrhage. - body temperature below 36 C is typically accepted as hypothermia

Empathy

means viewing the world from the other person's inner frame of reference while remaining you - It is a recognition and acceptance of the other person's feelings without criticism - described as the ability to understand and be sensitive to the feelings of someone else.

Venous stasis

decreased blood flow from area, engorged venules light skin: ducky rubor of dependent extremities; a prelude to necrosis with pressure sore dark skin: easily masked; use palpation for warmth or edema

What conditions may cause neuropathic pain?

diabetes mellitus, herpes zoster (shingles), HIV/AIDS, sciatica, trigeminal neuralgia, phantom limb pain, and chemotherapy; CNS lesions such as a stroke, MS, and tumor

A varicocele is a

dilated, tortuous varicose vein in the spermatic cord

Abnormal skin findings in AIDS

multiple patch-stage early lesions are faint pink on the temple and beard area

Can you date bruises by color?

no

Describe the appearance of a person with intoxication of opiates.

pinpoint pupils; decreased blood pressure, pulse, respirations, and temperature

Priapism

prolonged painful erection of the penis.

Subjective data

things a person tells you about that you cannot observe through your senses; symptoms - Pain is defined as an "unpleasant sensory and emotional experience associated with actual or potential tissue damage, or described in terms of such damage. Pain is a subjective experience and as such the person's most reliable indicator of pain.

compulsion

unwanted repetitive, purposeful act; driven to do it; behavior thought to neutralize or prevent discomfort or some dreaded event

What is the heart rate of a 1 week to 3 month old baby during exercise/fever?

up to 220

What is the heart rate of a 3 month to 2 year old baby during exercise/fever?

up to 220

What is the heart rate of a newborn during exercise/fever?

up to 220

List examples of visceral pain.

ureteral colic, acute appendicitis, ulcer pain, and cholecystitis

Mixed hearing loss

a combination of conductive and sensorineural types in the same ear

tinnitus

is the perception of sound without an external source it occurs with sensorineural hearing loss, cerumen impaction, middle ear infection, and other ear disorders

List the age related changes and assessment findings of the eyes.

- Loss of skin elasticity causes drooping and wrinkling - Fat tissue and muscle atrophy - Lacrimal glands involute, causing decreased tear production and a feeling of dryness and burning - The globes infiltration of degenerative lipid material shows around the limbus - Pupil size decreases - The lens loses elasticity, becoming hard and glasslike causing a decreased ability of the lends to change shape to accommodate for near vision which is known as presbyopia - By 70 years of age the normally transparent fibers of the lens begin to thicken and yellow; this is the beginning of a cataract - Vitreous humor is not renewed continuously causing floaters to appear from debris that accumulate - Decreased adaptation to darkness causing them to need more light to see

A safe environment: standard precautions

- Wash your hands before and after every patient encounter, after contact with blood, body fluids, secretions, and excretions, after equipment contaminated with body fluids, and after removing gloves. - Use alcohol based hand sanitizer - Wear gloves when the potential exists for contact with any body fluids

Note the behavioral manifestations of hearing loss.

1) The child is inattentive in casual conversation 2) The child reacts more to movement and facial expression than to sound. 3) The child's facial expression is strained or puzzled. 4) The child frequently asks to have statements repeated. 5) The child confuses words that sound alike. 6) The child has an accompanying speech problem: speech is monotonous or garbled; the child mispronounces or omits sounds. 7) The child appears shy and withdrawn and "lives in a world of his or her own." 8) The child frequently complains of earaches. 9) The child hears better at times when the environment is more conducive.

What is the resting (asleep) heart rate of a 2 year to 10 year old?

60-90 bpm

Parents or caretakers accompany children to the health care setting. Starting at ___ years of age, the interviewer asks the child directly about his or her presenting symptoms.

7

C An empathetic response recognizes the feeling and puts it into words. It names the feeling, allows its expression, and strengthens rapport. Other empathetic responses are, This must be very hard for you, I understand, or simply placing your hand on the persons arm. Simply reflecting the persons words or agreeing with the person is not an empathetic response.

A 17-year-old single mother is describing how difficult it is to raise a 3-year-old child by herself. During the course of the interview she states, I cant believe my boyfriend left me to do this by myself! What a terrible thing to do to me! Which of these responses by the nurse uses empathy? a. You feel alone. b. You cant believe he left you alone? c. It must be so hard to face this all alone. d. I would be angry, too; raising a child alone is no picnic.

B With otitis externa (swimmers ear), swimming causes the external canal to become waterlogged and swell; skinfolds are set up for infection. Otitis externa can be prevented by using rubbing alcohol or 2% acetic acid eardrops after every swim.

A 17-year-old student is a swimmer on her high schools swim team. She has had three bouts of otitis externa this season and wants to know what to do to prevent it. The nurse instructs her to: a. Use a cotton-tipped swab to dry the ear canals thoroughly after each swim. b. Use rubbing alcohol or 2% acetic acid eardrops after every swim. c. Irrigate the ears with warm water and a bulb syringe after each swim. d. Rinse the ears with a warmed solution of mineral oil and hydrogen peroxide.

A By 2 to 4 weeks an infant can fixate on an object. By the age of 1 month, the infant should fixate and follow a bright light or toy.

A 2-week-old infant can fixate on an object but cannot follow a light or bright toy. The nurse would: a. Consider this a normal finding. b. Assess the pupillary light reflex for possible blindness. c. Continue with the examination, and assess visual fields. d. Expect that a 2-week-old infant should be able to fixate and follow an object.

A Despite the increased cardiac output, arterial blood pressure decreases in pregnancy because of peripheral vasodilatation. The blood pressure drops to its lowest point during the second trimester and then rises after that

A 25-year-old woman in her fifth month of pregnancy has a blood pressure of 100/70 mm Hg. In reviewing her previous examination, the nurse notes that her blood pressure in her second month was 124/80 mm Hg. In evaluating this change, what does the nurse know to be true? a. This decline in blood pressure is the result of peripheral vasodilatation and is an expected change. b. Because of increased cardiac output, the blood pressure should be higher at this time. c. This change in blood pressure is not an expected finding because it means a decrease in cardiac output. d. This decline in blood pressure means a decrease in circulating blood volume, which is dangerous for the fetus.

D The symptom of pain is difficult to quantify because of individual interpretation. With pain, adjectives should be avoided and the patient should be asked how the pain affects his or her daily activities. The other responses are not appropriate.

A 29-year-old woman tells the nurse that she has excruciating pain in her back. Which would be the nurses appropriate response to the womans statement? a. How does your family react to your pain? b. The pain must be terrible. You probably pinched a nerve. c. Ive had back pain myself, and it can be excruciating. d. How would you say the pain affects your ability to do your daily activities?

D These findings are consistent with mitral regurgitation. Its subjective findings include fatigue, palpitation, and orthopnea, and its objective findings are: (1) a thrill in systole at the apex; (2) a lift at the apex; (3) the apical impulse displaced down and to the left; (4) the S1 is diminished, the S2 is accentuated, and the S3 at the apex is often present; and (5) a pansystolic murmur that is often loud, blowing, best heard at the apex, and radiating well to the left axilla.

A 30-year-old woman with a history of mitral valve problems states that she has been very tired. She has started waking up at night and feels like her heart is pounding. During the assessment, the nurse palpates a thrill and lift at the fifth left intercostal space midclavicular line. In the same area, the nurse also auscultates a blowing, swishing sound right after the S1. These findings would be most consistent with: a. Heart failure. b. Aortic stenosis. c. Pulmonary edema. d. Mitral regurgitation.

A Otosclerosis is a common cause of conductive hearing loss in young adults between the ages of 20 and 40 years. Presbycusis is a type of hearing loss that occurs with aging. Trauma and frequent ear infections are not a likely cause of his hearing loss.

A 31-year-old patient tells the nurse that he has noticed a progressive loss in his hearing. He says that it does seem to help when people speak louder or if he turns up the volume of a television or radio. The most likely cause of his hearing loss is: a. Otosclerosis. b. Presbycusis. c. Trauma to the bones. d. Frequent ear infections.

B Recruitment is significant hearing loss occurring when speech is at low intensity, but sound actually becomes painful when the speaker repeats at a louder volume. The other responses are not correct.

A 31-year-old patient tells the nurse that he has noticed pain in his left ear when people speak loudly to him. The nurse knows that this finding: a. Is normal for people of his age. b. Is a characteristic of recruitment. c. May indicate a middle ear infection. d. Indicates that the patient has a cerumen impaction.

D Fordyce granules are small, isolated white or yellow papules on the mucosa of the cheek, tongue, and lips. These little sebaceous cysts are painless and are not significant. Chalky, white raised patches would indicate leukoplakia. In strep throat, the examiner would see tonsils that are bright red, swollen, and may have exudates or white spots.

A 32-year-old woman is at the clinic for little white bumps in my mouth. During the assessment, the nurse notes that she has a 0.5 cm white, nontender papule under her tongue and one on the mucosa of her right cheek. What would the nurse tell the patient? a. These spots indicate an infection such as strep throat. b. These bumps could be indicative of a serious lesion, so I will refer you to a specialist. c. This condition is called leukoplakia and can be caused by chronic irritation such as with smoking. d. These bumps are Fordyce granules, which are sebaceous cysts and are not a serious condition.

ANS: B The inguinal nodes in the groin drain most of the lymph of the lower extremities. With local inflammation, the nodes in that area become swollen and tender.

A 35-year-old man is seen in the clinic for an infection in his left foot. Which of these findings should the nurse expect to see during an assessment of this patient? a. Hard and fixed cervical nodes b. Enlarged and tender inguinal nodes c. Bilateral enlargement of the popliteal nodes d. Pelletlike nodes in the supraclavicular region

B Candidiasis is a white, cheesy, curdlike patch on the buccal mucosa and tongue. It scrapes off, leaving a raw, red surface that easily bleeds. It also occurs after the use of antibiotics or corticosteroids and in persons who are immunosuppressed.

A 40-year-old patient who has just finished chemotherapy for breast cancer tells the nurse that she is concerned about her mouth. During the assessment the nurse finds areas of buccal mucosa that are raw and red with some bleeding, as well as other areas that have a white, cheesy coating. The nurse recognizes that this abnormality is: a. Aphthous ulcers. b. Candidiasis. c. Leukoplakia. d. Koplik spots.

D The inclusion of cultural considerations in the health assessment is of paramount importance to gathering data that are accurate and meaningful and to intervening with culturally sensitive and appropriate care.

A 42-year-old patient of Asian descent is being seen at the clinic for an initial examination. The nurse knows that including cultural information in his health assessment is important to: a. Identify the cause of his illness. b. Make accurate disease diagnoses. c. Provide cultural health rights for the individual. d. Provide culturally sensitive and appropriate care.

C Paroxysmal nocturnal dyspnea (shortness of breath generally occurring at night) occurs with heart failure. Lying down increases the volume of intrathoracic blood, and the weakened heart cannot accommodate the increased load. Classically, the person awakens after 2 hours of sleep, arises, and flings open a window with the perception of needing fresh air.

A 45-year-old man is in the clinic for a routine physical examination. During the recording of his health history, the patient states that he has been having difficulty sleeping. Ill be sleeping great, and then I wake up and feel like I cant get my breath. The nurses best response to this would be: a. When was your last electrocardiogram? b. Its probably because its been so hot at night. c. Do you have any history of problems with your heart? d. Have you had a recent sinus infection or upper respiratory infection?

D Floaters are a common sensation with myopia or after middle age and are attributable to condensed vitreous fibers. Floaters or spots are not usually significant, but the acute onset of floaters may occur with retinal detachment.

A 52-year-old patient describes the presence of occasional floaters or spots moving in front of his eyes. The nurse should: a. Examine the retina to determine the number of floaters. b. Presume the patient has glaucoma and refer him for further testing. c. Consider these to be abnormal findings, and refer him to an ophthalmologist. d. Know that floaters are usually insignificant and are caused by condensed vitreous fibers.

D The reason for seeking care is a brief spontaneous statement in the persons own words that describes the reason for the visit. It states one (possibly two) signs or symptoms and their duration. It is enclosed in quotation marks to indicate the persons exact words.

A 59-year-old patient tells the nurse that he has ulcerative colitis. He has been having black stools for the last 24 hours. How would the nurse best document his reason for seeking care? a. J.M. is a 59-year-old man seeking treatment for ulcerative colitis. b. J.M. came into the clinic complaining of having black stools for the past 24 hours. c. J.M. is a 59-year-old man who states that he has ulcerative colitis and wants it checked. d. J.M. is a 59-year-old man who states that he has been having black stools for the past 24 hours.

C Ptosis is a drooping of the upper eyelid that would be apparent by observing the distance between the upper and lower eyelids. The confrontation test measures peripheral vision. Measuring near vision or the corneal light test does not check for ptosis.

A 60-year-old man is at the clinic for an eye examination. The nurse suspects that he has ptosis of one eye. How should the nurse check for this? a. Perform the confrontation test. b. Assess the individuals near vision. c. Observe the distance between the palpebral fissures. d. Perform the corneal light test, and look for symmetry of the light reflex.

C Ischemia is a deficient supply of oxygenated arterial blood to a tissue. A partial blockage creates an insufficient supply, and the ischemia may be apparent only during exercise when oxygen needs increase.

A 65-year-old patient is experiencing pain in his left calf when he exercises that disappears after resting for a few minutes. The nurse recognizes that this description is most consistent with _______ the left leg. a. Venous obstruction of b. Claudication due to venous abnormalities in c. Ischemia caused by a partial blockage of an artery supplying d. Ischemia caused by the complete blockage of an artery supplying

C The patient is experiencing paroxysmal nocturnal dyspneabeing awakened from sleep with shortness of breath and the need to be upright to achieve comfort.

A 65-year-old patient with a history of heart failure comes to the clinic with complaints of being awakened from sleep with shortness of breath. Which action by the nurse is most appropriate? a. Obtaining a detailed health history of the patients allergies and a history of asthma b. Telling the patient to sleep on his or her right side to facilitate ease of respirations c. Assessing for other signs and symptoms of paroxysmal nocturnal dyspnea d. Assuring the patient that paroxysmal nocturnal dyspnea is normal and will probably resolve within the next week

A Intermittent claudication feels like a cramp and is usually relieved by rest within 2 minutes. The other responses are not correct.

A 67-year-old patient states that he recently began to have pain in his left calf when climbing the 10 stairs to his apartment. This pain is relieved by sitting for approximately 2 minutes; then he is able to resume his activities. The nurse interprets that this patient is most likely experiencing: a. Claudication. b. Sore muscles. c. Muscle cramps. d. Venous insufficiency

A Macular degeneration is the most common cause of blindness. It is characterized by the loss of central vision. Cataracts would show lens opacity. Chronic open-angle glaucoma, the most common type of glaucoma, involves a gradual loss of peripheral vision. These findings are not consistent with vision that is considered normal at any age.

A 68-year-old woman is in the eye clinic for a checkup. She tells the nurse that she has been having trouble reading the paper, sewing, and even seeing the faces of her grandchildren. On examination, the nurse notes that she has some loss of central vision but her peripheral vision is normal. These findings suggest that she may have: a. Macular degeneration. b. Vision that is normal for someone her age. c. The beginning stages of cataract formation. d. Increased intraocular pressure or glaucoma.

A A person with heart failure often exhibits increased respiratory rate, shortness of breath on exertion, orthopnea, paroxysmal nocturnal dyspnea, nocturia, ankle edema, and pallor in light-skinned individuals. A patient with rasping cough, thick mucoid sputum, and wheezing may have bronchitis. Productive cough, dyspnea, weight loss, and dyspnea indicate tuberculosis; fever, dry nonproductive cough, and diminished breath sounds may indicate Pneumocystis jiroveci (P. carinii) pneumonia

A 70-year-old patient is being seen in the clinic for severe exacerbation of his heart failure. Which of these findings is the nurse most likely to observe in this patient? a. Shortness of breath, orthopnea, paroxysmal nocturnal dyspnea, and ankle edema b. Rasping cough, thick mucoid sputum, wheezing, and bronchitis c. Productive cough, dyspnea, weight loss, anorexia, and tuberculosis d. Fever, dry nonproductive cough, and diminished breath sounds

D As long as the femoral and popliteal veins remain intact, the superficial veins can be excised without harming circulation. The other responses are not correct.

A 70-year-old patient is scheduled for open-heart surgery. The surgeon plans to use the great saphenous vein for the coronary bypass grafts. The patient asks, What happens to my circulation when this vein is removed? The nurse should reply: a. Venous insufficiency is a common problem after this type of surgery. b. Oh, you have lots of veinsyou wont even notice that it has been removed. c. You will probably experience decreased circulation after the vein is removed. d. This vein can be removed without harming your circulation because the deeper veins in your leg are in good condition.

C Presbycusis is a type of hearing loss that occurs in 60% of those older than 65 years of age, even in those living in a quiet environment. This sensorineural loss is gradual and caused by nerve degeneration in the inner ear. Words sound garbled, and the ability to localize sound is also impaired. This communication dysfunction is accentuated when background noise is present.

A 70-year-old patient tells the nurse that he has noticed that he is having trouble hearing, especially in large groups. He says that he cant always tell where the sound is coming from and the words often sound mixed up. What might the nurse suspect as the cause for this change? a. Atrophy of the apocrine glands b. Cilia becoming coarse and stiff c. Nerve degeneration in the inner ear d. Scarring of the tympanic membrane

B A pathologic S4 is termed an atrial gallop or an S4 gallop. It occurs with decreased compliance of the ventricle and with systolic overload (afterload), including outflow obstruction to the ventricle (aortic stenosis) and systemic hypertension. A left-sided S4 occurs with these conditions and is heard best at the apex with the patient in the left lateral position.

A 70-year-old patient with a history of hypertension has a blood pressure of 180/100 mm Hg and a heart rate of 90 beats per minute. The nurse hears an extra heart sound at the apex immediately before the S1. The sound is heard only with the bell of the stethoscope while the patient is in the left lateral position. With these findings and the patients history, the nurse knows that this extra heart sound is most likely a(n): a. Split S1. b. Atrial gallop. c. Diastolic murmur. d. Summation sound.

A The interview usually takes longer with older adults because they have a longer story to tell. It is not necessarily true that all older adults are lonely, have lost mental abilities, or are hard of hearing.

A 75-year-old woman is at the office for a preoperative interview. The nurse is aware that the interview may take longer than interviews with younger persons. What is the reason for this? a. An aged person has a longer story to tell. b. An aged person is usually lonely and likes to have someone with whom to talk. c. Aged persons lose much of their mental abilities and require longer time to complete an interview. d. As a person ages, he or she is unable to hear; thus the interviewer usually needs to repeat much of what is said.

D The person may not know the drug name or purpose. When this occurs, ask the person or a family member to bring in the drug to be identified. The other responses would not help to identify the medications.

A 90-year-old patient tells the nurse that he cannot remember the names of the medications he is taking or for what reason he is taking them. An appropriate response from the nurse would be: a. Can you tell me what they look like? b. Dont worry about it. You are only taking two medications. c. How long have you been taking each of the pills? d. Would you have a family member bring in your medications?

C Dysphagia is difficulty with swallowing and may occur with a variety of disorders, including stroke and other neurologic diseases. Rhinorrhea is a runny nose, epistaxis is a bloody nose, and xerostomia is a dry mouth.

A 92-year-old patient has had a stroke. The right side of his face is drooping. The nurse might also suspect which of these assessment findings? a. Epistaxis b. Rhinorrhea c. Dysphagia d. Xerostomia

D The nurses response must make it clear that she is a health professional who can best care for the person by maintaining a professional relationship. At the same time, the nurse should communicate that he or she accepts the person and understands the persons need to be self-assertive but that sexual advances cannot be tolerated.

A female nurse is interviewing a male patient who is near the same age as the nurse. During the interview, the patient makes an overtly sexual comment. The nurses best reaction would be: a. Stop that immediately! b. Oh, you are too funny. Lets keep going with the interview. c. Do you really think I would be interested? d. It makes me uncomfortable when you talk that way. Please stop.

A Both the patients and the nurses sense of spatial distance are significant throughout the interview and physical examination, with culturally appropriate distance zones varying widely. Some cultural groups value close physical proximity and may perceive a health care provider who is distancing him or herself as being aloof and unfriendly.

A female nurse is interviewing a man who has recently immigrated. During the course of the interview, he leans forward and then finally moves his chair close enough that his knees are nearly touching the nurses knees. The nurse begins to feel uncomfortable with his proximity. Which statement most closely reflects what the nurse should do next? a. The nurse should try to relax; these behaviors are culturally appropriate for this person. b. The nurse should discreetly move his or her chair back until the distance is more comfortable, and then continue with the interview. c. These behaviors are indicative of sexual aggression, and the nurse should confront this person about his behaviors. d. The nurse should laugh but tell him that he or she is uncomfortable with his proximity and ask him to move away.

B Obstetric history includes the number of pregnancies (gravidity), number of deliveries in which the fetus reached term (term), number of preterm pregnancies (preterm), number of incomplete pregnancies (abortions), and number of children living (living). This is recorded: Grav _____ Term _____ Preterm _____ Ab _____ Living _____. For any incomplete pregnancies, the duration is recorded and whether the pregnancy resulted in a spontaneous (S) or an induced (I) abortion.

A female patient tells the nurse that she has had six pregnancies, with four live births at term and two spontaneous abortions. Her four children are still living. How would the nurse record this information? a. P-6, B-4, (S)Ab-2 b. Grav 6, Term 4, (S)Ab-2, Living 4 c. Patient has had four living babies. d. Patient has been pregnant six times.

Bell Palsy

A lower motor neuron lesion (peripheral), producing rapid onset of cranial nerve VII paralysis of facial muscles; almost always unilateral. This may be a reactivation of herpes simplex virus (HSV-1) latent since childhood. Note complete paralysis of one-half of the face; person cannot wrinkle forehead, raise eyebrow, close eyelid, whistle, or show teeth on the left side. Usually present with smooth forehead, wide palpebral fissure, flat nasolabial fold, drooling, and pain behind the ear. This is greatly improved if corticosteroids and antivirals are given within 72 hours of onset.

D Open-ended questions are used for gathering narrative information. This type of questioning should be used to begin the interview, to introduce a new section of questions, and whenever the person introduces a new topic.

A man arrives at the clinic for his annual wellness physical. He is experiencing no acute health problems. Which question or statement by the nurse is most appropriate when beginning the interview? a. How is your family? b. How is your job? c. Tell me about your hypertension. d. How has your health been since your last visit?

D In the case of confrontation, a certain action, feeling, or statement has been observed, and the nurse now focuses the patients attention on it. The nurse should give honest feedback about what is seen or felt. Confrontation may focus on a discrepancy, or the nurse may confront the patient when parts of the story are inconsistent. The other statements are not appropriate.

A man has been admitted to the observation unit for observation after being treated for a large cut on his forehead. As the nurse works through the interview, one of the standard questions has to do with alcohol, tobacco, and drug use. When the nurse asks him about tobacco use, he states, I quit smoking after my wife died 7 years ago. However, the nurse notices an open pack of cigarettes in his shirt pocket. Using confrontation, the nurse could say: a. Mr. K., I know that you are lying. b. Mr. K., come on, tell me how much you smoke. c. Mr. K., I didnt realize your wife had died. It must be difficult for you at this time. Please tell me more about that. d. Mr. K., you have said that you dont smoke, but I see that you have an open pack of cigarettes in your pocket.

C Eye movements may be poorly coordinated at birth, but by 3 to 4 months of age, the infant should establish binocularity and should be able to fixate simultaneously on a single image with both eyes.

A mother asks when her newborn infants eyesight will be developed. The nurse should reply: a. Vision is not totally developed until 2 years of age. b. Infants develop the ability to focus on an object at approximately 8 months of age. c. By approximately 3 months of age, infants develop more coordinated eye movements and can fixate on an object. d. Most infants have uncoordinated eye movements for the first year of life.

B Although most of the communication is with the parent, the nurse should not completely ignore the child. Making contact will help ease the toddler later during the physical examination. The nurse should begin by asking about the toys the child is playing with or about a special doll or teddy bear brought from home. Does your doll have a name? or What can your truck do? Stoop down to meet the child at his or her eye level.

A mother brings her 28-month-old daughter into the clinic for a well-child visit. At the beginning of the visit, the nurse focuses attention away from the toddler, but as the interview progresses, the toddler begins to warm up and is smiling shyly at the nurse. The nurse will be most successful in interacting with the toddler if which is done next? a. Tickle the toddler, and get her to laugh. b. Stoop down to her level, and ask her about the toy she is holding. c. Continue to ignore her until it is time for the physical examination. d. Ask the mother to leave during the examination of the toddler, because toddlers often fuss less if their parent is not in view.

B The infant is an obligatory nose breather until the age of 3 months. Normally, no flaring of the nostrils and no sternal or intercostal retraction occurs. Significant retractions of the sternum and intercostal muscles and nasal flaring indicate increased inspiratory effort, as in pneumonia, acute airway obstruction, asthma, and atelectasis; therefore, immediate referral to the physician is warranted. These signs do not indicate heart failure, and an assessment of the infants feeding is not a priority at this time.

A mother brings her 3-month-old infant to the clinic for evaluation of a cold. She tells the nurse that he has had a runny nose for a week. When performing the physical assessment, the nurse notes that the child has nasal flaring and sternal and intercostal retractions. The nurses next action should be to: a. Assure the mother that these signs are normal symptoms of a cold. b. Recognize that these are serious signs, and contact the physician. c. Ask the mother if the infant has had trouble with feedings. d. Perform a complete cardiac assessment because these signs are probably indicative of early heart failure.

D A normal finding in infants is the sucking tubercle, a small pad in the middle of the upper lip from the friction of breastfeeding or bottle-feeding. This condition is not caused by irritation, teething, or excessive drooling, and evaluation by another health care provider is not warranted.

A mother brings her 4-month-old infant to the clinic with concerns regarding a small pad in the middle of the upper lip that has been there since 1 month of age. The infant has no health problems. On physical examination, the nurse notices a 0.5-cm, fleshy, elevated area in the middle of the upper lip. No evidence of inflammation or drainage is observed. What would the nurse tell this mother? a. This area of irritation is caused from teething and is nothing to worry about. b. This finding is abnormal and should be evaluated by another health care provider. c. This area of irritation is the result of chronic drooling and should resolve within the next month or two. d. This elevated area is a sucking tubercle caused from the friction of breastfeeding or bottle-feeding and is normal.

When assessing muscle strength, the nurse observes that a patient has complete range of motion against gravity with full resistance. What grade of muscle strength should the nurse record using a 0- to 5-point scale? a. 2 b. 3 c. 4 d. 5

ANS: D Complete range of motion against gravity is normal muscle strength and is recorded as grade 5 muscle strength. The other options are not correct.

C The guidelines for the number of teeth for children younger than 2 years old are as follows: the childs age in months minus the number 6 should be equal to the expected number of deciduous teeth. Normally, all 20 teeth are in by 2 years old. In this instance, the child is 18 months old, minus 6, equals 12 deciduous teeth expected.

A mother is concerned because her 18-month-old toddler has 12 teeth. She is wondering if this is normal for a child of this age. The nurses best response would be: a. How many teeth did you have at this age? b. All 20 deciduous teeth are expected to erupt by age 4 years. c. This is a normal number of teeth for an 18 month old. d. Normally, by age 2 years, 16 deciduous teeth are expected.

A First-level priority problems are immediate priorities, remembering the ABCs (airway, breathing, and circulation), followed by second-level problems, and then third-level problems.

A newly admitted patient is in acute pain, has not been sleeping well lately, and is having difficulty breathing. How should the nurse prioritize these problems? a. Breathing, pain, and sleep b. Breathing, sleep, and pain c. Sleep, breathing, and pain d. Sleep, pain, and breathing

C This question is an example of using leading or biased questions. Asking, You dont smoke, do you? implies that one answer is better than another. If the person wants to please someone, then he or she is either forced to answer in a way that corresponds to his or her implied values or is made to feel guilty when admitting the other answer.

A nurse is taking complete health histories on all of the patients attending a wellness workshop. On the history form, one of the written questions asks, You dont smoke, drink, or take drugs, do you? This question is an example of: a. Talking too much. b. Using confrontation. c. Using biased or leading questions. d. Using blunt language to deal with distasteful topics.

C Signs of maxillary sinusitis include facial pain after upper respiratory infection, red swollen nasal mucosa, swollen turbinates, and purulent discharge. The person also has fever, chills, and malaise. With maxillary sinusitis, dull throbbing pain occurs in the cheeks and teeth on the same side, and pain with palpation is present. With frontal sinusitis, pain is above the supraorbital ridge.

A patient comes into the clinic complaining of facial pain, fever, and malaise. On examination, the nurse notes swollen turbinates and purulent discharge from the nose. The patient also complains of a dull, throbbing pain in his cheeks and teeth on the right side and pain when the nurse palpates the areas. The nurse recognizes that this patient has: a. Posterior epistaxis. b. Frontal sinusitis. c. Maxillary sinusitis. d. Nasal polyps.

B A hordeolum, or stye, is a painful, red, and swollen pustule at the lid margin. A chalazion is a nodule protruding on the lid, toward the inside, and is nontender, firm, with discrete swelling. Dacryocystitis is an inflammation of the lacrimal sac. Blepharitis is inflammation of the eyelids

A patient comes into the clinic complaining of pain in her right eye. On examination, the nurse sees a pustule at the lid margin that is painful to touch, red, and swollen. The nurse recognizes that this is a: a. Chalazion. b. Hordeolum (stye). c. Dacryocystitis. d. Blepharitis.

D A corneal abrasion causes irregular ridges in reflected light, which produce a shattered appearance to light rays. No opacities should be observed in the cornea. The other responses are not correct.

A patient comes into the emergency department after an accident at work. A machine blew dust into his eyes, and he was not wearing safety glasses. The nurse examines his corneas by shining a light from the side across the cornea. What findings would suggest that he has suffered a corneal abrasion? a. Smooth and clear corneas b. Opacity of the lens behind the cornea c. Bleeding from the areas across the cornea d. Shattered look to the light rays reflecting off the cornea

B A cough that primarily occurs at night may indicate postnasal drip or sinusitis. Exposure to irritants at work causes an afternoon or evening cough. Smokers experience early morning coughing. Coughing associated with acute illnesses such as pneumonia is continuous throughout the day.

A patient comes to the clinic complaining of a cough that is worse at night but not as bad during the day. The nurse recognizes that this cough may indicate: a. Pneumonia. b. Postnasal drip or sinusitis. c. Exposure to irritants at work. d. Chronic bronchial irritation from smoking.

B With retinal detachment, the person has shadows or diminished vision in one quadrant or one half of the visual field. The other responses are not signs of retinal detachment.

A patient comes to the emergency department after a boxing match, and his left eye is swollen almost shut. He has bruises on his face and neck. He says he is worried because he cant see well from his left eye. The physician suspects retinal damage. The nurse recognizes that signs of retinal detachment include: a. Loss of central vision. b. Shadow or diminished vision in one quadrant or one half of the visual field. c. Loss of peripheral vision. d. Sudden loss of pupillary constriction and accommodation.

B Night leg pain is common in aging adults and may indicate the ischemic rest pain of peripheral vascular disease. Alterations in arterial circulation cause pain that becomes worse with leg elevation and is eased when the extremity is dangled.

A patient complains of leg pain that wakes him at night. He states that he has been having problems with his legs. He has pain in his legs when they are elevated that disappears when he dangles them. He recently noticed a sore on the inner aspect of the right ankle. On the basis of this health history information, the nurse interprets that the patient is most likely experiencing: a. Pain related to lymphatic abnormalities. b. Problems related to arterial insufficiency. c. Problems related to venous insufficiency. d. Pain related to musculoskeletal abnormalities.

D An anteroposterior-to-transverse diameter ratio of 1:1 or barrel chest is observed in individuals with COPD because of hyperinflation of the lungs. The ribs are more horizontal, and the chest appears as if held in continuous inspiration. Neck muscles are hypertrophied from aiding in forced respiration. Chest expansion may be decreased but symmetric. Decreased tactile fremitus occurs from decreased transmission of vibrations.

A patient has a long history of chronic obstructive pulmonary disease (COPD). During the assessment, the nurse will most likely observe which of these? a. Unequal chest expansion b. Increased tactile fremitus c. Atrophied neck and trapezius muscles d. Anteroposterior-to-transverse diameter ratio of 1:1

D The pupillary light reflex is the normal constriction of the pupils when bright light shines on the retina. The other responses are not correct.

A patient has a normal pupillary light reflex. The nurse recognizes that this reflex indicates that: a. The eyes converge to focus on the light. b. Light is reflected at the same spot in both eyes. c. The eye focuses the image in the center of the pupil. d. Constriction of both pupils occurs in response to bright light.

C Any sudden loss of hearing in one or both ears that is not associated with an upper respiratory infection needs to be reported at once to the patients health care provider. Hearing loss associated with trauma is often sudden. Irrigating the ear or removing cerumen is not appropriate at this time.

A patient has been admitted after an accident at work. During the assessment, the patient is having trouble hearing and states, I dont know what the matter is. All of a sudden, I cant hear you out of my left ear! What should the nurse do next? a. Make note of this finding for the report to the next shift. b. Prepare to remove cerumen from the patients ear. c. Notify the patients health care provider. d. Irrigate the ear with rubbing alcohol.

C Hypoventilation is characterized by an irregular, shallow pattern, and can be caused by an overdose of narcotics or anesthetics. Bradypnea is slow breathing, with a rate less than 10 respirations per minute

A patient has been admitted to the emergency department for a suspected drug overdose. His respirations are shallow, with an irregular pattern, with a rate of 12 respirations per minute. The nurse interprets this respiration pattern as which of the following? a. Bradypnea b. Cheyne-Stokes respirations c. Hypoventilation d. Chronic obstructive breathing

C Findings for pulmonary embolism include chest pain that is worse on deep inspiration, dyspnea, apprehension, anxiety, restlessness, partial arterial pressure of oxygen (PaO2) less than 80 mm Hg, diaphoresis, hypotension, crackles, and wheezes.

A patient has been admitted to the emergency department with a possible medical diagnosis of pulmonary embolism. The nurse expects to see which assessment findings related to this condition? a. Absent or decreased breath sounds b. Productive cough with thin, frothy sputum c. Chest pain that is worse on deep inspiration and dyspnea d. Diffuse infiltrates with areas of dullness upon percussion

A, B, E Patients with chronic arterial symptoms often have a history of smoking and diabetes (among other risk factors). The pain has a gradual onset with exertion and is relieved with rest or dangling. The skin appears cool and pale

A patient has been admitted with chronic arterial symptoms. During the assessment, the nurse should expect which findings? Select all that apply. a. Patient has a history of diabetes and cigarette smoking. b. Skin of the patient is pale and cool. c. His ankles have two small, weeping ulcers. d. Patient works long hours sitting at a computer desk. e. He states that the pain gets worse when walking. f. Patient states that the pain is worse at the end of the day.

C Untreated strep throat may lead to rheumatic fever. When performing a health history, the patient should be asked whether his or her sore throat has been documented as streptococcal.

A patient has been diagnosed with strep throat. The nurse is aware that without treatment, which complication may occur? a. Rubella b. Leukoplakia c. Rheumatic fever d. Scarlet fever

D A brown discoloration occurs with chronic venous stasis as a result of hemosiderin deposits (a by-product of red blood cell degradation). Pallor, cyanosis, atrophic skin, and unilateral coolness are all signs associated with arterial problems.

A patient has been diagnosed with venous stasis. Which of these findings would the nurse most likely observe? a. Unilateral cool foot b. Thin, shiny, atrophic skin c. Pallor of the toes and cyanosis of the nail beds d. Brownish discoloration to the skin of the lower leg

C A simple increase in amplitude may not enable the person to understand spoken words. Sensorineural hearing loss may be caused by presbycusis, which is a gradual nerve degeneration that occurs with aging and by ototoxic drugs, which affect the hair cells in the cochlea.

A patient has been shown to have a sensorineural hearing loss. During the assessment, it would be important for the nurse to: a. Speak loudly so the patient can hear the questions. b. Assess for middle ear infection as a possible cause. c. Ask the patient what medications he is currently taking. d. Look for the source of the obstruction in the external ear.

D The nurse should use direct questions after the persons opening narrative to fill in any details he or she left out. The nurse also should use direct questions when specific facts are needed, such as when asking about past health problems or during the review of systems.

A patient has finished giving the nurse information about the reason he is seeking care. When reviewing the data, the nurse finds that some information about past hospitalizations is missing. At this point, which statement by the nurse would be most appropriate to gather these data? a. Mr. Y., at your age, surely you have been hospitalized before! b. Mr. Y., I just need permission to get your medical records from County Medical. c. Mr. Y., you mentioned that you have been hospitalized on several occasions. Would you tell me more about that? d. Mr. Y., I just need to get some additional information about your past hospitalizations. When was the last time you were admitted for chest pain?

A Unilateral edema occurs with occlusion of a deep vein and with unilateral lymphatic obstruction. With these factors, the edema is nonpitting and feels hard to the touch (brawny edema).

A patient has hard, nonpitting edema of the left lower leg and ankle. The right leg has no edema. Based on these findings, the nurse recalls that: a. Nonpitting, hard edema occurs with lymphatic obstruction. b. Alterations in arterial function will cause edema. c. Phlebitis of a superficial vein will cause bilateral edema. d. Long-standing arterial obstruction will cause pitting edema.

B If maternal rubella infection occurs during the first trimester, then it can damage the organ of Corti and impair hearing.

A patient in her first trimester of pregnancy is diagnosed with rubella. Which of these statements is correct regarding the significance of this in relation to the infants hearing? a. Rubella may affect the mothers hearing but not the infants. b. Rubella can damage the infants organ of Corti, which will impair hearing. c. Rubella is only dangerous to the infant in the second trimester of pregnancy. d. Rubella can impair the development of CN VIII and thus affect hearing.

A A follow-up data base is used in all settings to follow up short-term or chronic health problems. The other responses are not appropriate for the situation.

A patient is at the clinic to have her blood pressure checked. She has been coming to the clinic weekly since she changed medications 2 months ago. The nurse should: a. Collect a follow-up data base and then check her blood pressure. b. Ask her to read her health record and indicate any changes since her last visit. c. Check only her blood pressure because her complete health history was documented 2 months ago. d. Obtain a complete health history before checking her blood pressure because much of her history information may have changed.

B The emergency data base calls for a rapid collection of the data base, often concurrently compiled with life-saving measures. The other responses are not appropriate for the situation.

A patient is brought by ambulance to the emergency department with multiple traumas received in an automobile accident. He is alert and cooperative, but his injuries are quite severe. How would the nurse proceed with data collection? a. Collect history information first, then perform the physical examination and institute life-saving measures. b. Simultaneously ask history questions while performing the examination and initiating life-saving measures. c. Collect all information on the history form, including social support patterns, strengths, and coping patterns. d. Perform life-saving measures and delay asking any history questions until the patient is transferred to the intensive care unit.

D The setting describes where the person is or what the person is doing when the symptom starts. Describing the pain as sharp and burning reflects the character or quality of the pain; stating that the pain is telling the patient that something bad is wrong with him reflects the patients perception of the pain; and describing the sweats and nausea reflects associated factors that occur with the pain.

A patient is describing his symptoms to the nurse. Which of these statements reflects a description of the setting of his symptoms? a. It is a sharp, burning pain in my stomach. b. I also have the sweats and nausea when I feel this pain. c. I think this pain is telling me that something bad is wrong with me. d. This pain happens every time I sit down to use the computer.

A, D, E Signs and symptoms of acute venous problems include pain in the calf that has a sudden onset and that is intense and sharp with tenderness in the deep muscle when touched. The calf is warm, red, and swollen. The other options are symptoms of chronic venous problems.

A patient is recovering from several hours of orthopedic surgery. During an assessment of the patients lower legs, the nurse will monitor for signs of acute venous symptoms. Signs of acute venous symptoms include which of the following? Select all that apply. a. Intense, sharp pain, with the deep muscle tender to the touch b. Aching, tired pain, with a feeling of fullness c. Pain that is worse at the end of the day d. Sudden onset e. Warm, red, and swollen calf f. Pain that is relieved with elevation of the leg

D If the person is unable to see even the largest letters when standing 20 feet from the chart, then the nurse should shorten the distance to the chart until the letters are seen, and record that distance (e.g., 10/200). If visual acuity is even lower, then the nurse should assess whether the person can count fingers when they are spread in front of the eyes or can distinguish light perception from a penlight. If vision is poorer than 20/30, then a referral to an ophthalmologist or optometrist is necessary, but the nurse must first assess the visual acuity.

A patient is unable to read even the largest letters on the Snellen chart. The nurse should take which action next? a. Refer the patient to an ophthalmologist or optometrist for further evaluation. b. Assess whether the patient can count the nurses fingers when they are placed in front of his or her eyes. c. Ask the patient to put on his or her reading glasses and attempt to read the Snellen chart again. d. Shorten the distance between the patient and the chart until the letters are seen, and record that distance.

D Note both the allergen (medication, food, or contact agent, such as fabric or environmental agent) and the reaction (rash, itching, runny nose, watery eyes, or difficulty breathing). With a drug, this symptom should not be a side effect but a true allergic reaction.

A patient tells the nurse that he is allergic to penicillin. What would be the nurses best response to this information? a. Are you allergic to any other drugs? b. How often have you received penicillin? c. Ill write your allergy on your chart so you wont receive any penicillin. d. Describe what happens to you when you take penicillin.

C Subjective data are what the person says about him or herself during history taking. Objective data arintrospective are not used to describe data.e what the health professional observes by inspecting, percussing, palpating, and auscultating during the physical examination. The terms reflective and

A patient tells the nurse that he is very nervous, is nauseated, and feels hot. These types of data would be: a. Objective. b. Reflective. c. Subjective. d. Introspective.

A A final summary of any symptom the person has should include, along with seven other critical characteristics, Location: specific. The person is asked to point to the location.

A patient tells the nurse that she has had abdominal pain for the past week. What would be the nurses best response? a. Can you point to where it hurts? b. Well talk more about that later in the interview. c. What have you had to eat in the last 24 hours? d. Have you ever had any surgeries on your abdomen?

C Among its other functions, the middle ear conducts sound vibrations from the outer ear to the central hearing apparatus in the inner ear. The other responses are not functions of the middle ear.

A patient with a middle ear infection asks the nurse, What does the middle ear do? The nurse responds by telling the patient that the middle ear functions to: a. Maintain balance. b. Interpret sounds as they enter the ear. c. Conduct vibrations of sounds to the inner ear. d. Increase amplitude of sound for the inner ear to function.

B A patient with pleuritis will exhibit a pleural friction rub upon auscultation. This sound is made when the pleurae become inflamed and rub together during respiration. The sound is superficial, coarse, and low-pitched, as if two pieces of leather are being rubbed together. Stridor is associated with croup, acute epiglottitis in children, and foreign body inhalation. Crackles are associated with pneumonia, heart failure, chronic bronchitis, and other diseases (see Table 18-6). Wheezes are associated with diffuse airway obstruction caused by acute asthma or chronic emphysema.

A patient with pleuritis has been admitted to the hospital and complains of pain with breathing. What other key assessment finding would the nurse expect to find upon auscultation? a. Stridor b. Friction rub c. Crackles d. Wheezing

B The top number indicates the distance the person is standing from the chart; the denominator gives the distance at which a normal eye can see.

A patients vision is recorded as 20/30 when the Snellen eye chart is used. The nurse interprets these results to indicate that: a. At 30 feet the patient can read the entire chart. b. The patient can read at 20 feet what a person with normal vision can read at 30 feet. c. The patient can read the chart from 20 feet in the left eye and 30 feet in the right eye. d. The patient can read from 30 feet what a person with normal vision can read from 20 feet.

B By providing false assurance or reassurance, this courage builder relieves the womans anxiety and gives the nurse the false sense of having provided comfort. However, for the woman, providing false assurance or reassurance actually closes off communication, trivializes her anxiety, and effectively denies any further talk of it.

A pregnant woman states, I just know labor will be so painful that I wont be able to stand it. I know it sounds awful, but I really dread going into labor. The nurse responds by stating, Oh, dont worry about labor so much. I have been through it, and although it is painful, many good medications are available to decrease the pain. Which statement is true regarding this response? The nurses reply was a: a. Therapeutic response. By sharing something personal, the nurse gives hope to this woman. b. Nontherapeutic response. By providing false reassurance, the nurse actually cut off further discussion of the womans fears. c. Therapeutic response. By providing information about the medications available, the nurse is giving information to the woman. d. Nontherapeutic response. The nurse is essentially giving the message to the woman that labor cannot be tolerated without medication.

The corona is

A shoulder where the glans joins the shaft The corona is a shoulder where the glans joins the shaft. Over the glans, the skin folds in and back on itself forming a hood or flap called the foreskin or prepuce. The penis is composed of three cylindrical columns of erectile tissue two corpora cavernosa on the dorsal side and the corpus spongiosum ventrally. The scrotal wall consists of thin skin lying in folds, or rugae, and the underlying cremaster muscle.

B With a pneumothorax, free air in the pleural space causes partial or complete lung collapse. If the pneumothorax is large, then tachypnea and cyanosis are evident. Unequal chest expansion, decreased or absent tactile fremitus, tracheal deviation to the unaffected side, decreased chest expansion, hyperresonant percussion tones, and decreased or absent breath sounds are found with the presence of pneumothorax

A teenage patient comes to the emergency department with complaints of an inability to breathe and a sharp pain in the left side of his chest. The assessment findings include cyanosis, tachypnea, tracheal deviation to the right, decreased tactile fremitus on the left, hyperresonance on the left, and decreased breath sounds on the left. The nurse interprets that these assessment findings are consistent with: a. Bronchitis. b. Pneumothorax. c. Acute pneumonia. d. Asthmatic attack.

C The complete data base is collected in a primary care setting, such as a pediatric or family practice clinic, independent or group private practice, college health service, womens health care agency, visiting nurse agency, or community health agency. In these settings, the nurse is the first health professional to see the patient and has the primary responsibility for monitoring the persons health care.

A visiting nurse is making an initial home visit for a patient who has many chronic medical problems. Which type of data base is most appropriate to collect in this setting? a. A follow-up data base to evaluate changes at appropriate intervals b. An episodic data base because of the continuing, complex medical problems of this patient c. A complete health data base because of the nurses primary responsibility for monitoring the patients health d. An emergency data base because of the need to collect information and make accurate diagnoses rapidly

D Address the person by using his or her surname. The nurse should introduce him or herself and give the reason for the interview. Friendly small talk is not needed to build rapport.

A woman has just entered the emergency department after being battered by her husband. The nurse needs to get some information from her to begin treatment. What is the best choice for an opening phase of the interview with this patient? a. Hello, Nancy, my name is Mrs. C. b. Hello, Mrs. H., my name is Mrs. C. It sure is cold today! c. Mrs. H., my name is Mrs. C. How are you? d. Mrs. H., my name is Mrs. C. Ill need to ask you a few questions about what happened.

B The nurse should use clarification when the persons word choice is ambiguous or confusing (e.g., Tell me what you mean by fits.). Clarification is also used to summarize the persons words or to simplify the words to make them clearer; the nurse should then ask if he or she is on the right track.

A woman is discussing the problems she is having with her 2-year-old son. She says, He wont go to sleep at night, and during the day he has several fits. I get so upset when that happens. The nurses best verbal response would be: a. Go on, Im listening. b. Fits? Tell me what you mean by this. c. Yes, it can be upsetting when a child has a fit. d. Dont be upset when he has a fit; every 2 year old has fits.

B Nasal stuffiness and epistaxis may occur during pregnancy as a result of increased vascularity in the upper respiratory tract.

A woman who is in the second trimester of pregnancy mentions that she has had more nosebleeds than ever since she became pregnant. The nurse recognizes that this is a result of: a. A problem with the patients coagulation system. b. Increased vascularity in the upper respiratory tract as a result of the pregnancy. c. Increased susceptibility to colds and nasal irritation. d. Inappropriate use of nasal sprays.

In a person with an upper motor neuron lesion such as a cerebrovascular accident, which of these physical assessment findings should the nurse expect? a. Hyperreflexia b. Fasciculations c. Loss of muscle tone and flaccidity d. Atrophy and wasting of the muscles

ANS: A Hyperreflexia, diminished or absent superficial reflexes, and increased muscle tone or spasticity can be expected with upper motor neuron lesions. The other options reflect a lesion of lower motor neurons (see Table 23-7)

A woman who has lived in the United States for a year after moving from Europe has learned to speak English and is almost finished with her college studies. She now dresses like her peers and says that her family in Europe would hardly recognize her. This nurse recognizes that this situation illustrates which concept? a. Assimilation b. Heritage consistency c. Biculturalism d. Acculturation

ANS: A Assimilation is the process by which a person develops a new cultural identity and becomes like members of the dominant culture. This concept does not reflect heritage consistency. Biculturalism is a dual pattern of identification; acculturation is the process of adapting to and acquiring another culture.

A woman has come to the clinic to seek help with a substance abuse problem. She admits to using cocaine just before arriving. Which of these assessment findings would the nurse expect to find when examining this woman? a. Dilated pupils, pacing, and psychomotor agitation b. Dilated pupils, unsteady gait, and aggressiveness c. Pupil constriction, lethargy, apathy, and dysphoria d. Constricted pupils, euphoria, and decreased temperature

ANS: A A cocaine users appearance includes pupillary dilation, tachycardia or bradycardia, elevated or lowered blood pressure, sweating, chills, nausea, vomiting, and weight loss. The persons behavior includes euphoria, talkativeness, hypervigilance, pacing, psychomotor agitation, impaired social or occupational functioning, fighting, grandiosity, and visual or tactile hallucinations.

The nurse is reviewing a patients medical record and notes that he is in a coma. Using the Glasgow Coma Scale, which number indicates that the patient is in a coma? a. 6 b. 12 c. 15 d. 24

ANS: A A fully alert, normal person has a score of 15, whereas a score of 7 or less reflects coma on the Glasgow Coma Scale (see Figure 23-59).

In the assessment of a 1-month-old infant, the nurse notices a lack of response to noise or stimulation. The mother reports that in the last week he has been sleeping all of the time, and when he is awake all he does is cry. The nurse hears that the infants cries are very high pitched and shrill. What should be the nurses appropriate response to these findings? a. Refer the infant for further testing. b. Talk with the mother about eating habits. c. Do nothing; these are expected findings for an infant this age. d. Tell the mother to bring the baby back in 1 week for a recheck.

ANS: A A high-pitched, shrill cry or cat-sounding screech occurs with central nervous system damage. Lethargy, hyporeactivity, and hyperirritability, as well as the parents report of significant changes in behavior all warrant referral. The other options are not correct responses.

The nurse is assessing bruising on an injured patient. Which color indicates a new bruise that is less than 2 hours old? a. Red b. Purple-blue c. Greenish-brown d. Brownish-yellow

ANS: A A new bruise is usually red and will often develop a purple or purple-blue appearance 12 to 36 hours after blunt-force trauma. The color of bruises (and ecchymoses) generally progresses from purple-blue to bluish-green to greenish-brown to brownish-yellow before fading away

A patient describes feeling an unreasonable, irrational fear of snakes. His fear is so persistent that he can no longer comfortably look at even pictures of snakes and has made an effort to identify all the places he might encounter a snake and avoids them. The nurse recognizes that he: a. Has a snake phobia. b. Is a hypochondriac; snakes are usually harmless. c. Has an obsession with snakes. d. Has a delusion that snakes are harmful, which must stem from an early traumatic incident involving snakes.

ANS: A A phobia is a strong, persistent, irrational fear of an object or situation; the person feels driven to avoid it

A 21-year-old patient has a head injury resulting from trauma and is unconscious. There are no other injuries. During the assessment what would the nurse expect to find when testing the patients deep tendon reflexes? a. Reflexes will be normal. b. Reflexes cannot be elicited. c. All reflexes will be diminished but present. d. Some reflexes will be present, depending on the area of injury.

ANS: A A reflex is a defense mechanism of the nervous system. It operates below the level of conscious control and permits a quick reaction to potentially painful or damaging situations.

To assess the head control of a 4-month-old infant, the nurse lifts up the infant in a prone position while supporting his chest. The nurse looks for what normal response? The infant: a. Raises the head, and arches the back. b. Extends the arms, and drops down the head. c. Flexes the knees and elbows with the back straight. d. Holds the head at 45 degrees, and keeps the back straight.

ANS: A At 3 months of age, the infant raises the head and arches the back as if in a swan dive. This response is the Landau reflex, which persists until 1 years of age (see Figure 23-43). The other responses are incorrect.

A patient states, I can hear a crunching or grating sound when I kneel. She also states that it is very difficult to get out of bed in the morning because of stiffness and pain in my joints. The nurse should assess for signs of what problem? a. Crepitation b. Bone spur c. Loose tendon d. Fluid in the knee joint

ANS: A Crepitation is an audible and palpable crunching or grating that accompanies movement and occurs when articular surfaces in the joints are roughened, as with rheumatoid arthritis. The other options are not correct.

A 30-year-old woman has recently moved to the United States with her husband. They are living with the womans sister until they can get a home of their own. When company arrives to visit with the womans sister, the woman feels suddenly shy and retreats to the back bedroom to hide until the company leaves. She explains that her reaction to guests is simply because she does not know how to speak perfect English. This woman could be experiencing: a. Culture shock. b. Cultural taboos. c. Cultural unfamiliarity. d. Culture disorientation.

ANS: A Culture shock is a term used to describe the state of disorientation or inability to respond to the behavior of a different cultural group because of its sudden strangeness, unfamiliarity, and incompatibility with the individuals perceptions and expectations. The other terms are not correct.

When documenting IPV and elder abuse, the nurse should include: a. Photographic documentation of the injuries. b. Summary of the abused patients statements. c. Verbatim documentation of every statement made. d. General description of injuries in the progress notes.

ANS: A Documentation of IPV and elder abuse must include detailed nonbiased progress notes, the use of injury maps, and photographic documentation. Written documentation needs to be verbatim, within reason. Not every statement can be documented.

A 59-year-old patient has been diagnosed with prostatitis and is being seen at the clinic for complaints of burning and pain during urination. He is experiencing: a. Dysuria. b. Nocturia. c. Polyuria. d. Hematuria

ANS: A Dysuria (burning with urination) is common with acute cystitis, prostatitis, and urethritis. Nocturia is voiding during the night. Polyuria is voiding in excessive quantities. Hematuria is voiding with blood in the urine.

During morning rounds, the nurse asks a patient, How are you today? The patient responds, You today, you today, you today! and mumbles the words. This speech pattern is an example of: a. Echolalia b. Clanging c. Word salad d. Perseveration

ANS: A Echolalia occurs when a person imitates or repeats anothers words or phrases, often with a mumbling, mocking, or a mechanical tone.

The nurse is assessing a patient who has been admitted for cirrhosis of the liver, secondary to chronic alcohol use. During the physical assessment, the nurse looks for cardiac problems that are associated with chronic use of alcohol, such as: a. Hypertension. b. Ventricular fibrillation. c. Bradycardia. d. Mitral valve prolapse.

ANS: A Even moderate drinking leads to hypertension and cardiomyopathy, with an increase in left ventricular mass, dilation of ventricles, and wall thinning. Ventricular fibrillation, bradycardia, and mitral valve prolapse are not associated with chronic heavy use of alcohol

The nurse is examining a 6-month-old infant and places the infants feet flat on the table and flexes his knees up. The nurse notes that the right knee is significantly lower than the left. Which of these statements is true of this finding? a. This finding is a positive Allis sign and suggests hip dislocation. b. The infant probably has a dislocated patella on the right knee. c. This finding is a negative Allis sign and normal for an infant of this age. d. The infant should return to the clinic in 2 weeks to see if his condition has changed.

ANS: A Finding one knee significantly lower than the other is a positive Allis sign and suggests hip dislocation. Normally, the tops of the knees are at the same elevation. The other statements are not correct.

A patient tells the nurse that she is having a hard time bringing her hand to her mouth when she eats or tries to brush her teeth. The nurse knows that for her to move her hand to her mouth, she must perform which movement? a. Flexion b. Abduction c. Adduction d. Extension

ANS: A Flexion, or bending a limb at a joint, is required to move the hand to the mouth. Extension is straightening a limb at a joint. Moving a limb toward the midline of the body is called adduction; abduction is moving a limb away from the midline of the body.

The nurse discovers speech problems in a patient during an assessment. The patient has spontaneous speech, but it is mostly absent or is reduced to a few stereotypical words or sounds. This finding reflects which type of aphasia? a. Global b. Brocas c. Dysphonic d. Wernickes

ANS: A Global aphasia is the most common and severe form of aphasia. Spontaneous speech is absent or reduced to a few stereotyped words or sounds, and prognosis for language recovery is poor. (Brocas aphasia and Wernickes aphasia are described in Table 5-4.) Dysphonic aphasia is not a valid condition.

During reporting, the nurse hears that a patient is experiencing hallucinations. Which is an example of a hallucination? a. Man believes that his dead wife is talking to him. b. Woman hears the doorbell ring and goes to answer it, but no one is there. c. Child sees a man standing in his closet. When the lights are turned on, it is only a dry cleaning bag. d. Man believes that the dog has curled up on the bed, but when he gets closer he sees that it is a blanket.

ANS: A Hallucinations are sensory perceptions for which no external stimuli exist. They may strike any sense: visual, auditory, tactile, olfactory, or gustatory.

When reviewing the demographics of ethnic groups in the United States, the nurse recalls that the largest and fastest growing population is: a. Hispanic. b. Black. c. Asian. d. American Indian.

ANS: A Hispanics are the largest and fastest growing population in the United States, followed by Asians, Blacks, American Indians and Alaska natives, and other groups.

The nurse recognizes that working with children with a different cultural perspective may be especially difficult because: a. Children have spiritual needs that are influenced by their stages of development. b. Children have spiritual needs that are direct reflections of what is occurring in their homes. c. Religious beliefs rarely affect the parents perceptions of the illness. d. Parents are often the decision makers, and they have no knowledge of their childrens spiritual needs.

ANS: A Illness during childhood may be an especially difficult clinical situation. Children, as well as adults, have spiritual needs that vary according to the childs developmental level and the religious climate that exists in the family. The other statements are not correct.

An older man is concerned about his sexual performance. The nurse knows that in the absence of disease, a withdrawal from sexual activity later in life may be attributable to: a. Side effects of medications. b. Decreased libido with aging. c. Decreased sperm production. d. Decreased pleasure from sexual intercourse.

ANS: A In the absence of disease, a withdrawal from sexual activity may be attributable to side effects of medications such as antihypertensives, antidepressants, sedatives, psychotropics, antispasmotics, tranquilizers or narcotics, and estrogens. The other options are not correct.

A patient is admitted to the unit after an automobile accident. The nurse begins the mental status examination and finds that the patient has dysarthric speech and is lethargic. The nurses best approach regarding this examination is to: a. Plan to defer the rest of the mental status examination. b. Skip the language portion of the examination, and proceed onto assessing mood and affect. c. Conduct an in-depth speech evaluation, and defer the mental status examination to another time. d. Proceed with the examination, and assess the patient for suicidal thoughts because dysarthria is often accompanied by severe depression.

ANS: A In the mental status examination, the sequence of steps forms a hierarchy in which the most basic functions (consciousness, language) are assessed first. The first steps must be accurately assessed to ensure validity of the steps that follow. For example, if consciousness is clouded, then the person cannot be expected to have full attention and to cooperate with new learning. If language is impaired, then a subsequent assessment of new learning or abstract reasoning (anything that requires language functioning) can give erroneous conclusions.

The functional units of the musculoskeletal system are the: a. Joints. b. Bones. c. Muscles. d. Tendons.

ANS: A Joints are the functional units of the musculoskeletal system because they permit the mobility needed to perform the activities of daily living. The skeleton (bones) is the framework of the body. The other options are not correct

During a mental status examination, the nurse wants to assess a patients affect. The nurse should ask the patient which question? a. How do you feel today? b. Would you please repeat the following words? c. Have these medications had any effect on your pain? d. Has this pain affected your ability to get dressed by yourself?

ANS: A Judge mood and affect by body language and facial expression and by directly asking, How do you feel today? or How do you usually feel? The mood should be appropriate to the persons place and condition and should appropriately change with the topics.

A patient drifts off to sleep when she is not being stimulated. The nurse can easily arouse her by calling her name, but the patient remains drowsy during the conversation. The best description of this patients level of consciousness would be: a. Lethargic b. Obtunded c. Stuporous d. Semialert

ANS: A Lethargic (or somnolent) is when the person is not fully alert, drifts off to sleep when not stimulated, and can be aroused when called by name in a normal voice but looks drowsy. He or she appropriately responds to questions or commands, but thinking seems slow and fuzzy. He or she is inattentive and loses the train of thought. Spontaneous movements are decreased. (See Table 5-3 for the definitions of the other terms.)

A woman who is 8 months pregnant comments that she has noticed a change in her posture and is having lower back pain. The nurse tells her that during pregnancy, women have a posture shift to compensate for the enlarging fetus. This shift in posture is known as: a. Lordosis. b. Scoliosis. c. Ankylosis. d. Kyphosis.

ANS: A Lordosis compensates for the enlarging fetus, which would shift the center of balance forward. This shift in balance, in turn, creates a strain on the low back muscles, felt as low back pain during late pregnancy by some women. Scoliosis is lateral curvature of portions of the spine; ankylosis is extreme flexion of the wrist, as observed with severe rheumatoid arthritis; and kyphosis is an enhanced thoracic curvature of the spine.

To test for gross motor skill and coordination of a 6-year-old child, which of these techniques would be appropriate? Ask the child to: a. Hop on one foot. b. Stand on his head. c. Touch his finger to his nose. d. Make funny faces at the nurse.

ANS: A Normally, a child can hop on one foot and can balance on one foot for approximately 5 seconds by 4 years of age and can balance on one foot for 8 to 10 seconds at 5 years of age. Children enjoy performing these tests. Failure to hop after 5 years of age indicates incoordination of gross motor skills. Asking the child to touch his or her finger to the nose checks fine motor coordination; and asking the child to make funny faces tests CN VII. Asking a child to stand on his or her head is not appropriate.

When performing a genitourinary assessment, the nurse notices that the urethral meatus is ventrally positioned. This finding is: a. Called hypospadias. b. A result of phimosis. c. Probably due to a stricture. d. Often associated with aging.

ANS: A Normally, the urethral meatus is positioned just about centrally. Hypospadias is the ventral location of the urethral meatus. The position of the meatus does not change with aging. Phimosis is the inability to retract the foreskin. A stricture is a narrow opening of the meatus.

A woman brings her husband to the clinic for an examination. She is particularly worried because after a recent fall, he seems to have lost a great deal of his memory of recent events. Which statement reflects the nurses best course of action? a. Perform a complete mental status examination. b. Refer him to a psychometrician. c. Plan to integrate the mental status examination into the history and physical examination. d. Reassure his wife that memory loss after a physical shock is normal and will soon subside.

ANS: A Performing a complete mental status examination is necessary when any abnormality in affect or behavior is discovered or when family members are concerned about a persons behavioral changes (e.g., memory loss, inappropriate social interaction) or after trauma, such as a head injury.

While obtaining a health history of a 3-month-old infant from the mother, the nurse asks about the infants ability to suck and grasp the mothers finger. What is the nurse assessing? a. Reflexes b. Intelligence c. CNs d. Cerebral cortex function

ANS: A Questions regarding reflexes include such questions as, What have you noticed about the infants behavior, Are the infants sucking and swallowing seem coordinated, and Does the infant grasp your finger? The other responses are incorrect.

The nurse is inspecting the scrotum and testes of a 43-year-old man. Which finding would require additional follow-up and evaluation? a. Skin on the scrotum is taut. b. Left testicle hangs lower than the right testicle. c. Scrotal skin has yellowish 1-cm nodules that are firm and nontender. d. Testes move closer to the body in response to cold temperatures.

ANS: A Scrotal swelling may cause the skin to be taut and to display pitting edema. Normal scrotal skin is rugae, and asymmetry is normal with the left scrotal half usually lower than the right. The testes may move closer to the body in response to cold temperatures.

During the assessment of an 80-year-old patient, the nurse notices that his hands show tremors when he reaches for something and his head is always nodding. No associated rigidity is observed with movement. Which of these statements is most accurate? a. These findings are normal, resulting from aging. b. These findings could be related to hyperthyroidism. c. These findings are the result of Parkinson disease. d. This patient should be evaluated for a cerebellar lesion

ANS: A Senile tremors occasionally occur. These benign tremors include an intention tremor of the hands, head nodding (as if saying yes or no), and tongue protrusion. Tremors associated with Parkinson disease include rigidity, slowness, and a weakness of voluntary movement. The other responses are incorrect.

The nurse is assessing the mental status of a child. Which statement about children and mental status is true? a. All aspects of mental status in children are interdependent. b. Children are highly labile and unstable until the age of 2 years. c. Childrens mental status is largely a function of their parents level of functioning until the age of 7 years. d. A childs mental status is impossible to assess until the child develops the ability to concentrate.

ANS: A Separating and tracing the development of only one aspect of mental status is difficult. All aspects are interdependent. For example, consciousness is rudimentary at birth because the cerebral cortex is not yet developed. The infant cannot distinguish the self from the mothers body. The other statements are not true

The nurse is testing the deep tendon reflexes of a 30-year-old woman who is in the clinic for an annual physical examination. When striking the Achilles heel and quadriceps muscle, the nurse is unable to elicit a reflex. The nurses next response should be to: a. Ask the patient to lock her fingers and pull. b. Complete the examination, and then test these reflexes again. c. Refer the patient to a specialist for further testing. d. Document these reflexes as 0 on a scale of 0 to 4+

ANS: A Sometimes the reflex response fails to appear. Documenting the reflexes as absent is inappropriate this soon in the examination. The nurse should try to further encourage relaxation, varying the persons position or increasing the strength of the blow. Reinforcement is another technique to relax the muscles and enhance the response. The person should be asked to perform an isometric exercise in a muscle group somewhat away from the one being tested. For example, to enhance a patellar reflex, the person should be asked to lock the fingers together and pull.

The nurse is administering a Mini-Cog test to an older adult woman. When asked to draw a clock showing the time of 10:45, the patient drew a clock with the numbers out of order and with an incorrect time. This result indicates which finding? a. Cognitive impairment b. Amnesia c. Delirium d. Attention-deficit disorder

ANS: A The Mini-Cog is a newer instrument that screens for cognitive impairment, often found with dementia. The result of an abnormal drawing of a clock and time indicates a cognitive impairment

The ankle joint is the articulation of the tibia, fibula, and: a. Talus. b. Cuboid. c. Calcaneus. d. cuneiform bones

ANS: A The ankle or tibiotalar joint is the articulation of the tibia, fibula, and talus. The other bones listed are foot bones and not part of the ankle joint.

The nurse knows that determining whether a person is oriented to his or her surroundings will test the functioning of which structure(s)? a. Cerebrum b. Cerebellum c. CNs d. Medulla oblongata

ANS: A The cerebral cortex is responsible for thought, memory, reasoning, sensation, and voluntary movement. The other structures are not responsible for a persons level of consciousness.

The wife of a 65-year-old man tells the nurse that she is concerned because she has noticed a change in her husbands personality and ability to understand. He also cries very easily and becomes angry. The nurse recalls that the cerebral lobe responsible for these behaviors is the __________ lobe. a. Frontal b. Parietal c. Occipital d. Temporal

ANS: A The frontal lobe has areas responsible for personality, behavior, emotions, and intellectual function. The parietal lobe has areas responsible for sensation; the occipital lobe is responsible for visual reception; and the temporal lobe is responsible for hearing, taste, and smell.

The nurse is checking the range of motion in a patients knee and knows that the knee is capable of which movement(s)? a. Flexion and extension b. Supination and pronation c. Circumduction d. Inversion and eversion

ANS: A The knee is a hinge joint, permitting flexion and extension of the lower leg on a single plane. The knee is not capable of the other movements listed.

When performing a musculoskeletal assessment, the nurse knows that the correct approach for the examination should be: a. Proximal to distal. b. Distal to proximal. c. Posterior to anterior. d. Anterior to posterior.

ANS: A The musculoskeletal assessment should be performed in an orderly approach, head to toe, proximal to distal, from the midline outward. The other options are not correct.

Illness is considered part of lifes rhythmic course and is an outward sign of disharmony within. This statement most accurately reflects the views about illness from which theory? a. Naturalistic b. Biomedical c. Reductionist d. Magicoreligious

ANS: A The naturalistic perspective states that the laws of nature create imbalances, chaos, and disease. From the perspective of the Chinese, for example, illness is not considered an introducing agent; rather, illness is considered a part of lifes rhythmic course and an outward sign of disharmony within. The other options are not correct.

During an assessment, the nurse notices that a patient is handling a small charm that is tied to a leather strip around his neck. Which action by the nurse is appropriate? a. Ask the patient about the item and its significance. b. Ask the patient to lock the item with other valuables in the hospitals safe. c. Tell the patient that a family member should take valuables home. d. No action is necessary.

ANS: A The nurse should inquire about the amulets meaning. Amulets, such as charms, are often considered an important means of protection from evil spirits by some cultures.

Which of these statements is most appropriate when the nurse is obtaining a genitourinary history from an older man? a. Do you need to get up at night to urinate? b. Do you experience nocturnal emissions, or wet dreams? c. Do you know how to perform a testicular self-examination? d. Has anyone ever touched your genitals when you did not want them to?

ANS: A The older male patient should be asked about the presence of nocturia. Awaking at night to urinate may be attributable to a diuretic medication, fluid retention from mild heart failure or varicose veins, or fluid ingestion 3 hours before bedtime, especially coffee and alcohol. The other questions are more appropriate for younger men.

During an assessment of a 62-year-old man, the nurse notices the patient has a stooped posture, shuffling walk with short steps, flat facial expression, and pill-rolling finger movements. These findings would be consistent with: a. Parkinsonism. b. Cerebral palsy. c. Cerebellar ataxia. d. Muscular dystrophy.

ANS: A The stooped posture, shuffling walk, short steps, flat facial expression, and pill-rolling finger movements are all found in parkinsonism. (See Table 23-8 for more information and for the descriptions of the other options.)

Of the 33 vertebrae in the spinal column, there are: a. 5 lumbar. b. 5 thoracic. c. 7 sacral. d. 12 cervical.

ANS: A There are 7 cervical, 12 thoracic, 5 lumbar, 5 sacral, and 3 to 4 coccygeal vertebrae in the spinal column.

Which of these tests would the nurse use to check the motor coordination of an 11-month-old infant? a. Denver II b. Stereognosis c. Deep tendon reflexes d. Rapid alternating movements

ANS: A To screen gross and fine motor coordination, the nurse should use the Denver II with its age-specific developmental milestones. Stereognosis tests a persons ability to recognize objects by feeling them and is not appropriate for an 11-month-old infant. Testing the deep tendon reflexes is not appropriate for checking motor coordination. Testing rapid alternating movements is appropriate for testing coordination in adults.

During the taking of the health history, a patient tells the nurse that it feels like the room is spinning around me. The nurse would document this finding as: a. Vertigo. b. Syncope. c. Dizziness. d. Seizure activity.

ANS: A True vertigo is rotational spinning caused by a neurologic dysfunction or a problem in the vestibular apparatus or the vestibular nuclei in the brainstem. Syncope is a sudden loss of strength or a temporary loss of consciousness. Dizziness is a lightheaded, swimming sensation. Seizure activity is characterized by altered or loss of consciousness, involuntary muscle movements, and sensory disturbances.

A 59-year-old patient has a herniated intervertebral disk. Which of the following findings should the nurse expect to see on physical assessment of this individual? a. Hyporeflexia b. Increased muscle tone c. Positive Babinski sign d. Presence of pathologic reflexes

ANS: A With a herniated intervertebral disk or lower motor neuron lesion, loss of tone, flaccidity, atrophy, fasciculations, and hyporeflexia or areflexia are demonstrated. No Babinski sign or pathologic reflexes would be observed (see Table 23-7). The other options reflect a lesion of upper motor neurons.

During a physical examination, the nurse finds that a male patients foreskin is fixed and tight and will not retract over the glans. The nurse recognizes that this condition is: a. Phimosis. b. Epispadias. c. Urethral stricture. d. Peyronie disease.

ANS: A With phimosis, the foreskin is nonretractable, forming a pointy tip of the penis with a tiny orifice at the end of the glans. The foreskin is advanced and so tight that it is impossible to retract over the glans. This condition may be congenital or acquired from adhesions related to infection. (See Table 24-3 for information on urethral stricture. See Table 24-4 for information on epispadias and Peyronie disease.)

An Asian-American woman is experiencing diarrhea, which is believed to be cold or yin. The nurse expects that the woman is likely to try to treat it with: a. Foods that are hot or yang. b. Readings and Eastern medicine meditations. c. High doses of medicines believed to be cold. d. No treatment is tried because diarrhea is an expected part of life.

ANS: A Yin foods are cold and yang foods are hot. Cold foods are eaten with a hot illness, and hot foods are eaten with a cold illness. The other explanations do not reflect the yin/yang theory.

A patient visits the clinic to ask about smoking cessation. He has smoked heavily for 30 years and wants to stop cold turkey. He asks the nurse, What symptoms can I expect if I do this? Which of these symptoms should the nurse share with the patient as possible symptoms of nicotine withdrawal? Select all that apply. a. Headaches b. Hunger c. Sleepiness d. Restlessness e. Nervousness f. Sweating

ANS: A, B, D, E Symptoms of nicotine withdrawal include vasodilation, headaches, anger, irritability, frustration, anxiety, nervousness, awakening at night, difficulty concentrating, depression, hunger, impatience, and the desire to smoke

The nurse is assessing a patient who is admitted with possible delirium. Which of these are manifestations of delirium? Select all that apply. a. Develops over a short period. b. Person is experiencing apraxia. c. Person is exhibiting memory impairment or deficits. d. Occurs as a result of a medical condition, such as systemic infection. e. Person is experiencing agnosia.

ANS: A, C, D Delirium is a disturbance of consciousness that develops over a short period and may be attributable to a medical condition. Memory deficits may also occur. Apraxia and agnosia occur with dementia.

A patient tells the nurse that, All my life Ive been called knock knees. The nurse knows that another term for knock knees is: a. Genu varum. b. Genu valgum. c. Pes planus. d. Metatarsus adductus.

ANS: B Genu valgum is also known as knock knees and is present when more than 2.5 cm is between the medial malleoli when the knees are together.

The nurse assesses an older woman and suspects physical abuse. Which questions are appropriate for screening for abuse? Select all that apply. a. Has anyone made you afraid, touched you in ways that you did not want, or hurt you physically? b. Are you being abused? c. Have you relied on people for any of the following: bathing, dressing, shopping, banking, or meals? d. Have you been upset because someone talked to you in a way that made you feel shamed or threatened? e. Have you relied on people for any of the following: bathing, dressing, shopping, banking, or meals?

ANS: A, C, D, E Directly asking Are you being abused? is not an appropriate screening question for abuse because the woman could easily say no, and no further information would be obtained. The other questions are among the questions recommended by the Elder Abuse Suspicion Index (EASI) when screening for elder abuse.

The nurse is asking questions about a patients health beliefs. Which questions are appropriate? Select all that apply. a. What is your definition of health? b. Does your family have a history of cancer? c. How do you describe illness? d. What did your mother do to keep you from getting sick? e. Have you ever had any surgeries? f. How do you keep yourself healthy?

ANS: A, C, D, F The questions listed are appropriate questions for an assessment of a patients health beliefs and practices. The questions regarding family history and surgeries are part of the patients physical history, not the patients health beliefs.

A 16-year-old boy is brought to the clinic for a problem that he refused to let his mother see. The nurse examines him, and finds that he has scrotal swelling on the left side. He had the mumps the previous week, and the nurse suspects that he has orchitis. Which of the following assessment findings support this diagnosis? Select all that apply. a. Swollen testis b. Mass that transilluminates c. Mass that does not transilluminate d. Scrotum that is nontender upon palpation e. Scrotum that is tender upon palpation f. Scrotal skin that is reddened

ANS: A, C, E, F With orchitis, the testis is swollen, with a feeling of weight, and is tender or painful. The mass does not transilluminate, and the scrotal skin is reddened. Transillumination of a mass occurs with a hydrocele, not orchitis.

A 55-year-old man is in the clinic for a yearly checkup. He is worried because his father died of prostate cancer. The nurse knows which tests should be performed at this time? Select all that apply. a. Blood test for prostate-specific antigen (PSA) b. Urinalysis c. Transrectal ultrasound d. Digital rectal examination (DRE) e. Prostate biopsy

ANS: A, D Prostate cancer is typically detected by testing the blood for PSA or by a DRE. It is recommended that both PSA and DRE be offered to men annually, beginning at age 50 years. If the PSA is elevated, then further laboratory work or a transrectal ultrasound (TRUS) and biopsy may be recommended.

During a mental status assessment, which question by the nurse would best assess a persons judgment? a. Do you feel that you are being watched, followed, or controlled? b. Tell me what you plan to do once you are discharged from the hospital. c. What does the statement, People in glass houses shouldnt throw stones, mean to you? d. What would you do if you found a stamped, addressed envelope lying on the sidewalk?

ANS: B A person exercises judgment when he or she can compare and evaluate the alternatives in a situation and reach an appropriate course of action. Rather than testing the persons response to a hypothetical situation (as illustrated in the option with the envelope), the nurse should be more interested in the persons judgment about daily or long-term goals, the likelihood of acting in response to delusions or hallucinations, and the capacity for violent or suicidal behavior.

When performing a genital examination on a 25-year-old man, the nurse notices deeply pigmented, wrinkled scrotal skin with large sebaceous follicles. On the basis of this information, the nurse would: a. Squeeze the glans to check for the presence of discharge. b. Consider this finding as normal, and proceed with the examination. c. Assess the testicles for the presence of masses or painless lumps. d. Obtain a more detailed history, focusing on any scrotal abnormalities the patient has noticed.

ANS: B After adolescence, the scrotal skin is deeply pigmented and has large sebaceous follicles and appears corrugated.

A patient has been diagnosed with osteoporosis and asks the nurse, What is osteoporosis? The nurse explains that osteoporosis is defined as: a. Increased bone matrix. b. Loss of bone density. c. New, weaker bone growth. d. Increased phagocytic activity.

ANS: B After age 40 years, a loss of bone matrix (resorption) occurs more rapidly than new bone formation. The net effect is a gradual loss of bone density, or osteoporosis. The other options are not correct.

The nurse is reviewing theories of illness. The germ theory, which states that microscopic organisms such as bacteria and viruses are responsible for specific disease conditions, is a basic belief of which theory of illness? a. Holistic b. Biomedical c. Naturalistic d. Magicoreligious

ANS: B Among the biomedical explanations for disease is the germ theory, which states that microscopic organisms such as bacteria and viruses are responsible for specific disease conditions. The naturalistic, or holistic, perspective holds that the forces of nature must be kept in natural balance. The magicoreligious perspective holds that supernatural forces dominate and cause illness or health.

An imaginary line connecting the highest point on each iliac crest would cross the __________ vertebra. a. First sacral b. Fourth lumbar c. Seventh cervical d. Twelfth thoracic

ANS: B An imaginary line connecting the highest point on each iliac crest crosses the fourth lumbar vertebra. The other options are not correct.

The nurse is conducting a heritage assessment. Which question is most appropriate for this assessment? a. What is your religion? b. Do you mostly participate in the religious traditions of your family? c. Do you smoke? d. Do you have a history of heart disease?

ANS: B Asking questions about participation in the religious traditions of family enables the nurse to assess a persons heritage. Simply asking about ones religion, smoking history, or health history does not reflect heritage.

During an interview, a woman has answered yes to two of the Abuse Assessment Screen questions. What should the nurse say next? a. I need to report this abuse to the authorities. b. Tell me about this abuse in your relationship. c. So you were abused? d. Do you know what caused this abuse?

ANS: B If a woman answers yes to any of the Abuse Assessment Screen questions, then the nurse should ask questions designed to assess how recent and how serious the abuse was. Asking the woman an open-ended question, such as tell me about this abuse in your relationship is a good way to start.

The assessment of a 60-year-old patient has taken longer than anticipated. In testing his pain perception, the nurse decides to complete the test as quickly as possible. When the nurse applies the sharp point of the pin on his arm several times, he is only able to identify these as one very sharp prick. What would be the most accurate explanation for this? a. The patient has hyperesthesia as a result of the aging process. b. This response is most likely the result of the summation effect. c. The nurse was probably not poking hard enough with the pin in the other areas. d. The patient most likely has analgesia in some areas of arm and hyperalgesia in others.

ANS: B At least 2 seconds should be allowed to elapse between each stimulus to avoid summation. With summation, frequent consecutive stimuli are perceived as one strong stimulus. The other responses are incorrect.

A patient repeats, I feel hot. Hot, cot, rot, tot, got. Im a spot. The nurse documents this as an illustration of: a. Blocking b. Clanging c. Echolalia d. Neologism

ANS: B Clanging is word choice based on sound, not meaning, and includes nonsense rhymes and puns

A 40-year-old man has come into the clinic with complaints of extreme pain in his toes. The nurse notices that his toes are slightly swollen, reddened, and warm to the touch. His complaints would suggest: a. Osteoporosis. b. Acute gout. c. Ankylosing spondylitis. d. Degenerative joint disease.

ANS: B Clinical findings for acute gout consist of redness, swelling, heat, and extreme pain like a continuous throbbing. Gout is a metabolic disorder of disturbed purine metabolism, associated with elevated serum uric acid. (See Table 22-1 for descriptions of the other terms.)

The ability that humans have to perform very skilled movements such as writing is controlled by the: a. Basal ganglia. b. Corticospinal tract. c. Spinothalamic tract. d. Extrapyramidal tract.

ANS: B Corticospinal fibers mediate voluntary movement, particularly very skilled, discrete, and purposeful movements, such as writing. The corticospinal tract, also known as the pyramidal tract, is a newer, higher motor system that humans have that permits very skilled and purposeful movements. The other responses are not related to skilled movements

The nurse is reviewing the development of culture. Which statement is correct regarding the development of ones culture? Culture is: a. Genetically determined on the basis of racial background. b. Learned through language acquisition and socialization. c. A nonspecific phenomenon and is adaptive but unnecessary. d. Biologically determined on the basis of physical characteristics.

ANS: B Culture is learned from birth through language acquisition and socialization. It is not biologically or genetically determined and is learned by the individual.

When reviewing the use of alcohol by older adults, the nurse notes that older adults have several characteristics that can increase the risk of alcohol use. Which would increase the bioavailability of alcohol in the blood for longer periods in the older adult? a. Increased muscle mass b. Decreased liver and kidney functioning c. Decreased blood pressure d. Increased cardiac output

ANS: B Decreased liver and kidney functioning increases the bioavailability of alcohol in the blood for longer periods. Aging people experience decreased muscle mass (not increased), which also increases the alcohol concentration in the blood because the alcohol is distributed to less tissue over time. Blood pressure and cardiac output are not factors regarding bioavailability.

The nurse is assessing the neurologic status of a patient who has a late-stage brain tumor. With the reflex hammer, the nurse draws a light stroke up the lateral side of the sole of the foot and inward, across the ball of the foot. In response, the patients toes fan out, and the big toe shows dorsiflexion. The nurse interprets this result as: a. Negative Babinski sign, which is normal for adults. b. Positive Babinski sign, which is abnormal for adults. c. Clonus, which is a hyperactive response. d. Achilles reflex, which is an expected response.

ANS: B Dorsiflexion of the big toe and fanning of all toes is a positive Babinski sign, also called up-going toes. This response occurs with upper motor neuron disease of the corticospinal (or pyramidal) tract and is an abnormal finding for adults.

During an examination, the nurse notices severe nystagmus in both eyes of a patient. Which conclusion by the nurse is correct? Severe nystagmus in both eyes: a. Is a normal occurrence. b. May indicate disease of the cerebellum or brainstem. c. Is a sign that the patient is nervous about the examination. d. Indicates a visual problem, and a referral to an ophthalmologist is indicated.

ANS: B End-point nystagmus at an extreme lateral gaze normally occurs; however, the nurse should carefully assess any other nystagmuses. Severe nystagmus occurs with disease of the vestibular system, cerebellum, or brainstem.

A 55-year-old man is experiencing severe pain of sudden onset in the scrotal area. It is somewhat relieved by elevation. On examination the nurse notices an enlarged, red scrotum that is very tender to palpation. Distinguishing the epididymis from the testis is difficult, and the scrotal skin is thick and edematous. This description is consistent with which of these? a. Varicocele b. Epididymitis c. Spermatocele d. Testicular torsion

ANS: B Epididymitis presents as severe pain of sudden onset in the scrotum that is somewhat relieved by elevation. On examination, the scrotum is enlarged, reddened, and exquisitely tender. The epididymis is enlarged and indurated and may be hard to distinguish from the testis. The overlying scrotal skin may be thick and edematous. (See Table 24-6 for more information and for the descriptions of the other terms.)

A woman who has had rheumatoid arthritis for years is starting to notice that her fingers are drifting to the side. The nurse knows that this condition is commonly referred to as: a. Radial drift. b. Ulnar deviation. c. Swan-neck deformity. d. Dupuytren contracture.

ANS: B Fingers drift to the ulnar side because of stretching of the articular capsule and muscle imbalance caused by chronic rheumatoid arthritis. A radial drift is not observed. (See Table 22-4 for descriptions of swan-neck deformity and Dupuytren contracture.)

A patients annual physical examination reveals a lateral curvature of the thoracic and lumbar segments of his spine; however, this curvature disappears with forward bending. The nurse knows that this abnormality of the spine is called: a. Structural scoliosis. b. Functional scoliosis. c. Herniated nucleus pulposus. d. Dislocated hip.

ANS: B Functional scoliosis is flexible and apparent with standing but disappears with forward bending. Structural scoliosis is fixed; the curvature shows both when standing and when bending forward. (See Table 22-7 for description of herniated nucleus pulposus.) These findings are not indicative of a dislocated hip.

The nurse has completed an assessment on a patient who came to the clinic for a leg injury. As a result of the assessment, the nurse has determined that the patient has at-risk alcohol use. Which action by the nurse is most appropriate at this time? a. Record the results of the assessment, and notify the physician on call. b. State, You are drinking more than is medically safe. I strongly recommend that you quit drinking, and Im willing to help you. c. State, It appears that you may have a drinking problem. Here is the telephone number of our local Alcoholics Anonymous chapter. d. Give the patient information about a local rehabilitation clinic.

ANS: B If an assessment has determined that the patient has at-risk drinking behavior, then the nurse should give a short but clear statement of assistance and concern. Simply giving out a telephone number or referral to agencies may not be enough.

During an assessment of a 22-year-old woman who sustained a head injury from an automobile accident 4 hours earlier, the nurse notices the following changes: pupils were equal, but now the right pupil is fully dilated and nonreactive, and the left pupil is 4 mm and reacts to light. What do these findings suggest? a. Injury to the right eye b. Increased intracranial pressure c. Test inaccurately performed d. Normal response after a head injury

ANS: B In a person with a brain injury, a sudden, unilateral, dilated, and nonreactive pupil is ominous. CN III runs parallel to the brainstem. When increasing intracranial pressure pushes down the brainstem (uncal herniation), it puts pressure on CN III, causing pupil dilation. The other responses are incorrect.

The nurse is reviewing concepts of cultural aspects of pain. Which statement is true regarding pain? a. All patients will behave the same way when in pain. b. Just as patients vary in their perceptions of pain, so will they vary in their expressions of pain. c. Cultural norms have very little to do with pain tolerance, because pain tolerance is always biologically determined. d. A patients expression of pain is largely dependent on the amount of tissue injury associated with the pain.

ANS: B In addition to expecting variations in pain perception and tolerance, the nurse should expect variations in the expression of pain. It is well known that individuals turn to their social environment for validation and comparison. The other statements are incorrect.

The nurse is conducting a class on alcohol and the effects of alcohol on the body. How many standard drinks (each containing 14 grams of alcohol) per day in men are associated with increased deaths from cirrhosis, cancers of the mouth, esophagus, and injuries? a. 2 b. 4 c. 6 d. 8

ANS: B In men, alcohol consumption of at least four standard drinks per day is associated with increased deaths from liver cirrhosis, cancers of the mouth, esophagus and other areas, and deaths from injuries and other external causes.

The nurse knows that testing kinesthesia is a test of a persons: a. Fine touch. b. Position sense. c. Motor coordination. d. Perception of vibration.

ANS: B Kinesthesia, or position sense, is the persons ability to perceive passive movements of the extremities. The other options are incorrect.

An 80-year-old woman is visiting the clinic for a checkup. She states, I cant walk as much as I used to. The nurse is observing for motor dysfunction in her hip and should ask her to: a. Internally rotate her hip while she is sitting. b. Abduct her hip while she is lying on her back. c. Adduct her hip while she is lying on her back. d. Externally rotate her hip while she is standing.

ANS: B Limited abduction of the hip while supine is the most common motion dysfunction found in hip disease. The other options are not correct.

When performing a scrotal assessment, the nurse notices that the scrotal contents show a red glow with transillumination. On the basis of this finding the nurse would: a. Assess the patient for the presence of a hernia. b. Suspect the presence of serous fluid in the scrotum. c. Consider this finding normal, and proceed with the examination. d. Refer the patient for evaluation of a mass in the scrotum.

ANS: B Normal scrotal contents do not allow light to pass through the scrotum. However, serous fluid does transilluminate and shows as a red glow. Neither a mass nor a hernia would transilluminate.

An 85-year-old patient comments during his annual physical examination that he seems to be getting shorter as he ages. The nurse should explain that decreased height occurs with aging because: a. Long bones tend to shorten with age. b. The vertebral column shortens. c. A significant loss of subcutaneous fat occurs. d. A thickening of the intervertebral disks develops.

ANS: B Postural changes are evident with aging; decreased height is most noticeable and is due to shortening of the vertebral column. Long bones do not shorten with age. Intervertebral disks actually get thinner with age. Subcutaneous fat is not lost but is redistributed to the abdomen and hips.

The mother of a 10-year-old boy asks the nurse to discuss the recognition of puberty. The nurse should reply by saying: a. Puberty usually begins around 15 years of age. b. The first sign of puberty is an enlargement of the testes. c. The penis size does not increase until about 16 years of age. d. The development of pubic hair precedes testicular or penis enlargement.

ANS: B Puberty begins sometime between age 9 for African Americans and age 10 for Caucasians and Hispanics. The first sign is an enlargement of the testes. Pubic hair appears next, and then penis size increases.

The nurse is reviewing concepts related to ones heritage and beliefs. The belief in divine or superhuman power(s) to be obeyed and worshipped as the creator(s) and ruler(s) of the universe is known as: a. Culture. b. Religion. c. Ethnicity. d. Spirituality

ANS: B Religion is defined as an organized system of beliefs concerning the cause, nature, and purpose of the universe, especially belief in or the worship of God or gods. Spirituality is born out of each persons unique life experiences and his or her personal efforts to find purpose and meaning in life. Ethnicity pertains to a social group within the social system that claims to possess variable traits, such as a common geographic origin, religion, race, and others.

During an interview, the nurse notes that the patient gets up several times to wash her hands even though they are not dirty. This behavior is an example of: a. Social phobia b. Compulsive disorder c. Generalized anxiety disorder d. Posttraumatic stress disorder

ANS: B Repetitive behaviors, such as handwashing, are behaviors that the person feels driven to perform in response to an obsession. The behaviors are aimed at preventing or reducing distress or preventing some dreaded event or situation.

A patient is able to flex his right arm forward without difficulty or pain but is unable to abduct his arm because of pain and muscle spasms. The nurse should suspect: a. Crepitation. b. Rotator cuff lesions. c. Dislocated shoulder. d. Rheumatoid arthritis.

ANS: B Rotator cuff lesions may limit range of motion and cause pain and muscle spasms during abduction, whereas forward flexion remains fairly normal. The other options are not correct.

When assessing the scrotum of a male patient, the nurse notices the presence of multiple firm, nontender, yellow 1-cm nodules. The nurse knows that these nodules are most likely: a. From urethritis. b. Sebaceous cysts. c. Subcutaneous plaques. d. From an inflammation of the epididymis.

ANS: B Sebaceous cysts are commonly found on the scrotum. These yellowish 1-cm nodules are firm, nontender, and often multiple. The other options are not correct.

In obtaining a health history on a 74-year-old patient, the nurse notes that he drinks alcohol daily and that he has noticed a tremor in his hands that affects his ability to hold things. With this information, what response should the nurse make? a. Does your family know you are drinking every day? b. Does the tremor change when you drink alcohol? c. Well do some tests to see what is causing the tremor. d. You really shouldnt drink so much alcohol; it may be causing your tremor.

ANS: B Senile tremor is relieved by alcohol, although not a recommended treatment. The nurse should assess whether the person is abusing alcohol in an effort to relieve the tremor.

Symptoms, such as pain, are often influenced by a persons cultural heritage. Which of the following is a true statement regarding pain? a. Nurses attitudes toward their patients pain are unrelated to their own experiences with pain. b. Nurses need to recognize that many cultures practice silent suffering as a response to pain. c. A nurses area of clinical practice will most likely determine his or her assessment of a patients pain. d. A nurses years of clinical experience and current position are strong indicators of his or her response to patient pain.

ANS: B Silent suffering is a potential response to pain in many cultures. The nurses assessment of pain needs to be embedded in a cultural context. The other responses are not correct.

The nurse recognizes that an example of a person who is heritage consistent would be a: a. Woman who has adapted her clothing to the clothing style of her new country. b. Woman who follows the traditions that her mother followed regarding meals. c. Man who is not sure of his ancestors country of origin. d. Child who is not able to speak his parents native language.

ANS: B Someone who is heritage consistent lives a lifestyle that reflects his or her traditional heritage, not the norms and customs of the new country.

The nurse places a key in the hand of a patient and he identifies it as a penny. What term would the nurse use to describe this finding? a. Extinction b. Astereognosis c. Graphesthesia d. Tactile discrimination

ANS: B Stereognosis is the persons ability to recognize objects by feeling their forms, sizes, and weights. Astereognosis is an inability to identify objects correctly, and it occurs in sensory cortex lesions. Tactile discrimination tests fine touch. Extinction tests the persons ability to feel sensations on both sides of the body at the same point

An individual who takes the magicoreligious perspective of illness and disease is likely to believe that his or her illness was caused by: a. Germs and viruses. b. Supernatural forces. c. Eating imbalanced foods. d. An imbalance within his or her spiritual nature.

ANS: B The basic premise of the magicoreligious perspective is that the world is seen as an arena in which supernatural forces dominate. The fate of the world and those in it depends on the actions of supernatural forces for good or evil. The other answers do not reflect the magicoreligious perspective.

During an interview the patient states, I can feel this bump on the top of both of my shouldersit doesnt hurt but I am curious about what it might be. The nurse should tell the patient that it is his: a. Subacromial bursa. b. Acromion process. c. Glenohumeral joint. d. Greater tubercle of the humerus

ANS: B The bump of the scapulas acromion process is felt at the very top of the shoulder. The other options are not correct.

The nurse is reviewing aspects of substance abuse in preparation for a seminar. Which of these statements illustrates the concept of tolerance to an illicit substance? The person: a. Has a physiologic dependence on a substance. b. Requires an increased amount of the substance to produce the same effect. c. Requires daily use of the substance to function and is unable to stop using it. d. Experiences a syndrome of physiologic symptoms if the substance is not used.

ANS: B The concept of tolerance to a substance indicates that the person requires an increased amount of the substance to produce the same effect. Abuse indicates that the person needs to use the substance daily to function, and the person is unable to stop using it. Dependence is an actual physiologic dependence on the substance. Withdrawal occurs when cessation of the substance produces a syndrome of physiologic symptoms.

The external male genital structures include the: a. Testis. b. Scrotum. c. Epididymis. d. Vas deferens.

ANS: B The external male genital structures include the penis and scrotum. The testis, epididymis, and vas deferens are internal structures.

During an assessment of the CNs, the nurse finds the following: asymmetry when the patient smiles or frowns, uneven lifting of the eyebrows, sagging of the lower eyelids, and escape of air when the nurse presses against the right puffed cheek. This would indicate dysfunction of which of these CNs? a. Motor component of CN IV b. Motor component of CN VII c. Motor and sensory components of CN XI d. Motor component of CN X and sensory component of CN VII

ANS: B The findings listed reflect a dysfunction of the motor component of the facial nerve (CN VII).

The nurse is preparing to conduct a mental status examination. Which statement is true regarding the mental status examination? a. A patients family is the best resource for information about the patients coping skills. b. Gathering mental status information during the health history interview is usually sufficient. c. Integrating the mental status examination into the health history interview takes an enormous amount of extra time. d. To get a good idea of the patients level of functioning, performing a complete mental status examination is usually necessary.

ANS: B The full mental status examination is a systematic check of emotional and cognitive functioning. The steps described, however, rarely need to be taken in their entirety. Usually, one can assess mental status through the context of the health history interview.

The nurse is examining the hip area of a patient and palpates a flat depression on the upper, lateral side of the thigh when the patient is standing. The nurse interprets this finding as the: a. Ischial tuberosity. b. Greater trochanter. c. Iliac crest. d. Gluteus maximus muscle.

ANS: B The greater trochanter of the femur is palpated when the person is standing, and it appears as a flat depression on the upper lateral side of the thigh. The iliac crest is the upper part of the hip bone; the ischial tuberosity lies under the gluteus maximus muscle and is palpable when the hip is flexed; and the gluteus muscle is part of the buttocks.

Which statement concerning the areas of the brain is true? a. The cerebellum is the center for speech and emotions. b. The hypothalamus controls body temperature and regulates sleep. c. The basal ganglia are responsible for controlling voluntary movements. d. Motor pathways of the spinal cord and brainstem synapse in the thalamus.

ANS: B The hypothalamus is a vital area with many important functions: body temperature controller, sleep center, anterior and posterior pituitary gland regulator, and coordinator of autonomic nervous system activity and emotional status. The cerebellum controls motor coordination, equilibrium, and balance. The basal ganglia control autonomic movements of the body. The motor pathways of the spinal cord synapse in various areas of the spinal cord, not in the thalamus.

A mother of a 1-month-old infant asks the nurse why it takes so long for infants to learn to roll over. The nurse knows that the reason for this is: a. A demyelinating process must be occurring with her infant. b. Myelin is needed to conduct the impulses, and the neurons of a newborn are not yet myelinated. c. The cerebral cortex is not fully developed; therefore, control over motor function gradually occurs. d. The spinal cord is controlling the movement because the cerebellum is not yet fully developed.

ANS: B The infants sensory and motor development proceeds along with the gradual acquisition of myelin, which is needed to conduct most impulses. Very little cortical control exists, and the neurons are not yet myelinated. The other responses are not correct.

The nurse is assessing a patients ischial tuberosity. To palpate the ischial tuberosity, the nurse knows that it is best to have the patient: a. Standing. b. Flexing the hip. c. Flexing the knee. d. Lying in the supine position.

ANS: B The ischial tuberosity lies under the gluteus maximus muscle and is palpable when the hip is flexed. The other options are not correct

The two parts of the nervous system are the: a. Motor and sensory. b. Central and peripheral. c. Peripheral and autonomic. d. Hypothalamus and cerebral.

ANS: B The nervous system can be divided into two partscentral and peripheral. The central nervous system includes the brain and spinal cord. The peripheral nervous system includes the 12 pairs of cranial nerves (CNs), the 31 pairs of spinal nerves, and all of their branches.

In the majority culture of America, coughing, sweating, and diarrhea are symptoms of an illness. For some individuals of Mexican-American origin, however, these symptoms are a normal part of living. The nurse recognizes that this difference is true, probably because Mexican-Americans: a. Have less efficient immune systems and are often ill. b. Consider these symptoms part of normal living, not symptoms of ill health. c. Come from Mexico, and coughing is normal and healthy there. d. Are usually in a lower socioeconomic group and are more likely to be sick.

ANS: B The nurse needs to identify the meaning of health to the patient, remembering that concepts are derived, in part, from the way in which members of the cultural group define health.

During the neurologic assessment of a healthy 35-year-old patient, the nurse asks him to relax his muscles completely. The nurse then moves each extremity through full range of motion. Which of these results would the nurse expect to find? a. Firm, rigid resistance to movement b. Mild, even resistance to movement c. Hypotonic muscles as a result of total relaxation d. Slight pain with some directions of movement

ANS: B Tone is the normal degree of tension (contraction) in voluntarily relaxed muscles. It shows a mild resistance to passive stretching. Normally, the nurse will notice a mild, even resistance to movement. The other responses are not correct.

During an examination of an aging man, the nurse recognizes that normal changes to expect would be: a. Change in scrotal color. b. Decrease in the size of the penis. c. Enlargement of the testes and scrotum. d. Increase in the number of rugae over the scrotal sac.

ANS: B When assessing the genitals of an older man, the nurse may notice thinner, graying pubic hair and a decrease in the size of the penis. The size of the testes may be decreased, they may feel less firm, and the scrotal sac is pendulous with less rugae. No change in scrotal color is observed.

The nurse suspects abuse when a 10-year-old child is taken to the urgent care center for a leg injury. The best way to document the history and physical findings is to: a. Document what the childs caregiver tells the nurse. b. Use the words the child has said to describe how the injury occurred. c. Record what the nurse observes during the conversation. d. Rely on photographs of the injuries.

ANS: B When documenting the history and physical findings of suspected child abuse and neglect, use the words the child has said to describe how his or her injury occurred. Remember, the abuser may be accompanying the child.

A patient is brought to the emergency department. He is restless, has dilated pupils, is sweating, has a runny nose and tearing eyes, and complains of muscle and joint pains. His girlfriend thinks he has influenza, but she became concerned when his temperature went up to 39.4 C. She admits that he has been a heavy drug user, but he has been trying to stop on his own. The nurse suspects that the patient is experiencing withdrawal symptoms from which substance? a. Alcohol b. Heroin c. Crack cocaine d. Sedatives

ANS: B Withdrawal symptoms of opiates, such as heroin, are similar to the clinical picture of influenza and include symptoms such as dilated pupils, lacrimation, runny nose, tachycardia, fever, restlessness, muscle and joint pains, and other symptoms.

The nurse is assessing the joints of a woman who has stated, I have a long family history of arthritis, and my joints hurt. The nurse suspects that she has osteoarthritis. Which of these are symptoms of osteoarthritis? Select all that apply. a. Symmetric joint involvement b. Asymmetric joint involvement c. Pain with motion of affected joints d. Affected joints are swollen with hard, bony protuberances e. Affected joints may have heat, redness, and swelling

ANS: B, C, D In osteoarthritis, asymmetric joint involvement commonly affects hands, knees, hips, and lumbar and cervical segments of the spine. Affected joints have stiffness, swelling with hard bony protuberances, pain with motion, and limitation of motion. The other options reflect the signs of rheumatoid arthritis.

A patient with a known history of heavy alcohol use has been admitted to the ICU after he was found unconscious outside a bar. The nurse closely monitors him for symptoms of withdrawal. Which of these symptoms may occur during this time? Select all that apply. a. Bradycardia b. Coarse tremor of the hands c. Transient hallucinations d. Somnolence e. Sweating

ANS: B, C, E Symptoms of uncomplicated alcohol withdrawal start shortly after the cessation of drinking, peak at the second day, and improve by the fourth or fifth day. Symptoms include coarse tremors of the hands, tongue, and eyelids; anorexia; nausea and vomiting; autonomic hyperactivity (e.g., tachycardia, sweating, elevated blood pressure); and transient hallucinations, among other symptoms

A 69-year-old patient has been admitted to an adult psychiatric unit because his wife thinks he is getting more and more confused. He laughs when he is found to be forgetful, saying Im just getting old! After the nurse completes a thorough neurologic assessment, which findings would be indicative of Alzheimer disease? Select all that apply. a. Occasionally forgetting names or appointments b. Difficulty performing familiar tasks, such as placing a telephone call c. Misplacing items, such as putting dish soap in the refrigerator d. Sometimes having trouble finding the right word e. Rapid mood swings, from calm to tears, for no apparent reason f. Getting lost in ones own neighborhood

ANS: B, C, E, F Difficulty performing familiar tasks, misplacing items, rapid mood swings, and getting lost in ones own neighborhood can be warning signs of Alzheimer disease. Occasionally forgetting names or appointments, and sometimes having trouble finding the right word are part of normal aging. (For other examples of Alzheimer disease, see Table 23-2.)

The nurse is reviewing aspects of cultural care. Which statements illustrate proper cultural care? Select all that apply. a. Examine the patient within the context of ones own cultural health and illness practices. b. Select questions that are not complex. c. Ask questions rapidly. d. Touch patients within the cultural boundaries of their heritage. e. Pace questions throughout the physical examination

ANS: B, D, E Patients should be examined within the context of their own cultural health and illness practices. Questions should be simply stated and not rapidly asked.

A patient has a severed spinal nerve as a result of trauma. Which statement is true in this situation? a. Because there are 31 pairs of spinal nerves, no effect results if only one nerve is severed. b. The dermatome served by this nerve will no longer experience any sensation. c. The adjacent spinal nerves will continue to carry sensations for the dermatome served by the severed nerve. d. A severed spinal nerve will only affect motor function of the patient because spinal nerves have no sensory component.

ANS: C A dermatome is a circumscribed skin area that is primarily supplied from one spinal cord segment through a particular spinal nerve. The dermatomes overlap, which is a form of biologic insurance; that is, if one nerve is severed, then most of the sensations can be transmitted by the spinal nerve above and the spinal nerve below the severed nerve.

A teenage girl has arrived complaining of pain in her left wrist. She was playing basketball when she fell and landed on her left hand. The nurse examines her hand and would expect a fracture if the girl complains of a: a. Dull ache. b. Deep pain in her wrist. c. Sharp pain that increases with movement. d. Dull throbbing pain that increases with rest.

ANS: C A fracture causes sharp pain that increases with movement. The other types of pain do not occur with a fracture.

The nurse is planning health teaching for a 65-year-old woman who has had a cerebrovascular accident (stroke) and has aphasia. Which of these questions is most important to use when assessing mental status in this patient? a. Please count backward from 100 by seven. b. I will name three items and ask you to repeat them in a few minutes. c. Please point to articles in the room and parts of the body as I name them. d. What would you do if you found a stamped, addressed envelope on the sidewalk?

ANS: C Additional tests for persons with aphasia include word comprehension (asking the individual to point to articles in the room or parts of the body), reading (asking the person to read available print), and writing (asking the person to make up and write a sentence).

The nurse is assessing a 1-week-old infant and is testing his muscle strength. The nurse lifts the infant with hands under the axillae and notices that the infant starts to slip between the hands. The nurse should: a. Suspect a fractured clavicle. b. Suspect that the infant may have a deformity of the spine. c. Suspect that the infant may have weakness of the shoulder muscles. d. Conclude that this is a normal finding because the musculature of an infant at this age is undeveloped.

ANS: C An infant who starts to slip between the nurses hands shows weakness of the shoulder muscles. An infant with normal muscle strength wedges securely between the nurses hands. The other responses are not correct

The nurse is planning to assess new memory with a patient. The best way for the nurse to do this would be to: a. Administer the FACT test. b. Ask him to describe his first job. c. Give him the Four Unrelated Words Test. d. Ask him to describe what television show he was watching before coming to the clinic.

ANS: C Ask questions that can be corroborated, which screens for the occasional person who confabulates or makes up answers to fill in the gaps of memory loss. The Four Unrelated Words Test tests the persons ability to lay down new memories and is a highly sensitive and valid memory test.

When taking the health history on a patient with a seizure disorder, the nurse assesses whether the patient has an aura. Which of these would be the best question for obtaining this information? a. Does your muscle tone seem tense or limp? b. After the seizure, do you spend a lot of time sleeping? c. Do you have any warning sign before your seizure starts? d. Do you experience any color change or incontinence during the seizure?

ANS: C Aura is a subjective sensation that precedes a seizure; it could be auditory, visual, or motor. The other questions do not solicit information about an aura.

A patient who has had rheumatoid arthritis for years comes to the clinic to ask about changes in her fingers. The nurse will assess for signs of what problems? a. Heberden nodes b. Bouchard nodules c. Swan-neck deformities d. Dupuytren contractures

ANS: C Changes in the fingers caused by chronic rheumatoid arthritis include swan-neck and boutonniere deformities. Heberden nodes and Bouchard nodules are associated with osteoarthritis. Dupuytren contractures of the digits occur because of chronic hyperplasia of the palmar fascia

A patient repeatedly seems to have difficulty coming up with a word. He says, I was on my way to work, and when I got there, the thing that you step into that goes up in the air was so full that I decided to take the stairs. The nurse will note on his chart that he is using or experiencing: a. Blocking b. Neologism c. Circumlocution d. Circumstantiality

ANS: C Circumlocution is a roundabout expression, substituting a phrase when one cannot think of the name of the object.

During a class on cultural practices, the nurse hears the term cultural taboo. Which statement illustrates the concept of a cultural taboo? a. Believing that illness is a punishment of sin b. Trying prayer before seeking medical help c. Refusing to accept blood products as part of treatment d. Stating that a childs birth defect is the result of the parents sins

ANS: C Cultural taboos are practices that are to be avoided, such as receiving blood products, eating pork, and consuming caffeine. The other answers do not reflect cultural taboos.

A female patient has denied any abuse when answering the Abuse Assessment Screen, but the nurse has noticed some other conditions that are associated with IPV. Examples of such conditions include: a. Asthma. b. Confusion. c. Depression. d. Frequent colds.

ANS: C Depression is one of the conditions that is particularly associated with IPV. Abused women also have been found to have more chronic health problems, such as neurologic, gastrointestinal, and gynecologic symptoms; chronic pain; and symptoms of suicidality and posttraumatic stress disorder.

The nurse is performing a genitourinary assessment on a 50-year-old obese male laborer. On examination, the nurse notices a painless round swelling close to the pubis in the area of the internal inguinal ring that is easily reduced when the individual is supine. These findings are most consistent with a(n) ______ hernia. a. Scrotal b. Femoral c. Direct inguinal d. Indirect inguinal

ANS: C Direct inguinal hernias occur most often in men over the age of 40 years. It is an acquired weakness brought on by heavy lifting, obesity, chronic cough, or ascites. The direct inguinal hernia is usually a painless, round swelling close to the pubis in the area of the internal inguinal ring that is easily reduced when the individual is supine. (See Table 24-6 for a description of scrotal hernia. See Table 24-7 for the descriptions of femoral hernias and indirect inguinal hernias.)

A man who was found wandering in a park at 2 AM has been brought to the emergency department for an examination; he said he fell and hit his head. During the examination, the nurse asks him to use his index finger to touch the nurses finger, then his own nose, then the nurses finger again (which has been moved to a different location). The patient is clumsy, unable to follow the instructions, and overshoots the mark, missing the finger. The nurse should suspect which of the following? a. Cerebral injury b. Cerebrovascular accident c. Acute alcohol intoxication d. Peripheral neuropathy

ANS: C During the finger-to-finger test, if the person has clumsy movement with overshooting the mark, either a cerebellar disorder or acute alcohol intoxication should be suspected. The persons movements should be smooth and accurate. The other options are not correct.

After a class on culture and ethnicity, the new graduate nurse reflects a correct understanding of the concept of ethnicity with which statement? a. Ethnicity is dynamic and ever changing. b. Ethnicity is the belief in a higher power. c. Ethnicity pertains to a social group within the social system that claims shared values and traditions. d. Ethnicity is learned from birth through the processes of language acquisition and socialization.

ANS: C Ethnicity pertains to a social group within the social system that claims to have variable traits, such as a common geographic origin, migratory status, religion, race, language, values, traditions, symbols, or food preferences.Culture is dynamic, ever changing, and learned from birth through the processes of language acquisition and socialization. Religion is the belief in a higher power

During an examination, the nurse notes that a patient is exhibiting flight of ideas. Which statement by the patient is an example of flight of ideas? a. My stomach hurts. Hurts, spurts, burts. b. Kiss, wood, reading, ducks, onto, maybe. c. Take this pill? The pill is red. I see red. Red velvet is soft, soft as a babys bottom. d. I wash my hands, wash them, wash them. I usually go to the sink and wash my hands.

ANS: C Flight of ideas is demonstrated by an abrupt change, rapid skipping from topic to topic, and practically continuous flow of accelerated speech. Topics usually have recognizable associations or are plays on words.

A mother brings her newborn baby boy in for a checkup; she tells the nurse that he does not seem to be moving his right arm as much as his left and that he seems to have pain when she lifts him up under the arms. The nurse suspects a fractured clavicle and would observe for: a. Negative Allis test. b. Positive Ortolani sign. c. Limited range of motion during the Moro reflex. d. Limited range of motion during Lasgue test.

ANS: C For a fractured clavicle, the nurse should observe for limited arm range of motion and unilateral response to the Moro reflex. The other tests are not appropriate for this type of fracture.

The nurse suspects that a patient has carpal tunnel syndrome and wants to perform the Phalen test. To perform this test, the nurse should instruct the patient to: a. Dorsiflex the foot. b. Plantarflex the foot. c. Hold both hands back to back while flexing the wrists 90 degrees for 60 seconds. d. Hyperextend the wrists with the palmar surface of both hands touching, and wait for 60 seconds.

ANS: C For the Phalen test, the nurse should ask the person to hold both hands back to back while flexing the wrists 90 degrees. Acute flexion of the wrist for 60 seconds produces no symptoms in the normal hand. The Phalen test reproduces numbness and burning in a person with carpal tunnel syndrome. The other actions are not correct when testing for carpal tunnel syndrome.

During a genital examination, the nurse notices that a male patient has clusters of small vesicles on the glans, surrounded by erythema. The nurse recognizes that these lesions are: a. Peyronie disease. b. Genital warts. c. Genital herpes. d. Syphilitic cancer.

ANS: C Genital herpes, or herpes simplex virus 2 (HSV-2), infections are indicated with clusters of small vesicles with surrounding erythema, which are often painful and erupt on the glans or foreskin. (See Table 24-4 for the descriptions of the other options.)

An accessory glandular structure for the male genital organs is the: a. Testis. b. Scrotum. c. Prostate. d. Vas deferens.

ANS: C Glandular structures accessory to the male genital organs are the prostate, seminal vesicles, and bulbourethral glands.

he nurse is asking an adolescent about illicit substance abuse. The adolescent answers, Yes, Ive used marijuana at parties with my friends. What is the next question the nurse should ask? a. Who are these friends? b. Do your parents know about this? c. When was the last time you used marijuana? d. Is this a regular habit?

ANS: C If a patient admits to the use of illicit substances, then the nurse should ask, When was the last time you used drugs? and How much did you take that time? The other questions may be considered accusatory and are not conducive to gathering information.

A 2-month-old uncircumcised infant has been brought to the clinic for a well-baby checkup. How would the nurse proceed with the genital examination? a. Eliciting the cremasteric reflex is recommended. b. The glans is assessed for redness or lesions. c. Retracting the foreskin should be avoided until the infant is 3 months old. d. Any dirt or smegma that has collected under the foreskin should be noted.

ANS: C If uncircumcised, then the foreskin is normally tight during the first 3 months and should not be retracted because of the risk of tearing the membrane attaching the foreskin to the shaft. The other options are not correct.

An older Mexican-American woman with traditional beliefs has been admitted to an inpatient care unit. A culturally sensitive nurse would: a. Contact the hospital administrator about the best course of action. b. Automatically get a curandero for her, because requesting one herself is not culturally appropriate. c. Further assess the patients cultural beliefs and offer the patient assistance in contacting a curandero or priest if she desires. d. Ask the family what they would like to do because Mexican-Americans traditionally give control of decision making to their families.

ANS: C In addition to seeking help from the biomedical or scientific health care provider, patients may also seek help from folk or religious healers. Some people, such as those of Mexican-American or American-Indian origins, may believe that the cure is incomplete unless the body, mind, and spirit are also healed (although the division of the person into parts is a Western concept).

The nurse is conducting a patient interview. Which statement made by the patient should the nurse more fully explore during the interview? a. I sleep like a baby. b. I have no health problems. c. I never did too good in school. d. I am not currently taking any medications.

ANS: C In every mental status examination, the following factors from the health history that could affect the findings should be noted: any known illnesses or health problems, such as alcoholism or chronic renal disease; current medications, the side effects of which may cause confusion or depression; the usual educational and behavioral level, noting this level as the patients normal baseline and not expecting a level of performance on the mental status examination to exceed it; and responses to personal history questions, indicating current stress, social interaction patterns, and sleep habits.

During an examination of an aging man, the nurse recognizes that normal changes to expect would be: a. Enlarged scrotal sac. b. Increased pubic hair. c. Decreased penis size. d. Increased rugae over the scrotum.

ANS: C In the aging man, the amount of pubic hair decreases, the penis size decreases, and the rugae over the scrotal sac decreases. The scrotal sac does not enlarge.

During an examination, the nurse asks a patient to bend forward from the waist and notices that the patient has lateral tilting. When his leg is raised straight up, the patient complains of a pain going down his buttock into his leg. The nurse suspects: a. Scoliosis. b. Meniscus tear. c. Herniated nucleus pulposus. d. Spasm of paravertebral muscles.

ANS: C Lateral tilting and sciatic pain with straight leg raising are findings that occur with a herniated nucleus pulposus. The other options are not correct.

The nurse is explaining the mechanism of the growth of long bones to a mother of a toddler. Where does lengthening of the bones occur? a. Bursa b. Calcaneus c. Epiphyses d. Tuberosities

ANS: C Lengthening occurs at the epiphyses, or growth plates. The other options are not correct.

The nurse is performing a neurologic assessment on a 41-year-old woman with a history of diabetes. When testing her ability to feel the vibrations of a tuning fork, the nurse notices that the patient is unable to feel vibrations on the great toe or ankle bilaterally, but she is able to feel vibrations on both patellae. Given this information, what would the nurse suspect? a. Hyperalgesia b. Hyperesthesia c. Peripheral neuropathy d. Lesion of sensory cortex

ANS: C Loss of vibration sense occurs with peripheral neuropathy (e.g., diabetes and alcoholism). Peripheral neuropathy is worse at the feet and gradually improves as the examiner moves up the leg, as opposed to a specific nerve lesion, which has a clear zone of deficit for its dermatome. The other responses are incorrect.

A patient states, I feel so sad all of the time. I cant feel happy even doing things I used to like to do. He also states that he is tired, sleeps poorly, and has no energy. To differentiate between a dysthymic disorder and a major depressive disorder, the nurse should ask which question? a. Have you had any weight changes? b. Are you having any thoughts of suicide? c. How long have you been feeling this way? d. Are you having feelings of worthlessness?

ANS: C Major depressive disorder is characterized by one or more major depressive episodes, that is, at least 2 weeks of depressed mood or loss of interest accompanied by at least four additional symptoms of depression. Dysthymic disorder is characterized by at least 2 years of depressed mood for more days than not, accompanied by additional depressive symptoms.

Many Asians believe in the yin/yang theory, which is rooted in the ancient Chinese philosophy of Tao. Which statement most accurately reflects health in an Asian with this belief? a. A person is able to work and produce. b. A person is happy, stable, and feels good. c. All aspects of the person are in perfect balance. d. A person is able to care for others and function socially

ANS: C Many Asians believe in the yin/yang theory, in which health is believed to exist when all aspects of the person are in perfect balance. The other statements do not describe this theory.

The nurse is aware that intimate partner violence (IPV) screening should occur with which situation? a. When IPV is suspected b. When a woman has an unexplained injury c. As a routine part of each health care encounter d. When a history of abuse in the family is known

ANS: C Many nursing professional organizations have called for routine, universal screening for IPV to assist women in getting help for the problem.

The nurse is aware of which statement to be true regarding the incidence of testicular cancer? a. Testicular cancer is the most common cancer in men aged 30 to 50 years. b. The early symptoms of testicular cancer are pain and induration. c. Men with a history of cryptorchidism are at the greatest risk for the development of testicular cancer. d. The cure rate for testicular cancer is low.

ANS: C Men with undescended testicles (cryptorchidism) are at the greatest risk for the development of testicular cancer. The overall incidence of testicular cancer is rare. Although testicular cancer has no early symptoms, when detected early and treated before metastasizing, the cure rate is almost 100%.

During an examination, the nurse can assess mental status by which activity? a. Examining the patients electroencephalogram b. Observing the patient as he or she performs an intelligence quotient (IQ) test c. Observing the patient and inferring health or dysfunction d. Examining the patients response to a specific set of questions

ANS: C Mental status cannot be directly scrutinized like the characteristics of skin or heart sounds. Its functioning is inferred through an assessment of an individuals behaviors, such as consciousness, language, mood and affect, and other aspects.

The nurse is performing a mental status examination. Which statement is true regarding the assessment of mental status? a. Mental status assessment diagnoses specific psychiatric disorders. b. Mental disorders occur in response to everyday life stressors. c. Mental status functioning is inferred through the assessment of an individuals behaviors. d. Mental status can be directly assessed, similar to other systems of the body (e.g., heart sounds, breath sounds).

ANS: C Mental status functioning is inferred through the assessment of an individuals behaviors. It cannot be directly assessed like the characteristics of the skin or heart sounds.

A 2-year-old boy has been diagnosed with physiologic cryptorchidism. Considering this diagnosis, during assessment the nurse will most likely observe: a. Testes that are hard and painful to palpation. b. Atrophic scrotum and a bilateral absence of the testis. c. Absence of the testis in the scrotum, but the testis can be milked down. d. Testes that migrate into the abdomen when the child squats or sits cross-legged.

ANS: C Migratory testes (physiologic cryptorchidism) are common because of the strength of the cremasteric reflex and the small mass of the prepubertal testes. The affected side has a normally developed scrotum and the testis can be milked down. The other responses are not correct.

A patient is being assessed for range-of-joint movement. The nurse asks him to move his arm in toward the center of his body. This movement is called: a. Flexion. b. Abduction. c. Adduction. d. Extension.

ANS: C Moving a limb toward the midline of the body is called adduction; moving a limb away from the midline of the body is called abduction. Flexion is bending a limb at a joint; and extension is straightening a limb at a joint.

A 14-year-old boy who has been diagnosed with Osgood-Schlatter disease reports painful swelling just below the knee for the past 5 months. Which response by the nurse is appropriate? a. If these symptoms persist, you may need arthroscopic surgery. b. You are experiencing degeneration of your knee, which may not resolve. c. Your disease is due to repeated stress on the patellar tendon. It is usually self-limited, and your symptoms should resolve with rest. d. Increasing your activity and performing knee-strengthening exercises will help decrease the inflammation and maintain mobility in the knee.

ANS: C Osgood-Schlatter disease is a painful swelling of the tibial tubercle just below the knee and most likely due to repeated stress on the patellar tendon. It is usually self-limited, occurring during rapid growth and most often in boys. The symptoms resolve with rest. The other responses are not appropriate.

The nurse is teaching a class on preventing osteoporosis to a group of perimenopausal women. Which of these actions is the best way to prevent or delay bone loss in this group? a. Taking calcium and vitamin D supplements b. Taking medications to prevent osteoporosis c. Performing physical activity, such as fast walking d. Assessing bone density annually

ANS: C Physical activity, such as fast walking, delays or prevents bone loss in perimenopausal women. The faster the pace of walking, the higher the preventive effect is on the risk of hip fracture. The other options are not correct.

During a neonatal examination, the nurse notices that the newborn infant has six toes. This finding is documented as: a. Unidactyly. b. Syndactyly. c. Polydactyly. d. Multidactyly

ANS: C Polydactyly is the presence of extra fingers or toes. Syndactyly is webbing between adjacent fingers or toes. The other terms are not correct.

A patient is complaining of pain in his joints that is worse in the morning, better after he moves around for a while, and then gets worse again if he sits for long periods. The nurse should assess for other signs of what problem? a. Tendinitis b. Osteoarthritis c. Rheumatoid arthritis d. Intermittent claudication

ANS: C Rheumatoid arthritis is worse in the morning when a person arises. Movement increases most joint pain, except the pain with rheumatoid arthritis, which decreases with movement. The other options are not correct.

A patient is unable to perform rapid alternating movements such as rapidly patting her knees. The nurse should document this inability as: a. Ataxia. b. Astereognosis. c. Presence of dysdiadochokinesia. d. Loss of kinesthesia.

ANS: C Slow clumsy movements and the inability to perform rapid alternating movements occur with cerebellar disease. The condition is termed dysdiadochokinesia. Ataxia is an uncoordinated or unsteady gait. Astereognosis is the inability to identify an object by feeling it. Kinesthesia is the persons ability to perceive passive movement of the extremities or the loss of position sense.

During an assessment of an 80-year-old patient, the nurse notices the following: an inability to identify vibrations at her ankle and to identify the position of her big toe, a slower and more deliberate gait, and a slightly impaired tactile sensation. All other neurologic findings are normal. The nurse should interpret that these findings indicate: a. CN dysfunction. b. Lesion in the cerebral cortex. c. Normal changes attributable to aging. d. Demyelination of nerves attributable to a lesion.

ANS: C Some aging adults show a slower response to requests, especially for those calling for coordination of movements. The findings listed are normal in the absence of other significant abnormal findings. The other responses are incorrect.

The nurse is caring for a patient who has just had neurosurgery. To assess for increased intracranial pressure, what would the nurse include in the assessment? a. CNs, motor function, and sensory function b. Deep tendon reflexes, vital signs, and coordinated movements c. Level of consciousness, motor function, pupillary response, and vital signs d. Mental status, deep tendon reflexes, sensory function, and pupillary response

ANS: C Some hospitalized persons have head trauma or a neurologic deficit from a systemic disease process. These people must be closely monitored for any improvement or deterioration in neurologic status and for any indication of increasing intracranial pressure. The nurse should use an abbreviation of the neurologic examination in the following sequence: level of consciousness, motor function, pupillary response, and vital signs.

The nurse is comparing the concepts of religion and spirituality. Which of the following is an appropriate component of ones spirituality? a. Belief in and the worship of God or gods b. Attendance at a specific church or place of worship c. Personal effort made to find purpose and meaning in life d. Being closely tied to ones ethnic background

ANS: C Spirituality refers to each persons unique life experiences and his or her personal effort to find purpose and meaning in life. The other responses apply to religion.

A 45-year-old mother of two children is seen at the clinic for complaints of losing my urine when I sneeze. The nurse documents that she is experiencing: a. Urinary frequency. b. Enuresis. c. Stress incontinence. d. Urge incontinence.

ANS: C Stress incontinence is involuntary urine loss with physical strain, sneezing, or coughing that occurs as a result to weakness of the pelvic floor. Urinary frequency is urinating more times than usual (more than five to six times per day). Enuresis is involuntary passage of urine at night after age 5 to 6 years (bed wetting). Urge incontinence is involuntary urine loss from overactive detrusor muscle in the bladder. It contracts, causing an urgent need to void.

The nurse is examining a 3-year-old child who was brought to the emergency department after a fall. Which bruise, if found, would be of most concern? a. Bruise on the knee b. Bruise on the elbow c. Bruising on the abdomen d. Bruise on the shin

ANS: C Studies have shown that children who are walking often have bruises over the bony prominences of the front of their bodies. Other studies have found that bruising in atypical places such as the buttocks, hands, feet, and abdomen were exceedingly rare and should arouse concern

When the nurse is performing a testicular examination on a 25-year-old man, which finding is considered normal? a. Nontender subcutaneous plaques b. Scrotal area that is dry, scaly, and nodular c. Testes that feel oval and movable and are slightly sensitive to compression d. Single, hard, circumscribed, movable mass, less than 1 cm under the surface of the testes

ANS: C Testes normally feel oval, firm and rubbery, smooth, and bilaterally equal and are freely movable and slightly tender to moderate pressure. The scrotal skin should not be dry, scaly, or nodular or contain subcutaneous plaques. Any mass would be an abnormal finding.

The nurse is performing the Denver II screening test on a 12-month-old infant during a routine well-child visit. The nurse should tell the infants parents that the Denver II: a. Tests three areas of development: cognitive, physical, and psychological b. Will indicate whether the child has a speech disorder so that treatment can begin. c. Is a screening instrument designed to detect children who are slow in development. d. Is a test to determine intellectual ability and may indicate whether problems will develop later in school.

ANS: C The Denver II is a screening instrument designed to detect developmental delays in infants and preschoolers. It tests four functions: gross motor, language, fine motor-adaptive, and personal-social. The Denver II is not an intelligence test; it does not predict current or future intellectual ability. It is not diagnostic; it does not suggest treatment regimens.

A 45-year-old woman is at the clinic for a mental status assessment. In giving her the Four Unrelated Words Test, the nurse would be concerned if she could not ____ four unrelated words ____. a. Invent; within 5 minutes b. Invent; within 30 seconds c. Recall; after a 30-minute delay d. Recall; after a 60-minute delay

ANS: C The Four Unrelated Words Test tests the persons ability to lay down new memories. It is a highly sensitive and valid memory test. It requires more effort than the recall of personal or historic events. To the person say, I am going to say four words. I want you to remember them. In a few minutes I will ask you to recall them. After 5 minutes, ask for the four words. The normal response for persons under 60 years is an accurate three- or four-word recall after a 5-, 10-, and 30-minute delay.

The nurse is providing instructions to newly hired graduates for the minimental state examination (MMSE). Which statement best describes this examination? a. Scores below 30 indicate cognitive impairment. b. The MMSE is a good tool to evaluate mood and thought processes. c. This examination is a good tool to detect delirium and dementia and to differentiate these from psychiatric mental illness. d. The MMSE is useful tool for an initial evaluation of mental status. Additional tools are needed to evaluate cognition changes over time.

ANS: C The MMSE is a quick, easy test of 11 questions and is used for initial and serial evaluations and can demonstrate a worsening or an improvement of cognition over time and with treatment. It evaluates cognitive functioning, not mood or thought processes. MMSE is a good screening tool to detect dementia and delirium and to differentiate these from psychiatric mental illness.

While assessing a 7-month-old infant, the nurse makes a loud noise and notices the following response: abduction and flexion of the arms and legs; fanning of the fingers, and curling of the index finger and thumb in a C position, followed by the infant bringing in the arms and legs to the body. What does the nurse know about this response? a. This response could indicate brachial nerve palsy. b. This reaction is an expected startle response at this age. c. This reflex should have disappeared between 1 and 4 months of age. d. This response is normal as long as the movements are bilaterally symmetric.

ANS: C The Moro reflex is present at birth and usually disappears at 1 to 4 months. Absence of the Moro reflex in the newborn or its persistence after 5 months of age indicates severe central nervous system injury. The other responses are incorrect.

Which term refers to a wound produced by the tearing or splitting of body tissue, usually from blunt impact over a bony surface? a. Abrasion b. Contusion c. Laceration d. Hematoma

ANS: C The term laceration refers to a wound produced by the tearing or splitting of body tissue. An abrasion is caused by the rubbing of the skin or mucous membrane. A contusion is injury to tissues without breakage of skin, and a hematoma is a localized collection of extravasated blood.

The nurse is assessing a 75-year-old man. As the nurse begins the mental status portion of the assessment, the nurse expects that this patient: a. Will have no decrease in any of his abilities, including response time. b. Will have difficulty on tests of remote memory because this ability typically decreases with age. c. May take a little longer to respond, but his general knowledge and abilities should not have declined. d. Will exhibit had a decrease in his response time because of the loss of language and a decrease in general knowledge.

ANS: C The aging process leaves the parameters of mental status mostly intact. General knowledge does not decrease, and little or no loss in vocabulary occurs. Response time is slower than in a youth. It takes a little longer for the brain to process information and to react to it. Recent memory, which requires some processing, is somewhat decreased with aging, but remote memory is not affected.

The articulation of the mandible and the temporal bone is known as the: a. Intervertebral foramen. b. Condyle of the mandible. c. Temporomandibular joint. d. Zygomatic arch of the temporal bone

ANS: C The articulation of the mandible and the temporal bone is the temporomandibular joint. The other responses are not correct.

A 30-year-old woman tells the nurse that she has been very unsteady and has had difficulty in maintaining her balance. Which area of the brain that is related to these findings would concern the nurse? a. Thalamus b. Brainstem c. Cerebellum d. Extrapyramidal tract

ANS: C The cerebellar system coordinates movement, maintains equilibrium, and helps maintain posture. The thalamus is the primary relay station where sensory pathways of the spinal cord, cerebellum, and brainstem form synapses on their way to the cerebral cortex. The brainstem consists of the midbrain, pons, and medulla and has various functions, especially concerning autonomic centers. The extrapyramidal tract maintains muscle tone for gross automatic movements, such as walking

During a seminar on cultural aspects of nursing, the nurse recognizes that the definition stating the specific and distinct knowledge, beliefs, skills, and customs acquired by members of a society reflects which term? a. Mores b. Norms c. Culture d. Social learning

ANS: C The culture that develops in any given society is always specific and distinctive, encompassing all of the knowledge, beliefs, customs, and skills acquired by members of the society. The other terms do not fit the given definition.

A patient is visiting the clinic for an evaluation of a swollen, painful knuckle. The nurse notices that the knuckle above his ring on the left hand is swollen and that he is unable to remove his wedding ring. This joint is called the _________ joint. a. Interphalangeal b. Tarsometatarsal c. Metacarpophalangeal d. Tibiotalar

ANS: C The joint located just above the ring on the finger is the metacarpophalangeal joint. The interphalangeal joint is located distal to the metacarpophalangeal joint. The tarsometatarsal and tibiotalar joints are found in the foot and ankle. (See Figure 22-10 for a diagram of the bones and joints of the hand and fingers.)

When the nurse is testing the triceps reflex, what is the expected response? a. Flexion of the hand b. Pronation of the hand c. Extension of the forearm d. Flexion of the forearm

ANS: C The normal response of the triceps reflex is extension of the forearm. The normal response of the biceps reflex causes flexion of the forearm. The other responses are incorrect.

A patient has been in the intensive care unit for 10 days. He has just been moved to the medical-surgical unit, and the admitting nurse is planning to perform a mental status examination. During the tests of cognitive function, the nurse would expect that he: a. May display some disruption in thought content. b. Will state, I am so relieved to be out of intensive care. c. Will be oriented to place and person, but the patient may not be certain of the date. d. May show evidence of some clouding of his level of consciousness.

ANS: C The nurse can discern the orientation of cognitive function through the course of the interview or can directly and tactfully ask, Some people have trouble keeping up with the dates while in the hospital. Do you know todays date? Many hospitalized people have trouble with the exact date but are fully oriented on the remaining items

A patient with a lack of oxygen to his heart will have pain in his chest and possibly in the shoulder, arms, or jaw. The nurse knows that the best explanation why this occurs is which one of these statements? a. A problem exists with the sensory cortex and its ability to discriminate the location. b. The lack of oxygen in his heart has resulted in decreased amount of oxygen to the areas experiencing the pain. c. The sensory cortex does not have the ability to localize pain in the heart; consequently, the pain is felt elsewhere. d. A lesion has developed in the dorsal root, which is preventing the sensation from being transmitted normally.

ANS: C The sensory cortex is arranged in a specific pattern, forming a corresponding map of the body. Pain in the right hand is perceived at a specific spot on the map. Some organs, such as the heart, liver, and spleen, are absent from the brain map. Pain originating in these organs is referred because no felt image exists in which to have pain. Pain is felt by proxy, that is, by another body part that does have a felt image. The other responses are not correct explanations.

The area of the nervous system that is responsible for mediating reflexes is the: a. Medulla. b. Cerebellum. c. Spinal cord. d. Cerebral cortex.

ANS: C The spinal cord is the main highway for ascending and descending fiber tracts that connect the brain to the spinal nerves; it is responsible for mediating reflexes.

While gathering equipment after an injection, a nurse accidentally received a prick from an improperly capped needle. To interpret this sensation, which of these areas must be intact? a. Corticospinal tract, medulla, and basal ganglia b. Pyramidal tract, hypothalamus, and sensory cortex c. Lateral spinothalamic tract, thalamus, and sensory cortex d. Anterior spinothalamic tract, basal ganglia, and sensory cortex

ANS: C The spinothalamic tract contains sensory fibers that transmit the sensations of pain, temperature, and crude or light touch. Fibers carrying pain and temperature sensations ascend the lateral spinothalamic tract, whereas the sensations of crude touch form the anterior spinothalamic tract. At the thalamus, the fibers synapse with another sensory neuron, which carries the message to the sensory cortex for full interpretation. The other options are not correct.

To palpate the temporomandibular joint, the nurses fingers should be placed in the depression __________ of the ear. a. Distal to the helix b. Proximal to the helix c. Anterior to the tragus d. Posterior to the tragus

ANS: C The temporomandibular joint can be felt in the depression anterior to the tragus of the ear. The other locations are not correct.

During an examination, the nurse notices that a male patient has a red, round, superficial ulcer with a yellowish serous discharge on his penis. On palpation, the nurse finds a nontender base that feels like a small button between the thumb and fingers. At this point the nurse suspects that this patient has: a. Genital warts. b. Herpes infection. c. Syphilitic chancre. d. Carcinoma lesion.

ANS: C This lesion indicates syphilitic chancre, which begins within 2 to 4 weeks of infection. (See Table 24-4 for the descriptions of the other options.

The nurse is performing an assessment on a 29-year-old woman who visits the clinic complaining of always dropping things and falling down. While testing rapid alternating movements, the nurse notices that the woman is unable to pat both of her knees. Her response is extremely slow and she frequently misses. What should the nurse suspect? a. Vestibular disease b. Lesion of CN IX c. Dysfunction of the cerebellum d. Inability to understand directions

ANS: C When a person tries to perform rapid, alternating movements, responses that are slow, clumsy, and sloppy are indicative of cerebellar disease. The other responses are incorrect.

If an American Indian woman has come to the clinic to seek help with regulating her diabetes, then the nurse can expect that she: a. Will comply with the treatment prescribed. b. Has obviously given up her belief in naturalistic causes of disease. c. May also be seeking the assistance of a shaman or medicine man. d. Will need extra help in dealing with her illness and may be experiencing a crisis of faith.

ANS: C When self-treatment is unsuccessful, the individual may turn to the lay or folk healing systems, to spiritual or religious healing, or to scientific biomedicine. In addition to seeking help from a biomedical or scientific health care provider, patients may also seek help from folk or religious healers.

A 30-year-old female patient is describing feelings of hopelessness and depression. She has attempted self-mutilation and has a history of suicide attempts. She describes difficulty sleeping at night and has lost 10 pounds in the past month. Which of these statements or questions is the nurses best response in this situation? a. Do you have a weapon? b. How do other people treat you? c. Are you feeling so hopeless that you feel like hurting yourself now? d. People often feel hopeless, but the feelings resolve within a few weeks.

ANS: C When the person expresses feelings of hopelessness, despair, or grief, assessing the risk of physical harm to him or herself is important. This process begins with more general questions. If the answers are affirmative, then the assessment continues with more specific questions.

The nurse is testing superficial reflexes on an adult patient. When stroking up the lateral side of the sole and across the ball of the foot, the nurse notices the plantar flexion of the toes. How should the nurse document this finding? a. Positive Babinski sign b. Plantar reflex abnormal c. Plantar reflex present d. Plantar reflex 2+ on a scale from 0 to 4+

ANS: C With the same instrument, the nurse should draw a light stroke up the lateral side of the sole of the foot and across the ball of the foot, similar to an upside-down J. The normal response is plantar flexion of the toes and sometimes of the entire foot. A positive Babinski sign is abnormal and occurs with the response of dorsiflexion of the big toe and fanning of all toes. The plantar reflex is not graded on a 0 to 4+ scale.

While the nurse is taking the history of a 68-year-old patient who sustained a head injury 3 days earlier, he tells the nurse that he is on a cruise ship and is 30 years old. The nurse knows that this finding is indicative of a(n): a. Great sense of humor. b. Uncooperative behavior. c. Inability to understand questions. d. Decreased level of consciousness.

ANS: D A change in consciousness may be subtle. The nurse should notice any decreasing level of consciousness, disorientation, memory loss, uncooperative behavior, or even complacency in a previously combative person. The other responses are incorrect.

A young swimmer comes to the sports clinic complaining of a very sore shoulder. He was running at the pool, slipped on some wet concrete, and tried to catch himself with his outstretched hand. He landed on his outstretched hand and has not been able to move his shoulder since. The nurse suspects: a. Joint effusion. b. Tear of rotator cuff. c. Adhesive capsulitis. d. Dislocated shoulder.

ANS: D A dislocated shoulder occurs with trauma involving abduction, extension, and external rotation (e.g., falling on an outstretched arm or diving into a pool). (See Table 22-2 for descriptions of the other conditions.)

A 62-year-old man states that his physician told him that he has an inguinal hernia. He asks the nurse to explain what a hernia is. The nurse should: a. Tell him not to worry and that most men his age develop hernias. b. Explain that a hernia is often the result of prenatal growth abnormalities. c. Refer him to his physician for additional consultation because the physician made the initial diagnosis. d. Explain that a hernia is a loop of bowel protruding through a weak spot in the abdominal muscles.

ANS: D A hernia is a loop of bowel protruding through a weak spot in the musculature. The other options are not correct responses to the patients question.

Which of these statements about the peripheral nervous system is correct? a. The CNs enter the brain through the spinal cord. b. Efferent fibers carry sensory input to the central nervous system through the spinal cord. c. The peripheral nerves are inside the central nervous system and carry impulses through their motor fibers. d. The peripheral nerves carry input to the central nervous system by afferent fibers and away from the central nervous system by efferent fibers.

ANS: D A nerve is a bundle of fibers outside of the central nervous system. The peripheral nerves carry input to the central nervous system by their sensory afferent fibers and deliver output from the central nervous system by their efferent fibers. The other responses are not related to the peripheral nervous system.

During an examination, the nurse notices a patterned injury on a patients back. Which of these would cause such an injury? a. Blunt force b. Friction abrasion c. Stabbing from a kitchen knife d. Whipping from an extension cord

ANS: D A patterned injury is an injury caused by an object that leaves a distinct pattern on the skin or organ. The other actions do not cause a patterned injury.

The nurse has completed the musculoskeletal examination of a patients knee and has found a positive bulge sign. The nurse interprets this finding to indicate: a. Irregular bony margins. b. Soft-tissue swelling in the joint. c. Swelling from fluid in the epicondyle. d. Swelling from fluid in the suprapatellar pouch.

ANS: D A positive bulge sign confirms the presence of swelling caused by fluid in the suprapatellar pouch. The other options are not correct.

The nurse is providing patient education for a man who has been diagnosed with a rotator cuff injury. The nurse knows that a rotator cuff injury involves the: a. Nucleus pulposus. b. Articular processes. c. Medial epicondyle. d. Glenohumeral joint.

ANS: D A rotator cuff injury involves the glenohumeral joint, which is enclosed by a group of four powerful muscles and tendons that support and stabilize it. The nucleus pulposus is located in the center of each intervertebral disk. The articular processes are projections in each vertebral disk that lock onto the next vertebra, thereby stabilizing the spinal column. The medial epicondyle is located at the elbow

A 15-year-old boy is seen in the clinic for complaints of dull pain and pulling in the scrotal area. On examination, the nurse palpates a soft, irregular mass posterior to and above the testis on the left. This mass collapses when the patient is supine and refills when he is upright. This description is consistent with: a. Epididymitis. b. Spermatocele. c. Testicular torsion. d. Varicocele.

ANS: D A varicocele consists of dilated, tortuous varicose veins in the spermatic cord caused by incompetent valves within the vein. Symptoms include dull pain or a constant pulling or dragging feeling, or the individual may be asymptomatic. When palpating the mass, the examiner will feel a soft, irregular mass posterior to and above the testis that collapses when the individual is supine and refills when the individual is upright. (See Table 24-6 for more information and for the descriptions of the other options.)

When the nurse asks a 68-year-old patient to stand with his feet together and arms at his side with his eyes closed, he starts to sway and moves his feet farther apart. The nurse would document this finding as: a. Ataxia. b. Lack of coordination. c. Negative Homans sign. d. Positive Romberg sign.

ANS: D Abnormal findings for the Romberg test include swaying, falling, and a widening base of the feet to avoid falling. A positive Romberg sign is a loss of balance that is increased by the closing of the eyes. Ataxia is an uncoordinated or unsteady gait. Homans sign is used to test the legs for deep-vein thrombosis.

After a symptom is recognized, the first effort at treatment is often self-care. Which of the following statements about self-care is true? Self-care is: a. Not recognized as valuable by most health care providers. b. Usually ineffective and may delay more effective treatment. c. Always less expensive than biomedical alternatives. d. Influenced by the accessibility of over-the-counter medicines.

ANS: D After a symptom is identified, the first effort at treatment is often self-care. The availability of over-the-counter medications, the relatively high literacy level of Americans, and the influence of the mass media in communicating health-related information to the general population have contributed to the high percentage of cases of self-treatment.

When assessing aging adults, the nurse knows that one of the first things that should be assessed before making judgments about their mental status is: a. Presence of phobias b. General intelligence c. Presence of irrational thinking patterns d. Sensory-perceptive abilities

ANS: D Age-related changes in sensory perception can affect mental status. For example, vision loss (as detailed in Chapter 14) may result in apathy, social isolation, and depression. Hearing changes are common in older adults, which produces frustration, suspicion, and social isolation and makes the person appear confused.

A 70-year-old woman tells the nurse that every time she gets up in the morning or after shes been sitting, she gets really dizzy and feels like she is going to fall over. The nurses best response would be: a. Have you been extremely tired lately? b. You probably just need to drink more liquids. c. Ill refer you for a complete neurologic examination. d. You need to get up slowly when youve been lying down or sitting.

ANS: D Aging is accompanied by a progressive decrease in cerebral blood flow. In some people, this decrease causes dizziness and a loss of balance with a position change. These individuals need to be taught to get up slowly. The other responses are incorrect.

A patient has been diagnosed with schizophrenia. During a recent interview, he shows the nurse a picture of a man holding a decapitated head. He describes this picture as horrifying but then laughs loudly at the content. This behavior is a display of: a. Confusion b. Ambivalence c. Depersonalization d. Inappropriate affect

ANS: D An inappropriate affect is an affect clearly discordant with the content of the persons speech.

Which of these statements is true regarding the penis? a. The urethral meatus is located on the ventral side of the penis. b. The prepuce is the fold of foreskin covering the shaft of the penis. c. The penis is made up of two cylindrical columns of erectile tissue. d. The corpus spongiosum expands into a cone of erectile tissue called the glans.

ANS: D At the distal end of the shaft, the corpus spongiosum expands into a cone of erectile tissue, the glans. The penis is made up of three cylindrical columns of erectile tissue. The skin that covers the glans of the penis is the prepuce. The urethral meatus forms at the tip of the glans.

A 23-year-old patient in the clinic appears anxious. Her speech is rapid, and she is fidgety and in constant motion. Which of these questions or statements would be most appropriate for the nurse to use in this situation to assess attention span? a. How do you usually feel? Is this normal behavior for you? b. I am going to say four words. In a few minutes, I will ask you to recall them. c. Describe the meaning of the phrase, Looking through rose-colored glasses. d. Pick up the pencil in your left hand, move it to your right hand, and place it on the table.

ANS: D Attention span is evaluated by assessing the individuals ability to concentrate and complete a thought or task without wandering. Giving a series of directions to follow is one method used to assess attention span.

A 32-year-old woman tells the nurse that she has noticed very sudden, jerky movements mainly in her hands and arms. She says, They seem to come and go, primarily when I am trying to do something. I havent noticed them when Im sleeping. This description suggests: a. Tics. b. Athetosis. c. Myoclonus. d. Chorea.

ANS: D Chorea is characterized by sudden, rapid, jerky, purposeless movements that involve the limbs, trunk, or face. Chorea occurs at irregular intervals, and the movements are all accentuated by voluntary actions

The nurse notices that a woman in an exercise class is unable to jump rope. The nurse is aware that to jump rope, ones shoulder has to be capable of: a. Inversion. b. Supination. c. Protraction. d. Circumduction.

ANS: D Circumduction is defined as moving the arm in a circle around the shoulder. The other options are not correct

The nurse manager is explaining culturally competent care during a staff meeting. Which statement accurately describes the concept of culturally competent care? The caregiver: a. Is able to speak the patients native language. b. Possesses some basic knowledge of the patients cultural background. c. Applies the proper background knowledge of a patients cultural background to provide the best possible health care. d. Understands and attends to the total context of the patients situation.

ANS: D Culturally competent implies that the caregiver understands and attends to the total context of the individuals situation. This competency includes awareness of immigration status, stress factors, other social factors, and cultural similarities and differences. It does not require the caregiver to speak the patients native language.

During a class on the aspects of culture, the nurse shares that culture has four basic characteristics. Which statement correctly reflects one of these characteristics? a. Cultures are static and unchanging, despite changes around them. b. Cultures are never specific, which makes them hard to identify. c. Culture is most clearly reflected in a persons language and behavior. d. Culture adapts to specific environmental factors and available natural resources.

ANS: D Culture has four basic characteristics. Culture adapts to specific conditions related to environmental and technical factors and to the availability of natural resources, and it is dynamic and ever changing. Culture is learned from birth through the process of language acquisition and socialization, but it is not most clearly reflected in ones language and behavior.

A 20-year-old construction worker has been brought into the emergency department with heat stroke. He has delirium as a result of a fluid and electrolyte imbalance. For the mental status examination, the nurse should first assess the patients: a. Affect and mood b. Memory and affect c. Language abilities d. Level of consciousness and cognitive abilities

ANS: D Delirium is a disturbance of consciousness (i.e., reduced clarity of awareness of the environment) with reduced ability to focus, sustain, or shift attention. Delirium is not an alteration in mood, affect, or language abilities.

Fibrous bands running directly from one bone to another that strengthen the joint and help prevent movement in undesirable directions are called: a. Bursa. b. Tendons. c. Cartilage. d. Ligaments.

ANS: D Fibrous bands running directly from one bone to another that strengthen the joint and help prevent movement in undesirable directions are called ligaments. The other options are not correct.

A 19-year-old woman comes to the clinic at the insistence of her brother. She is wearing black combat boots and a black lace nightgown over the top of her other clothes. Her hair is dyed pink with black streaks throughout. She has several pierced holes in her nares and ears and is wearing an earring through her eyebrow and heavy black makeup. The nurse concludes that: a. She probably does not have any problems. b. She is only trying to shock people and that her dress should be ignored. c. She has a manic syndrome because of her abnormal dress and grooming. d. More information should be gathered to decide whether her dress is appropriate.

ANS: D Grooming and hygiene should be notedthe person is clean and well groomed, hair is neat and clean, women have moderate or no makeup, and men are shaved or their beards or moustaches are well groomed. Care should be taken when interpreting clothing that is disheveled, bizarre, or in poor repair because these sometimes reflect the persons economic status or a deliberate fashion trend.

When planning a cultural assessment, the nurse should include which component? a. Family history b. Chief complaint c. Medical history d. Health-related beliefs

ANS: D Health-related beliefs and practices are one component of a cultural assessment. The other items reflect other aspects of the patients history

During a health history, a patient tells the nurse that he has trouble in starting his urine stream. This problem is known as: a. Urgency. b. Dribbling. c. Frequency. d. Hesitancy.

ANS: D Hesitancy is trouble in starting the urine stream. Urgency is the feeling that one cannot wait to urinate. Dribbling is the last of the urine before or after the main act of urination. Frequency is urinating more often than usual.

In assessing a 70-year-old patient who has had a recent cerebrovascular accident, the nurse notices right-sided weakness. What might the nurse expect to find when testing his reflexes on the right side? a. Lack of reflexes b. Normal reflexes c. Diminished reflexes d. Hyperactive reflexes

ANS: D Hyperreflexia is the exaggerated reflex observed when the monosynaptic reflex arc is released from the influence of higher cortical levels. This response occurs with upper motor neuron lesions (e.g., a cerebrovascular accident). The other responses are incorrect.

The nurse is performing a genital examination on a male patient and notices urethral drainage. When collecting urethral discharge for microscopic examination and culture, the nurse should: a. Ask the patient to urinate into a sterile cup. b. Ask the patient to obtain a specimen of semen. c. Insert a cotton-tipped applicator into the urethra. d. Compress the glans between the examiners thumb and forefinger, and collect any discharge.

ANS: D If urethral discharge is noticed, then the examiner should collect a smear for microscopic examination and culture by compressing the glans anteroposteriorly between the thumb and forefinger. The other options are not correct actions.

In the hot/cold theory, illnesses are believed to be caused by hot or cold entering the body. Which of these patient conditions is most consistent with a cold condition? a. Patient with diabetes and renal failure b. Teenager with an abscessed tooth c. Child with symptoms of itching and a rash d. Older man with gastrointestinal discomfort

ANS: D Illnesses believed to be caused by cold entering the body include earache, chest cramps, gastrointestinal discomfort, rheumatism, and tuberculosis. Those illnesses believed to be caused by heat, or overheating, include sore throats, abscessed teeth, rashes, and kidney disorders

A 26-year-old woman was robbed and beaten a month ago. She is returning to the clinic today for a follow-up assessment. The nurse will want to ask her which one of these questions? a. How are things going with the trial? b. How are things going with your job? c. Tell me about your recent engagement! d. Are you having any disturbing dreams?

ANS: D In posttraumatic stress disorder, the person has been exposed to a traumatic event. The traumatic event is persistently reexperienced by recurrent and intrusive, distressing recollections of the event, including images, thoughts, or perceptions; recurrent distressing dreams of the event; and acting or feeling as if the traumatic event were recurring.

The nurse is explaining to a patient that there are shock absorbers in his back to cushion the spine and to help it move. The nurse is referring to his: a. Vertebral column. b. Nucleus pulposus. c. Vertebral foramen. d. Intervertebral disks.

ANS: D Intervertebral disks are elastic fibrocartilaginous plates that cushion the spine similar to shock absorbers and help it move. The vertebral column is the spinal column itself. The nucleus pulposus is located in the center of each disk. The vertebral foramen is the channel, or opening, for the spinal cord in the vertebrae.

The nurse is assessing orientation in a 79-year-old patient. Which of these responses would lead the nurse to conclude that this patient is oriented? a. I know my name is John. I couldn't tell you where I am. I think it is 2010, though. b. I know my name is John, but to tell you the truth, I get kind of confused about the date. c. I know my name is John; I guess Im at the hospital in Spokane. No, I don't know the date. d. I know my name is John. I am at the hospital in Spokane. I couldn't tell you what date it is, but I know that it is February of a new year2010.

ANS: D Many aging persons experience social isolation, loss of structure without a job, a change in residence, or some short-term memory loss. These factors affect orientation, and the person may not provide the precise date or complete name of the agency. You may consider aging persons oriented if they generally know where they are and the present period. They should be considered oriented to time if the year and month are correctly stated. Orientation to place is accepted with the correct identification of the type of setting (e.g., hospital) and the name of the town.

As a mandatory reporter of elder abuse, which must be present before a nurse should notify the authorities? a. Statements from the victim b. Statements from witnesses c. Proof of abuse and/or neglect d. Suspicion of elder abuse and/or neglect

ANS: D Many health care workers are under the erroneous assumption that proof is required before notification of suspected abuse can occur. Only the suspicion of elder abuse or neglect is necessary.

A woman who has just discovered that she is pregnant is in the clinic for her first obstetric visit. She asks the nurse, How many drinks a day is safe for my baby? The nurses best response is: a. You should limit your drinking to once or twice a week. b. Its okay to have up to two glasses of wine a day. c. As long as you avoid getting drunk, you should be safe. d. No amount of alcohol has been determined to be safe during pregnancy.

ANS: D No amount of alcohol has been determined to be safe for pregnant women. The potential adverse effects of alcohol use on the fetus are well known; women who are pregnant should be screened for alcohol use, and abstinence should be recommended.

The nurse is using the danger assessment (DA) tool to evaluate the risk of homicide. Which of these statements best describes its use? a. The DA tool is to be administered by law enforcement personnel. b. The DA tool should be used in every assessment of suspected abuse. c. The number of yes answers indicates the womans understanding of her situation. d. The higher the number of yes answers, the more serious the danger of the womans situation.

ANS: D No predetermined cutoff scores exist on the DA. The higher the number yes answers, the more serious the danger of the womans situation. The use of this tool is not limited to law enforcement personnel and is not required in every case of suspected abuse.

The nurse is examining a 3-month-old infant. While the nurse holds his or her thumbs on the infants inner mid thighs and the fingers on the outside of the infants hips, touching the greater trochanter, the nurse adducts the legs until the his or her thumbs touch and then abducts the legs until the infants knees touch the table. The nurse does not notice any clunking sounds and is confident to record a: a. Positive Allis test. b. Negative Allis test. c. Positive Ortolani sign. d. Negative Ortolani sign.

ANS: D Normally, this maneuver feels smooth and has no sound. With a positive Ortolani sign, however, the nurse will feel and hear a clunk, as the head of the femur pops back into place. A positive Ortolani sign also reflects hip instability. The Allis test also tests for hip dislocation but is performed by comparing leg lengths.

The nurse is providing patient teaching about an erectile dysfunction drug. One of the drugs potential side effects is prolonged, painful erection of the penis without sexual stimulation, which is known as: a. Orchitis. b. Stricture. c. Phimosis. d. Priapism.

ANS: D Priapism is prolonged, painful erection of the penis without sexual desire. Orchitis is inflammation of the testes. Stricture is a narrowing of the opening of the urethral meatus. Phimosis is the inability to retract the foreskin.

During a class on religion and spirituality, the nurse is asked to define spirituality. Which answer is correct? Spirituality: a. Is a personal search to discover a supreme being. b. Is an organized system of beliefs concerning the cause, nature, and purpose of the universe. c. Is a belief that each person exists forever in some form, such as a belief in reincarnation or the afterlife. d. Arises out of each persons unique life experience and his or her personal effort to find purpose in life.

ANS: D Spirituality arises out of each persons unique life experience and his or her personal effort to find purpose and meaning in life. The other definitions reflect the concept of religion.

A 68-year-old woman has come in for an assessment of her rheumatoid arthritis, and the nurse notices raised, firm, nontender nodules at the olecranon bursa and along the ulna. These nodules are most commonly diagnosed as: a. Epicondylitis. b. Gouty arthritis. c. Olecranon bursitis. d. Subcutaneous nodules.

ANS: D Subcutaneous nodules are raised, firm, and nontender and occur with rheumatoid arthritis in the olecranon bursa and along the extensor surface of the ulna. (See Table 22-3 for a description of the other conditions.)

Which statement is best for the nurse to use when preparing to administer the Abuse Assessment Screen? a. We are required by law to ask these questions. b. We need to talk about whether you believe you have been abused. c. We are asking these questions because we suspect that you are being abused. d. We need to ask the following questions because domestic violence is so common in our society.

ANS: D Such an introduction alerts the woman that questions about domestic violence are coming and ensures the woman that she is not being singled out for these questions.

Which of the following reflects the traditional health and illness beliefs and practices of those of African heritage? Health is: a. Being rewarded for good behavior. b. The balance of the body and spirit. c. Maintained by wearing jade amulets. d. Being in harmony with nature.

ANS: D The belief that health is being in harmony with nature reflects the health beliefs of those of African heritages. The other examples represent Iberian and Central and South American heritages, American-Indian heritages, and Asian heritages (See Table 2-3)

A professional tennis player comes into the clinic complaining of a sore elbow. The nurse will assess for tenderness at the: a. Olecranon bursa. b. Annular ligament. c. Base of the radius. d. Medial and lateral epicondyle.

ANS: D The epicondyles, the head of the radius, and the tendons are common sites of inflammation and local tenderness, commonly referred to as tennis elbow. The other locations are not affected.

The nurse is testing the function of CN XI. Which statement best describes the response the nurse should expect if this nerve is intact? The patient: a. Demonstrates the ability to hear normal conversation. b. Sticks out the tongue midline without tremors or deviation. c. Follows an object with his or her eyes without nystagmus or strabismus. d. Moves the head and shoulders against resistance with equal strength.

ANS: D The following normal findings are expected when testing the spinal accessory nerve (CN XI): The patients sternomastoid and trapezius muscles are equal in size; the person can forcibly rotate the head both ways against resistance applied to the side of the chin with equal strength; and the patient can shrug the shoulders against resistance with equal strength on both sides. Checking the patients ability to hear normal conversation checks the function of CN VIII. Having the patient stick out the tongue checks the function of CN XII. Testing the eyes for nystagmus or strabismus is performed to check CNs III, IV, and VI.

The nurse is examining the glans and knows which finding is normal for this area? a. The meatus may have a slight discharge when the glans is compressed. b. Hair is without pest inhabitants. c. The skin is wrinkled and without lesions. d. Smegma may be present under the foreskin of an uncircumcised male.

ANS: D The glans looks smooth and without lesions and does not have hair. The meatus should not have any discharge when the glans is compressed. Some cheesy smegma may have collected under the foreskin of an uncircumcised male.

The nurse is reviewing the hot/cold theory of health and illness. Which statement best describes the basic tenets of this theory? a. The causation of illness is based on supernatural forces that influence the humors of the body. b. Herbs and medicines are classified on their physical characteristics of hot and cold and the humors of the body. c. The four humors of the body consist of blood, yellow bile, spiritual connectedness, and social aspects of the individual. d. The treatment of disease consists of adding or subtracting cold, heat, dryness, or wetness to restore the balance of the humors of the body.

ANS: D The hot/cold theory of health and illness is based on the four humors of the body: blood, phlegm, black bile, and yellow bile. These humors regulate the basic bodily functions, described in terms of temperature, dryness, and moisture. The treatment of disease consists of adding or subtracting cold, heat, dryness, or wetness to restore the balance of the humors. The other statements are not correct.

When the nurse is conducting sexual history from a male adolescent, which statement would be most appropriate to use at the beginning of the interview? a. Do you use condoms? b. You dont masturbate, do you? c. Have you had sex in the last 6 months? d. Often adolescents your age have questions about sexual activity.

ANS: D The interview should begin with a permission statement, which conveys that it is normal and acceptable to think or feel a certain way. Sounding judgmental should be avoided.

When performing a genital assessment on a middle-aged man, the nurse notices multiple soft, moist, painless papules in the shape of cauliflower-like patches scattered across the shaft of the penis. These lesions are characteristic of: a. Carcinoma. b. Syphilitic chancres. c. Genital herpes. d. Genital warts.

ANS: D The lesions of genital warts are soft, pointed, moist, fleshy, painless papules that may be single or multiple in a cauliflower-like patch. They occur on the shaft of the penis, behind the corona, or around the anus, where they may grow into large grapelike clusters. (See Table 24-4 for more information and for the descriptions of the other options.)

The nurse is performing a mental status assessment on a 5-year-old girl. Her parents are undergoing a bitter divorce and are worried about the effect it is having on their daughter. Which action or statement might lead the nurse to be concerned about the girls mental status? a. She clings to her mother whenever the nurse is in the room. b. She appears angry and will not make eye contact with the nurse. c. Her mother states that she has begun to ride a tricycle around their yard. d. Her mother states that her daughter prefers to play with toddlers instead of kids her own age while in daycare.

ANS: D The mental status assessment of infants and children covers behavioral, cognitive, and psychosocial development and examines how the child is coping with his or her environment. Essentially, the nurse should follow the same Association for Behavioral and Cognitive Therapies (ABCT) guidelines as those for the adult, with special consideration for developmental milestones. The best examination technique arises from a thorough knowledge of the developmental milestones (described in Chapter 2). Abnormalities are often problems of omission (e.g., the child does not achieve a milestone as expected).

When providing culturally competent care, nurses must incorporate cultural assessments into their health assessments. Which statement is most appropriate to use when initiating an assessment of cultural beliefs with an older American-Indian patient? a. Are you of the Christian faith? b. Do you want to see a medicine man? c. How often do you seek help from medical providers? d. What cultural or spiritual beliefs are important to you?

ANS: D The nurse needs to assess the cultural beliefs and practices of the patient. American Indians may seek assistance from a medicine man or shaman, but the nurse should not assume this. An open-ended question regarding cultural and spiritual beliefs is best used initially when performing a cultural assessment

D Papilledema, or choked disk, is a serious sign of increased intracranial pressure, which is caused by a space-occupying mass such as a brain tumor or hematoma. This pressure causes venous stasis in the globe, showing redness, congestion, and elevation of the optic disc, blurred margins, hemorrhages, and absent venous pulsations. Papilledema is not associated with the conditions in the other responses.

An ophthalmic examination reveals papilledema. The nurse is aware that this finding indicates: a. Retinal detachment. b. Diabetic retinopathy. c. Acute-angle glaucoma. d. Increased intracranial pressure.

During the taking of the health history of a 78-year-old man, his wife states that he occasionally has problems with short-term memory loss and confusion: He cant even remember how to button his shirt. When assessing his sensory system, which action by the nurse is most appropriate? a. The nurse would not test the sensory system as part of the examination because the results would not be valid. b. The nurse would perform the tests, knowing that mental status does not affect sensory ability. c. The nurse would proceed with an explanation of each test, making certain that the wife understands. d. Before testing, the nurse would assess the patients mental status and ability to follow directions.

ANS: D The nurse should ensure the validity of the sensory system testing by making certain that the patient is alert, cooperative, comfortable, and has an adequate attention span. Otherwise, the nurse may obtain misleading and invalid results.

A 50-year-old woman is in the clinic for weakness in her left arm and leg that she has noticed for the past week. The nurse should perform which type of neurologic examination? a. Glasgow Coma Scale b. Neurologic recheck examination c. Screening neurologic examination d. Complete neurologic examination

ANS: D The nurse should perform a complete neurologic examination on an individual who has neurologic concerns (e.g., headache, weakness, loss of coordination) or who is showing signs of neurologic dysfunction. The Glasgow Coma Scale is used to define a persons level of consciousness. The neurologic recheck examination is appropriate for those who are demonstrating neurologic deficits. The screening neurologic examination is performed on seemingly well individuals who have no significant subjective findings from the health history.

During an assessment of a 32-year-old patient with a recent head injury, the nurse notices that the patient responds to pain by extending, adducting, and internally rotating his arms. His palms pronate, and his lower extremities extend with plantar flexion. Which statement concerning these findings is most accurate? This patients response: a. Indicates a lesion of the cerebral cortex. b. Indicates a completely nonfunctional brainstem. c. Is normal and will go away in 24 to 48 hours. d. Is a very ominous sign and may indicate brainstem injury

ANS: D These findings are all indicative of decerebrate rigidity, which is a very ominous condition and may indicate a brainstem injury.

A patient has had a cerebrovascular accident (stroke). He is trying very hard to communicate. He seems driven to speak and says, I buy obie get spirding and take my train. What is the best description of this patients problem? a. Global aphasia b. Brocas aphasia c. Echolalia d. Wernickes aphasia

ANS: D This type of communication illustrates Wernickes or receptive aphasia. The person can hear sounds and words but cannot relate them to previous experiences. Speech is fluent, effortless, and well articulated, but it has many paraphasias (word substitutions that are malformed or wrong) and neologisms (made-up words) and often lacks substantive words. Speech can be totally incomprehensible. Often, a great urge to speak is present. Repetition, reading, and writing also are impaired. Echolalia is an imitation or the repetition of another persons words or phrases. (See Table 5-4 for the definitions of the other disorders.)

man who has had gout for several years comes to the clinic with a problem with his toe. On examination, the nurse notices the presence of hard, painless nodules over the great toe; one has burst open with a chalky discharge. This finding is known as: a. Callus. b. Plantar wart. c. Bunion. d. Tophi.

ANS: D Tophi are collections of monosodium urate crystals resulting from chronic gout in and around the joint that cause extreme swelling and joint deformity. They appear as hard, painless nodules (tophi) over the metatarsophalangeal joint of the first toe and they sometimes burst with a chalky discharge (see Table 22-6). (See Table 22-6 for descriptions of the other conditions.)

The nurse is examining a 2-month-old infant and notices asymmetry of the infants gluteal folds. The nurse should assess for other signs of what disorder? a. Fractured clavicle b. Down syndrome c. Spina bifida d. Hip dislocation

ANS: D Unequal gluteal folds may accompany hip dislocation after 2 to 3 months of age, but some asymmetry may occur in healthy children. Further assessment is needed. The other responses are not correct.

During a home visit, the nurse notices that an older adult woman is caring for her bedridden husband. The woman states that this is her duty, she does the best she can, and her children come to help when they are in town. Her husband is unable to care for himself, and she appears thin, weak, and exhausted. The nurse notices that several of his prescription medication bottles are empty. This situation is best described by the term: a. Physical abuse. b. Financial neglect. c. Psychological abuse. d. Unintentional physical neglect

ANS: D Unintentional physical neglect may occur, despite good intentions, and is the failure of a family member or caregiver to provide basic goods or services. Physical abuse is defined as violent acts that result or could result in injury, pain, impairment, or disease. Financial neglect is defined as the failure to use the assets of the older person to provide services needed by him or her. Psychological abuse is defined as behaviors that result in mental anguish.

Which statement concerning the testes is true? a. The lymphatic vessels of the testes drain into the abdominal lymph nodes. b. The vas deferens is located along the inferior portion of each testis. c. The right testis is lower than the left because the right spermatic cord is longer. d. The cremaster muscle contracts in response to cold and draws the testicles closer to the body.

ANS: D When it is cold, the cremaster muscle contracts, which raises the scrotal sac and brings the testes closer to the body to absorb heat necessary for sperm viability. The lymphatic vessels of the testes drain into the inguinal lymph nodes. The vas deferens is located along the upper portion of each testis. The left testis is lower than the right because the left spermatic cord is longer.

When the nurse is performing a genital examination on a male patient, which action is correct? a. Auscultating for the presence of a bruit over the scrotum b. Palpating for the vertical chain of lymph nodes along the groin, inferior to the inguinal ligament c. Palpating the inguinal canal only if a bulge is present in the inguinal region during inspection d. Having the patient shift his weight onto the left (unexamined) leg when palpating for a hernia on the right side

ANS: D When palpating for the presence of a hernia on the right side, the male patient is asked to shift his weight onto the left (unexamined) leg. Auscultating for a bruit over the scrotum is not appropriate. When palpating for lymph nodes, the horizontal chain is palpated. The inguinal canal should be palpated whether a bulge is present or not.

When the nurse is performing a genital examination on a male patient, the patient has an erection. The nurses most appropriate action or response is to: a. Ask the patient if he would like someone else to examine him. b. Continue with the examination as though nothing has happened. c. Stop the examination, leave the room while stating that the examination will resume at a later time. d. Reassure the patient that this is a normal response and continue with the examination

ANS: D When the male patient has an erection, the nurse should reassure the patient that this is a normal physiologic response to touch and proceed with the rest of the examination. The other responses are not correct and may be perceived as judgmental.

Which of these individuals would the nurse consider at highest risk for a suicide attempt? a. Man who jokes about death b. Woman who, during a past episode of major depression, attempted suicide c. Adolescent who just broke up with her boyfriend and states that she would like to kill herself d. Older adult man who tells the nurse that he is going to join his wife in heaven tomorrow and plans to use a gun

ANS: D When the person expresses feelings of sadness, hopelessness, despair, or grief, assessing any possible risk of physical harm to him or herself is important. The interview should begin with more general questions. If the nurse hears affirmative answers, then he or she should continue with more specific questions. A precise suicide plan to take place in the next 24 to 48 hours with use of a lethal method constitutes high risk.

A 63-year-old Chinese-American man enters the hospital with complaints of chest pain, shortness of breath, and palpitations. Which statement most accurately reflects the nurses best course of action? a. The nurse should focus on performing a full cardiac assessment. b. The nurse should focus on psychosomatic complaints because the patient has just learned that his wife has cancer. c. This patient is not in any danger at present; therefore, the nurse should send him home with instructions to contact his physician. d. It is unclear what is happening with this patient; consequently, the nurse should perform an assessment in both the physical and the psychosocial realms.

ANS: D Wide cultural variations exist in the manner in which certain symptoms and disease conditions are perceived, diagnosed, labeled, and treated. Chinese-Americans sometimes convert mental experiences or states into bodily symptoms (e.g., complaining of cardiac symptoms because the center of emotion in the Chinese culture is the heart).

When performing a genitourinary assessment on a 16-year-old male adolescent, the nurse notices a swelling in the scrotum that increases with increased intra-abdominal pressure and decreases when he is lying down. The patient complains of pain when straining. The nurse knows that this description is most consistent with a(n) ______ hernia. a. Femoral b. Incisional c. Direct inguinal d. Indirect inguinal

ANS: D With indirect inguinal hernias, pain occurs with straining and a soft swelling increases with increased intra-abdominal pressure, which may decrease when the patient lies down. These findings do not describe the other hernias. (See Table 24-7 for the descriptions of femoral, direct inguinal, and indirect inguinal hernias.)

A 78-year-old man has a history of a cerebrovascular accident. The nurse notes that when he walks, his left arm is immobile against the body with flexion of the shoulder, elbow, wrist, and fingers and adduction of the shoulder. His left leg is stiff and extended and circumducts with each step. What type of gait disturbance is this individual experiencing? a. Scissors gait b. Cerebellar ataxia c. Parkinsonian gait d. Spastic hemiparesis

ANS: D With spastic hemiparesis, the arm is immobile against the body. Flexion of the shoulder, elbow, wrist, and fingers occurs, and adduction of the shoulder, which does not swing freely, is observed. The leg is stiff and extended and circumducts with each step. Causes of this type of gait include cerebrovascular accident. (See Table 23-6 for more information and for the descriptions of the other abnormal gaits.)

The nurse recognizes that categories such as ethnicity, gender, and religion illustrate the concept of: a. Family. b. Cultures. c. Spirituality. d. Subcultures

ANS: D Within cultures, groups of people share different beliefs, values, and attitudes. Differences occur because of ethnicity, religion, education, occupation, age, and gender. When such groups function within a large culture, they are referred to as subcultural groups

A Objective data are what the health professional observes by inspecting, percussing, palpating, and auscultating during the physical examination. Subjective data is what the person says about him or herself during history taking. The terms reflective and introspective are not used to describe data.

After completing an initial assessment of a patient, the nurse has charted that his respirations are eupneic and his pulse is 58 beats per minute. These types of data would be: a. Objective. b. Reflective. c. Subjective. d. Introspective.

D Frostbite causes reddish-blue discoloration and swelling of the auricle after exposure to extreme cold. Vesicles or bullae may develop, and the person feels pain and tenderness.

An assessment of a 23-year-old patient reveals the following: an auricle that is tender and reddish-blue in color with small vesicles. The nurse would need to know additional information that includes which of these? a. Any change in the ability to hear b. Any recent drainage from the ear c. Recent history of trauma to the ear d. Any prolonged exposure to extreme cold

When auscultating the heart of a newborn within 24 hours after birth, the examiner hears a continuous sound that mimics the sound of a machine. This finding most likely indicates

An expected sound caused by the nonclosure of the ductus arteriosis.

B Because of recent outbreaks of measles across the United States, the American Academy of Pediatrics (2006) recommends two doses of the MMR vaccine, one at 12 to 15 months of age and one at age 4 to 6 years.

As part of the health history of a 6-year-old boy at a clinic for a sports physical examination, the nurse reviews his immunization record and notes that his last measles-mumps-rubella (MMR) vaccination was at 15 months of age. What recommendation should the nurse make? a. No further MMR immunizations are needed. b. MMR vaccination needs to be repeated at 4 to 6 years of age. c. MMR immunization needs to be repeated every 4 years until age 21 years. d. A recommendation cannot be made until the physician is consulted.

A Reflection echoes the patients words, repeating part of what the person has just said. Reflection can also help express the feelings behind a persons words.

As the nurse enters a patients room, the nurse finds her crying. The patient states that she has just found out that the lump in her breast is cancer and says, Im so afraid of, um, you know. The nurses most therapeutic response would be to say in a gentle manner: a. Youre afraid you might lose your breast? b. No, Im not sure what you are talking about. c. Ill wait here until you get yourself under control, and then we can talk. d. I can see that you are very upset. Perhaps we should discuss this later.

D Heart failure causes decreased cardiac output when the heart fails as a pump and the circulation becomes backed up and congested. Signs and symptoms include dyspnea, orthopnea, paroxysmal nocturnal dyspnea, decreased blood pressure, dependent and pitting edema; anxiety; confusion; jugular vein distention; and fatigue. The S3 is associated with heart failure and is always abnormal after 35 years of age. The S3 may be the earliest sign of heart failure.

During a cardiac assessment on a 38-year-old patient in the hospital for chest pain, the nurse finds the following: jugular vein pulsations 4 cm above the sternal angle when the patient is elevated at 45 degrees, blood pressure 98/60 mm Hg, heart rate 130 beats per minute, ankle edema, difficulty breathing when supine, and an S3 on auscultation. Which of these conditions best explains the cause of these findings? a. Fluid overload b. Atrial septal defect c. MI d. Heart failure

A A thrill is a palpable vibration that signifies turbulent blood flow and accompanies loud murmurs. The absence of a thrill does not rule out the presence of a murmur.

During a cardiovascular assessment, the nurse knows that a thrill is: a. Vibration that is palpable. b. Palpated in the right epigastric area. c. Associated with ventricular hypertrophy. d. Murmur auscultated at the third intercostal space.

C An S4 heart sound is heard at the end of diastole when the atria contract (atrial systole) and when the ventricles are resistant to filling. The S4 occurs just before the S1.

During a cardiovascular assessment, the nurse knows that an S4 heart sound is: a. Heard at the onset of atrial diastole. b. Usually a normal finding in the older adult. c. Heard at the end of ventricular diastole. d. Heard best over the second left intercostal space with the individual sitting upright.

D The misuse of over-the-counter nasal medications irritates the mucosa, causing rebound swelling, which is a common problem.

During a checkup, a 22-year-old woman tells the nurse that she uses an over-the-counter nasal spray because of her allergies. She also states that it does not work as well as it used to when she first started using it. The best response by the nurse would be: a. You should never use over-the-counter nasal sprays because of the risk of addiction. b. You should try switching to another brand of medication to prevent this problem. c. Continuing to use this spray is important to keep your allergies under control. d. Using these nasal medications irritates the lining of the nose and may cause rebound swelling.

A The adults use of why questions usually implies blame and condemnation and places the person on the defensive. The other statements are not correct.

During a follow-up visit, the nurse discovers that a patient has not been taking his insulin on a regular basis. The nurse asks, Why havent you taken your insulin? Which statement is an appropriate evaluation of this question? a. This question may place the patient on the defensive. b. This question is an innocent search for information. c. Discussing his behavior with his wife would have been better. d. A direct question is the best way to discover the reasons for his behavior.

D Sputum, alone, is not diagnostic, but some conditions have characteristic sputum production. Pink, frothy sputum indicates pulmonary edema or it may be a side effect of sympathomimetic medications. Croup is associated with a barking cough, not sputum production. Tuberculosis may produce rust-colored sputum. Viral infections may produce white or clear mucoid sputum.

During a morning assessment, the nurse notices that the patients sputum is frothy and pink. Which condition could this finding indicate? a. Croup b. Tuberculosis c. Viral infection d. Pulmonary edema

B Hyphema is the term for blood in the anterior chamber and is a serious result of blunt trauma (a fist or a baseball) or spontaneous hemorrhage and may indicate scleral rupture or major intraocular trauma.

During a physical education class, a student is hit in the eye with the end of a baseball bat. When examined in the emergency department, the nurse notices the presence of blood in the anterior chamber of the eye. This finding indicates the presence of: a. Hypopyon. b. Hyphema. c. Corneal abrasion. d. Pterygium.

B A beginning examiner usually feels horrified when the patient starts crying. When the nurse says something that makes the person cry, the nurse should not think he or she has hurt the person. The nurse has simply hit on an important topic; therefore, moving on to a new topic is essential. The nurse should allow the person to cry and to express his or her feelings fully. The nurse can offer a tissue and wait until the crying subsides to talk.

During a prenatal check, a patient begins to cry as the nurse asks her about previous pregnancies. She states that she is remembering her last pregnancy, which ended in miscarriage. The nurses best response to her crying would be: a. Im so sorry for making you cry! b. I can see that you are sad remembering this. It is all right to cry. c. Why dont I step out for a few minutes until youre feeling better? d. I can see that you feel sad about this; why dont we talk about something else?

C Arterial ischemic ulcers occur at the toes, metatarsal heads, heels, and lateral ankle and are characterized by a pale ischemic base, well-defined edges, and no bleeding

During a routine office visit, a patient takes off his shoes and shows the nurse this awful sore that wont heal. On inspection, the nurse notes a 3-cm round ulcer on the left great toe, with a pale ischemic base, well-defined edges, and no drainage. The nurse should assess for other signs and symptoms of: a. Varicosities. b. Venous stasis ulcer. c. Arterial ischemic ulcer. d. Deep-vein thrombophlebitis.

D Facilitating support for EBP would include teaching the nurses how to conduct electronic searches; time to visit the library may not be available for many nurses. Actually conducting research studies may be helpful in the long-run but not an immediate solution to reviewing existing research.

During a staff meeting, nurses discuss the problems with accessing research studies to incorporate evidence-based clinical decision making into their practice. Which suggestion by the nurse manager would best help these problems? a. Form a committee to conduct research studies. b. Post published research studies on the units bulletin boards. c. Encourage the nurses to visit the library to review studies. d. Teach the nurses how to conduct electronic searches for research studies.

B Clarification should be used when the persons word choice is ambiguous or confusing. Clarification is also used to summarize the persons words or to simplify the words to make them clearer; the nurse should then ask if he or she is on the right track. The other responses give unwanted advice or do not offer a helpful response.

During a visit to the clinic, a patient states, The doctor just told me he thought I ought to stop smoking. He doesnt understand how hard Ive tried. I just dont know the best way to do it. What should I do? The nurses most appropriate response in this case would be: a. Id quit if I were you. The doctor really knows what he is talking about. b. Would you like some information about the different ways a person can quit smoking? c. Stopping your dependence on cigarettes can be very difficult. I understand how you feel. d. Why are you confused? Didnt the doctor give you the information about the smoking cessation program we offer?

B Superficial varicose veins are caused by incompetent distant valves in the veins, which results in the reflux of blood, producing dilated, tortuous veins. Varicose veins are more common in women, and pregnancy can also be a cause. Symptoms include aching, heaviness in the calf, easy fatigability, and night leg or foot cramps. Dilated, tortuous veins are observed on assessment.

During a visit to the clinic, a woman in her seventh month of pregnancy complains that her legs feel heavy in the calf and that she often has foot cramps at night. The nurse notices that the patient has dilated, tortuous veins apparent in her lower legs. Which condition is reflected by these findings? a. Deep-vein thrombophlebitis b. Varicose veins c. Lymphedema d. Raynaud phenomenon

C Asians and Native Americans are more likely to have dry cerumen, whereas Blacks and Whites usually have wet cerumen.

During an assessment of a 20-year-old Asian patient, the nurse notices that he has dry, flaky cerumen in his canal. What is the significance of this finding? This finding: a. Is probably the result of lesions from eczema in his ear. b. Represents poor hygiene. c. Is a normal finding, and no further follow-up is necessary. d. Could be indicative of change in cilia; the nurse should assess for hearing loss.

A Dry mouth occurs with dehydration or fever. The tongue has deep vertical fissures.

During an assessment of a 20-year-old patient with a 3-day history of nausea and vomiting, the nurse notices dry mucosa and deep vertical fissures in the tongue. These findings are reflective of: a. Dehydration. b. Irritation by gastric juices. c. A normal oral assessment. d. Side effects from nausea medication.

C Cold sores are groups of clear vesicles with a surrounding erythematous base. These evolve into pustules or crusts and heal in 4 to 10 days. The most likely site is the lip-skin junction. Infection often recurs in the same site. Recurrent herpes infections may be precipitated by sunlight, fever, colds, or allergy.

During an assessment of a 26 year old at the clinic for a spot on my lip I think is cancer, the nurse notices a group of clear vesicles with an erythematous base around them located at the lip-skin border. The patient mentions that she just returned from Hawaii. What would be the most appropriate response by the nurse? a. Tell the patient she needs to see a skin specialist. b. Discuss the benefits of having a biopsy performed on any unusual lesion. c. Tell the patient that these vesicles are indicative of herpes simplex I or cold sores and that they will heal in 4 to 10 days. d. Tell the patient that these vesicles are most likely the result of a riboflavin deficiency and discuss nutrition.

B A bruit is a blowing, swishing sound indicating blood flow turbulence; normally, none is present.

During an assessment of a 68-year-old man with a recent onset of right-sided weakness, the nurse hears a blowing, swishing sound with the bell of the stethoscope over the left carotid artery. This finding would indicate: a. Valvular disorder. b. Blood flow turbulence. c. Fluid volume overload. d. Ventricular hypertrophy.

D The apical impulse should occupy only one intercostal space, the fourth or fifth, and it should be at or medial to the midclavicular line.

During an assessment of a healthy adult, where would the nurse expect to palpate the apical impulse? a. Third left intercostal space at the midclavicular line b. Fourth left intercostal space at the sternal border c. Fourth left intercostal space at the anterior axillary line d. Fifth left intercostal space at the midclavicular line

B The nurse should use the profile sign (viewing the finger from the side) to detect early clubbing.

During an assessment, the nurse uses the profile sign to detect: a. Pitting edema. b. Early clubbing. c. Symmetry of the fingers. d. Insufficient capillary refill.

B A genogram (or pedigree) is a graphic family tree that uses symbols to depict the gender, relationship, and age of immediate blood relatives in at least three generations (parents, grandparents, siblings). The other options do not describe a genogram.

During an assessment of a patients family history, the nurse constructs a genogram. Which statement best describes a genogram? a. List of diseases present in a persons near relatives b. Graphic family tree that uses symbols to depict the gender, relationship, and age of immediate family members c. Drawing that depicts the patients family members up to five generations back d. Description of the health of a persons children and grandchildren

B Unequal chest expansion occurs when part of the lung is obstructed or collapsed, as with pneumonia, or when guarding to avoid postoperative incisional pain.

During an assessment of an adult, the nurse has noted unequal chest expansion and recognizes that this occurs in which situation? a. In an obese patient b. When part of the lung is obstructed or collapsed c. When bulging of the intercostal spaces is present d. When accessory muscles are used to augment respiratory effort

C Peripheral blood vessels grow more rigid with age, resulting in a rise in systolic blood pressure. Aging produces progressive enlargement of the intramuscular calf veins, not atrophy. The other options are not correct.

During an assessment of an older adult, the nurse should expect to notice which finding as a normal physiologic change associated with the aging process? a. Hormonal changes causing vasodilation and a resulting drop in blood pressure b. Progressive atrophy of the intramuscular calf veins, causing venous insufficiency c. Peripheral blood vessels growing more rigid with age, producing a rise in systolic blood pressure d. Narrowing of the inferior vena cava, causing low blood flow and increases in venous pressure resulting in varicosities

D Normally in dark-skinned people, small brown macules may be observed in the sclera.

During an assessment of the sclera of a black patient, the nurse would consider which of these an expected finding? a. Yellow fatty deposits over the cornea b. Pallor near the outer canthus of the lower lid c. Yellow color of the sclera that extends up to the iris d. Presence of small brown macules on the sclera

B, D, E Sen The sense of smell diminishes with cigarette smoking, chronic allergies, and aging. Chronic alcohol use, a history of strep throat, and herpes simplex virus I are not associated with changes in the sense of smell.

During an assessment, a patient mentions that I just cant smell like I used to. I can barely smell the roses in my garden. Why is that? For which possible causes of changes in the sense of smell will the nurse assess? Select all that apply. a. Chronic alcohol use b. Cigarette smoking c. Frequent episodes of strep throat d. Chronic allergies e. Aging f. Herpes simplex virus I

B The condition with episodes of abrupt, progressive tricolor changes of the fingers in response to cold, vibration, or stress is known as Raynaud disease

During an assessment, a patient tells the nurse that her fingers often change color when she goes out in cold weather. She describes these episodes as her fingers first turning white, then blue, then red with a burning, throbbing pain. The nurse suspects that she is experiencing: a. Lymphedema. b. Raynaud disease. c. Deep-vein thrombosis. d. Chronic arterial insufficiency.

B In this test, it normally takes 10 seconds or less for the color to return to the feet and 15 seconds for the veins of the feet to fill. Significant elevational pallor, as well as delayed venous filling, occurs with arterial insufficiency.

During an assessment, the nurse has elevated a patients legs 12 inches off the table and has had him wag his feet to drain off venous blood. After helping him sit up and dangle his legs over the side of the table, the nurse should expect that a normal finding at this point would be: a. Significant elevational pallor. b. Venous filling within 15 seconds. c. No change in the coloration of the skin. d. Color returning to the feet within 20 seconds of assuming a sitting position.

C Normal lung findings include symmetric chest expansion, resonant percussion tones, vesicular breath sounds over the peripheral lung fields, muffled voice sounds, and no adventitious sounds.

During an assessment, the nurse knows that expected assessment findings in the normal adult lung include the presence of: a. Adventitious sounds and limited chest expansion. b. Increased tactile fremitus and dull percussion tones. c. Muffled voice sounds and symmetric tactile fremitus. d. Absent voice sounds and hyperresonant percussion tones.

B Lymphedema after breast cancer causes unilateral swelling and nonpitting brawny edema, with overlying skin indurated. It is caused by the removal of lymph nodes with breast surgery or damage to lymph nodes and channels with radiation therapy for breast cancer, and lymphedema can impede drainage of lymph. The other responses are not correct.

During an assessment, the nurse notices that a patients left arm is swollen from the shoulder down to the fingers, with nonpitting brawny edema. The right arm is normal. The patient had a left-sided mastectomy 1 year ago. The nurse suspects which problem? a. Venous stasis b. Lymphedema c. Arteriosclerosis d. Deep-vein thrombosis

C The condition described is known as ectropion, and it occurs in older adults and is attributable to atrophy of the elastic and fibrous tissues. The lower lid does not approximate to the eyeball, and, as a result, the puncta cannot effectively siphon tears; excessive tearing results. Ptosis is a drooping of the upper eyelid. These signs do not suggest the presence of a foreign body in the eye or basal cell carcinoma.

During an assessment, the nurse notices that an older adult patient has tears rolling down his face from his left eye. Closer examination shows that the lower lid is loose and rolling outward. The patient complains of his eye feeling dry and itchy. Which action by the nurse is correct? a. Assessing the eye for a possible foreign body b. Documenting the finding as ptosis c. Assessing for other signs of ectropion d. Contacting the prescriber; these are signs of basal cell carcinoma

D The CAGE test is known as the cut down, annoyed, guilty, and eye-opener test. If a person answers yes to two or more of the four CAGE questions, then the nurse should suspect alcohol abuse and continue with a more complete substance abuse assessment.

During an assessment, the nurse uses the CAGE test. The patient answers yes to two of the questions. What could this be indicating? a. The patient is an alcoholic. b. The patient is annoyed at the questions. c. The patient should be thoroughly examined for possible alcohol withdrawal symptoms. d. The nurse should suspect alcohol abuse and continue with a more thorough substance abuse assessment.

D Take the time to give a short, simple explanation with a concrete explanation for any unfamiliar equipment that will be used on the child. Preschoolers are animistic; they imagine inanimate objects can come alive and have human characteristics. Thus a blood pressure cuff can wake up and bite or pinch.

During an examination of a 3-year-old child, the nurse will need to take her blood pressure. What might the nurse do to try to gain the childs full cooperation? a. Tell the child that the blood pressure cuff is going to give her arm a big hug. b. Tell the child that the blood pressure cuff is asleep and cannot wake up. c. Give the blood pressure cuff a name and refer to it by this name during the assessment. d. Tell the child that by using the blood pressure cuff, we can see how strong her muscles are.

B The sternal angle marks the site of tracheal bifurcation into the right and left main bronchi; it corresponds with the upper borders of the atria of the heart, and it lies above the fourth thoracic vertebra on the back.

During an examination of the anterior thorax, the nurse is aware that the trachea bifurcates anteriorly at the: a. Costal angle. b. Sternal angle. c. Xiphoid process. d. Suprasternal notch.

D No swelling, redness, or drainage from the puncta should be observed when it is pressed. Regurgitation of fluid from the puncta, when pressed, indicates duct blockage. The lacrimal glands are not functional at birth.

During an examination of the eye, the nurse would expect what normal finding when assessing the lacrimal apparatus? a. Presence of tears along the inner canthus b. Blocked nasolacrimal duct in a newborn infant c. Slight swelling over the upper lid and along the bony orbit if the individual has a cold d. Absence of drainage from the puncta when pressing against the inner orbital rim

B, D, F Open-angle glaucoma is the most common type of glaucoma; virtually no symptoms are exhibited. Vision loss begins with the peripheral vision, which often goes unnoticed because individuals learn to compensate intuitively by turning their heads. The other characteristics are those of closed-angle glaucoma.

During an examination, a patient states that she was diagnosed with open-angle glaucoma 2 years ago. The nurse assesses for characteristics of open-angle glaucoma. Which of these are characteristics of open-angle glaucoma?Select all that apply. a. Patient may experience sensitivity to light, nausea, and halos around lights. b. Patient experiences tunnel vision in the late stages. c. Immediate treatment is needed. d. Vision loss begins with peripheral vision. e. Open-angle glaucoma causes sudden attacks of increased pressure that cause blurred vision. f. Virtually no symptoms are exhibited.

A With objective vertigo, the patient feels like the room spins; with subjective vertigo, the person feels like he or she is spinning. Tinnitus is a sound that comes from within a person; it can be a ringing, crackling, or buzzing sound. It accompanies some hearing or ear disorders. Dizziness is not the same as true vertigo; the person who is dizzy may feel unsteady and lightheaded.

During an examination, the nurse notices that the patient stumbles a little while walking, and, when she sits down, she holds on to the sides of the chair. The patient states, It feels like the room is spinning! The nurse notices that the patient is experiencing: a. Objective vertigo. b. Subjective vertigo. c. Tinnitus. d. Dizziness.

C Tinnitus is a sound that comes from within a person; it can be a ringing, crackling, or buzzing sound. It accompanies some hearing or ear disorders.

During an examination, the patient states he is hearing a buzzing sound and says that it is driving me crazy! The nurse recognizes that this symptom indicates: a. Vertigo. b. Pruritus. c. Tinnitus. d. Cholesteatoma.

D Normally, the examiner may or may not see an apical impulse; when visible, it occupies the fourth or fifth intercostal space at or inside the midclavicular line. A heave or lift is a sustained forceful thrusting of the ventricle during systole. It occurs with ventricular hypertrophy as a result of increased workload. A right ventricular heave is seen at the sternal border; a left ventricular heave is seen at the apex.

During an inspection of the precordium of an adult patient, the nurse notices the chest moving in a forceful manner along the sternal border. This finding most likely suggests a(n): a. Normal heart. b. Systolic murmur. c. Enlargement of the left ventricle. d. Enlargement of the right ventricle.

D The persons position is noted. An open position with the extension of large muscle groups shows relaxation, physical comfort, and a willingness to share information. A closed position with the arms and legs crossed tends to look defensive and anxious. Any change in posture should be noted. If a person in a relaxed position suddenly tenses, then this change in posture suggests possible discomfort with the new topic.

During an interview, a parent of a hospitalized child is sitting in an open position. As the interviewer begins to discuss his sons treatment, however, he suddenly crosses his arms against his chest and crosses his legs. This changed posture would suggest that the parent is: a. Simply changing positions. b. More comfortable in this position. c. Tired and needs a break from the interview. d. Uncomfortable talking about his sons treatment.

C This statement is not based on ones inference or conclusion. It links events, makes associations, or implies cause. Interpretation also ascribes feelings and helps the person understand his or her own feelings in relation to the verbal message. The other statements do not reflect interpretation.

During an interview, a woman says, I have decided that I can no longer allow my children to live with their fathers violence, but I just cant seem to leave him. Using interpretation, the nurses best response would be: a. You are going to leave him? b. If you are afraid for your children, then why cant you leave? c. It sounds as if you might be afraid of how your husband will respond. d. It sounds as though you have made your decision. I think it is a good one.

D The open-ended question asks for narrative information. It states the topic to be discussed but only in general terms. The nurse should use it to begin the interview, to introduce a new section of questions, and whenever the person introduces a new topic.

During an interview, the nurse states, You mentioned having shortness of breath. Tell me more about that. Which verbal skill is used with this statement? a. Reflection b. Facilitation c. Direct question d. Open-ended question

C Social distance, 4 to 12 feet, is usually the distance category for most of the interview. Public distance, over 12 feet, is too much distance; the intimate zone is inappropriate, and the personal distance will be used for the physical assessment.

During an interview, the nurse would expect that most of the interview will take place at what distance? a. Intimate zone b. Personal distance c. Social distance d. Public distance

D If the labyrinth ever becomes inflamed, then it feeds the wrong information to the brain, creating a staggering gait and a strong, spinning, whirling sensation called vertigo.

During an interview, the patient states he has the sensation that everything around him is spinning. The nurse recognizes that the portion of the ear responsible for this sensation is the: a. Cochlea. b. CN VIII. c. Organ of Corti. d. Labyrinth.

C Some Blacks may have bluish lips and a dark line on the gingival margin; this appearance is a normal finding.

During an oral assessment of a 30-year-old Black patient, the nurse notices bluish lips and a dark line along the gingival margin. What action would the nurse perform in response to this finding? a. Check the patients hemoglobin for anemia. b. Assess for other signs of insufficient oxygen supply. c. Proceed with the assessment, knowing that this appearance is a normal finding. d. Ask if he has been exposed to an excessive amount of carbon monoxide.

B Bifid uvula, a condition in which the uvula is split either completely or partially, occurs in some Native-American groups.

During an oral examination of a 4-year-old Native-American child, the nurse notices that her uvula is partially split. Which of these statements is accurate? a. This condition is a cleft palate and is common in Native Americans. b. A bifid uvula may occur in some Native-American groups. c. This condition is due to an injury and should be reported to the authorities. d. A bifid uvula is palatinus, which frequently occurs in Native Americans.

D A colony of black or white dots on the drum or canal wall suggests a yeast or fungal infection (otomycosis).

During an otoscopic examination, the nurse notices an area of black and white dots on the tympanic membrane and the ear canal wall. What does this finding suggest? a. Malignancy b. Viral infection c. Blood in the middle ear d. Yeast or fungal infection

A During auscultation of breath sounds with a stethoscope, listening to one full respiration in each location is important. During the examination, the nurse should monitor the breathing and offer times for the person to breathe normally to prevent possible dizziness.

During auscultation of breath sounds, the nurse should correctly use the stethoscope in which of the following ways? a. Listening to at least one full respiration in each location b. Listening as the patient inhales and then going to the next site during exhalation c. Instructing the patient to breathe in and out rapidly while listening to the breath sounds d. If the patient is modest, listening to sounds over his or her clothing or hospital gown

C Pathologic conditions that increase lung density, such as pulmonary consolidation, will enhance the transmission of voice sounds, such as bronchophony

During auscultation of the lungs of an adult patient, the nurse notices the presence of bronchophony. The nurse should assess for signs of which condition? a. Airway obstruction b. Emphysema c. Pulmonary consolidation d. Asthma

B Crepitus is a coarse, crackling sensation palpable over the skin surface. It occurs in subcutaneous emphysema when air escapes from the lung and enters the subcutaneous tissue, such as after open thoracic injury or surgery.

During palpation of the anterior chest wall, the nurse notices a coarse, crackling sensation over the skin surface. On the basis of these findings, the nurse suspects: a. Tactile fremitus. b. Crepitus. c. Friction rub. d. Adventitious sounds.

D A dull percussion note indicates an abnormal density in the lungs, as with pneumonia, pleural effusion, atelectasis, or a tumor. Resonance is the expected finding in normal lung tissue.

During percussion, the nurse knows that a dull percussion note elicited over a lung lobe most likely results from: a. Shallow breathing. b. Normal lung tissue. c. Decreased adipose tissue. d. Increased density of lung tissue.

D A split S2 is a normal phenomenon that occurs toward the end of inspiration in some people. A split S2 is heard only in the pulmonic valve area, the second left interspace. When the split S2 is first heard, the nurse should not be tempted to ask the person to hold his or her breath so that the nurse can concentrate on the sounds. Breath holding will only equalize ejection times in the right and left sides of the heart and cause the split to go away. Rather, the nurse should concentrate on the split while watching the persons chest rise up and down with breathing.

During the cardiac auscultation, the nurse hears a sound immediately occurring after the S2 at the second left intercostal space. To further assess this sound, what should the nurse do? a. Have the patient turn to the left side while the nurse listens with the bell of the stethoscope. b. Ask the patient to hold his or her breath while the nurse listens again. c. No further assessment is needed because the nurse knows this sound is an S3. d. Watch the patients respirations while listening for the effect on the sound.

D The interview is the first, and really the most important, part of data collection. During the interview, the nurse collects subjective data; that is, what the person says about him or herself.

During the interview portion of data collection, the nurse collects __________ data. a. Physical b. Historical c. Objective d. Subjective

C Palpation of the apical impulse is higher and more lateral, compared with the normal position, because the enlarging uterus elevates the diaphragm and displaces the heart up and to the left and rotates it on its long axis.

During the precordial assessment on an patient who is 8 months pregnant, the nurse palpates the apical impulse at the fourth left intercostal space lateral to the midclavicular line. This finding would indicate: a. Right ventricular hypertrophy. b. Increased volume and size of the heart as a result of pregnancy. c. Displacement of the heart from elevation of the diaphragm. d. Increased blood flow through the internal mammary artery.

Cushing Syndrome

Either administration of adrenocorticotropin (ACTH) or excessive production of ACTH by the pituitary stimulates the adrenal cortex to secrete excess cortisol. This causes the adrenal cortex to secrete excess cortisol. This causes Cushing Syndrome, characterized by weight gain and edema with central trunk and cervical obesity (buffalo hump) and round, plethoric face (moon face). Excessive catabolism causes muscle wasting; weakness; thin arms and legs; reduced height; and thin, fragile skin with purple abdominal striae, bruising, and acne. Note that the obesity here is markedly different from exogenous obesity caused by excessive caloric intake, in which body fat is evenly distributed and muscle strength is intact.

A According to the American Heart Association, the prevalence of hypertension is higher among Blacks than in other racial groups.

The nurse is preparing for a class on risk factors for hypertension and reviews recent statistics. Which racial group has the highest prevalence of hypertension in the world? a. Blacks b. Whites c. American Indians d. Hispanics

A patient admitted to the hospital with asthma has the following problems identified based on an admission health history and physical assessment. Which problem is a first-level priority?

Impaired gas exchange. First-level priority problems are problems that are emergent, life-threatening, and immediate. Impaired gas exchange is an emergent and immediate problem. Second-level priority problems are problems that are next in urgency; these problems require prompt intervention to forestall further deterioration. Risk for infection is an example of a second-level priority. Third-level priority problems are problems that are important to the patient's health but can be addressed after more urgent health problems are addressed. Ineffective self-health management is an example of a third-level priority.

D The lymphatic drainage of the external ear flows to the parotid, mastoid, and superficial cervical nodes. The signs are severe swelling of the canal, inflammation, and tenderness. Rhinorrhea, periorbital edema, and pain over the maxillary sinuses do not occur with otitis externa.

In an individual with otitis externa, which of these signs would the nurse expect to find on assessment? a. Rhinorrhea b. Periorbital edema c. Pain over the maxillary sinuses d. Enlarged superficial cervical nodes

A The use of history forms and note-taking may be unavoidable. However, the nurse must be aware that note-taking during the interview has disadvantages. It breaks eye contact too often and shifts the attention away from the patient, which diminishes his or her sense of importance. Note-taking may also interrupt the patients narrative flow, and it impedes the observation of the patients nonverbal behavior.

In an interview, the nurse may find it necessary to take notes to aid his or her memory later. Which statement is true regarding note-taking? a. Note-taking may impede the nurses observation of the patients nonverbal behaviors. b. Note-taking allows the patient to continue at his or her own pace as the nurse records what is said. c. Note-taking allows the nurse to shift attention away from the patient, resulting in an increased comfort level. d. Note-taking allows the nurse to break eye contact with the patient, which may increase his or her level of comfort.

B With aging, an increase in systolic blood pressure occurs. No significant change in diastolic pressure and no change in the resting heart rate occur with aging. Cardiac output at rest is does not changed with aging.

In assessing a 70-year-old man, the nurse finds the following: blood pressure 140/100 mm Hg; heart rate 104 beats per minute and slightly irregular; and the split S2 heart sound. Which of these findings can be explained by expected hemodynamic changes related to age? a. Increase in resting heart rate b. Increase in systolic blood pressure c. Decrease in diastolic blood pressure d. Increase in diastolic blood pressure

C To assess for major risk factors of coronary artery disease, the nurse should collect data regarding elevated serum cholesterol, elevated blood pressure, blood glucose levels above 100 mg/dL or known diabetes mellitus, obesity, any length of hormone replacement therapy for post menopausal women, cigarette smoking, and low activity level.

In assessing a patients major risk factors for heart disease, which would the nurse want to include when taking a history? a. Family history, hypertension, stress, and age b. Personality type, high cholesterol, diabetes, and smoking c. Smoking, hypertension, obesity, diabetes, and high cholesterol d. Alcohol consumption, obesity, diabetes, stress, and high cholesterol

B The S4 is a ventricular filling sound that occurs when the atria contract late in diastole and is heard immediately before the S1. The S4 is a very soft sound with a very low pitch. The nurse needs a good bell and must listen for this sound. An S4 is heard best at the apex, with the person in the left lateral position.

In assessing for an S4 heart sound with a stethoscope, the nurse would listen with the: a. Bell of the stethoscope at the base with the patient leaning forward. b. Bell of the stethoscope at the apex with the patient in the left lateral position. c. Diaphragm of the stethoscope in the aortic area with the patient sitting. d. Diaphragm of the stethoscope in the pulmonic area with the patient supine.

B If cardiovascular disease is suspected, then the nurse should auscultate each carotid artery for the presence of a bruit. The nurse should avoid compressing the artery, which could create an artificial bruit and compromise circulation if the carotid artery is already narrowed by atherosclerosis. Excessive pressure on the carotid sinus area high in the neck should be avoided, and excessive vagal stimulation could slow down the heart rate, especially in older adults. Palpating only one carotid artery at a time will avoid compromising arterial blood to the brain.

In assessing the carotid arteries of an older patient with cardiovascular disease, the nurse would: a. Palpate the artery in the upper one third of the neck. b. Listen with the bell of the stethoscope to assess for bruits. c. Simultaneously palpate both arteries to compare amplitude. d. Instruct the patient to take slow deep breaths during auscultation.

B The tonsils are the same color as the surrounding mucous membrane, although they look more granular and their surface shows deep crypts. Tonsillar tissue enlarges during childhood until puberty and then involutes.

In assessing the tonsils of a 30 year old, the nurse notices that they are involuted, granular in appearance, and appear to have deep crypts. What is correct response to these findings? a. Refer the patient to a throat specialist. b. No response is needed; this appearance is normal for the tonsils. c. Continue with the assessment, looking for any other abnormal findings. d. Obtain a throat culture on the patient for possible streptococcal (strep) infection.

B With the head 30 to 60 cm (1 to 2 feet) from the patients ear, the examiner exhales and slowly whispers a set of random numbers and letters, such as 5, B, 6. Normally, the patient is asked to repeat each number and letter correctly after hearing the examiner say them.

In performing a voice test to assess hearing, which of these actions would the nurse perform? a. Shield the lips so that the sound is muffled. b. Whisper a set of random numbers and letters, and then ask the patient to repeat them. c. Ask the patient to place his finger in his ear to occlude outside noise. d. Stand approximately 4 feet away to ensure that the patient can really hear at this distance.

D In addition to its place in the complete examination, eardrum assessment is mandatory for any infant or child requiring care for an illness or fever. For the infant or young child, the timing of the otoscopic examination is best toward the end of the complete examination.

In performing an examination of a 3-year-old child with a suspected ear infection, the nurse would: a. Omit the otoscopic examination if the child has a fever. b. Pull the ear up and back before inserting the speculum. c. Ask the mother to leave the room while examining the child. d. Perform the otoscopic examination at the end of the assessment.

D Childhood illnesses include measles, mumps, rubella, chickenpox, pertussis, and strep throat. Avoid recording usual childhood illnesses because an illness common in the persons childhood may be unusual today (e.g., measles).

In recording the childhood illnesses of a patient who denies having had any, which note by the nurse would be most accurate? a. Patient denies usual childhood illnesses. b. Patient states he was a very healthy child. c. Patient states his sister had measles, but he didnt. d. Patient denies measles, mumps, rubella, chickenpox, pertussis, and strep throat.

C Questions about coping and stress management include questions regarding the kinds of stresses in ones life, especially in the last year, any changes in lifestyle or any current stress, methods tried to relieve stress, and whether these methods have been helpful.

In response to a question about stress, a 39-year-old woman tells the nurse that her husband and mother both died in the past year. Which response by the nurse is most appropriate? a. This has been a difficult year for you. b. I dont know how anyone could handle that much stress in 1 year! c. What did you do to cope with the loss of both your husband and mother? d. That is a lot of stress; now lets go on to the next section of your history.

B Alcohol adversely interacts with all medications and is a factor in many social problems such as child or sexual abuse, automobile accidents, and assaults; alcohol also contributes to many illnesses and disease processes. Therefore, assessing for signs of hazardous alcohol use is important. The other options are not correct.

In response to a question regarding the use of alcohol, a patient asks the nurse why the nurse needs to know. What is the reason for needing this information? a. This information is necessary to determine the patients reliability. b. Alcohol can interact with all medications and can make some diseases worse. c. The nurse needs to be able to teach the patient about the dangers of alcohol use. d. This information is not necessary unless a drinking problem is obvious.

D In the health promotion model, the focus of the health professional is on helping the consumer choose a healthier lifestyle.

In the health promotion model, the focus of the health professional includes: a. Changing the patients perceptions of disease. b. Identifying biomedical model interventions. c. Identifying negative health acts of the consumer. d. Helping the consumer choose a healthier lifestyle.

C The red glow filling the persons pupil is the red reflex and is a normal finding caused by the reflection of the ophthalmoscope light off the inner retina. The other responses are not correct.

In using the ophthalmoscope to assess a patients eyes, the nurse notices a red glow in the patients pupils. On the basis of this finding, the nurse would: a. Suspect that an opacity is present in the lens or cornea. b. Check the light source of the ophthalmoscope to verify that it is functioning. c. Consider the red glow a normal reflection of the ophthalmoscope light off the inner retina. d. Continue with the ophthalmoscopic examination, and refer the patient for further evaluation.

Heberden and Bouchard nodes are hard and nontender and are associated with

Osteoarthritis

Techniques for examining the infant

Place the neonate or young infant flat on a padded examination table One the baby can sit without support (around 6 months) then the exam should be performed while the patient is in the caregiver's lap for as much as possible during the exam Remain in full view of the infant Should be 1 to 2 hours after feeding, when the baby is not too drowsy and not too hungry Maintain a warm environment Have the caregiver undress the baby but leave the diaper on Make sure your hands and stethoscope are warm Use a soft, crooning voice during the examination (they respond more to the feeling of tone) Lock eyes with the infant from time to time Be mindful of facial expressions and take time to play; smile at the baby Keep movements smooth and deliberate, not jerky Use a pacifier for crying or during invasive steps Offer brightly colored toys for a distraction when the infant is fussy Let the older baby touch the stethoscope or tongue blade Auscultate while the baby is sleeping Perform least distressing steps first If you elicit the Moro or "startle" reflex, do it at the end of the exam because it may cause the baby to cry

How should you record the gathered information from the snellen eye chart?

Record the result using the numeric fraction at the end of the last successful line read. Indicate whether the person missed any letters or if corrective lenses were worn. Normal acuity is 20/20. The top number indicates the distance the person is standing from the chart, and the denominator gives the distance at which a normal eye could have read that particular line.

A Side-to-side comparison is most important when auscultating the chest. The nurse should listen to at least one full respiration in each location. The other techniques are not correct.

The nurse is observing the auscultation technique of another nurse. The correct method to use when progressing from one auscultatory site on the thorax to another is _______ comparison. a. Side-to-side b. Top-to-bottom c. Posterior-to-anterior d. Interspace-by-interspace

B When documenting the force, or amplitude, of pulses, 3+ indicates an increased, full, or bounding pulse, 2+ indicates a normal pulse, 1+ indicates a weak pulse, and 0 indicates an absent pulse.

The nurse is reviewing an assessment of a patients peripheral pulses and notices that the documentation states that the radial pulses are 2+. The nurse recognizes that this reading indicates what type of pulse? a. Bounding b. Normal c. Weak d. Absent

B Returning blood from the body empties into the right atrium and flows into the right ventricle and then goes to the lungs through the pulmonary artery. The lungs oxygenate the blood, and it is then returned to the left atrium through the pulmonary vein. The blood goes from there to the left ventricle and then out to the body through the aorta.

The direction of blood flow through the heart is best described by which of these? a. Vena cava right atrium right ventricle lungs pulmonary artery left atrium left ventricle b. Right atrium right ventricle pulmonary artery lungs pulmonary vein left atrium left ventricle c. Aorta right atrium right ventricle lungs pulmonary vein left atrium left ventricle vena cava d. Right atrium right ventricle pulmonary vein lungs pulmonary artery left atrium left ventricle

B A normal response for the diagnostic positions test is parallel tracking of the object with both eyes. Eye movement that is not parallel indicates a weakness of an extraocular muscle or dysfunction of the CN that innervates it.

The nurse is performing the diagnostic positions test. Normal findings would be which of these results? a. Convergence of the eyes b. Parallel movement of both eyes c. Nystagmus in extreme superior gaze d. Slight amount of lid lag when moving the eyes from a superior to an inferior position

D Specialized cells in the SA node near the superior vena cava initiate an electrical impulse. The current flows in an orderly sequence, first across the atria to the AV node low in the atrial septum. There it is delayed slightly, allowing the atria the time to contract before the ventricles are stimulated. Then the impulse travels to the bundle of His, the right and left bundle branches, and then through the ventricles.

The electrical stimulus of the cardiac cycle follows which sequence? a. AV node SA node bundle of His b. Bundle of His AV node SA node c. SA node AV node bundle of His bundle branches d. AV node SA node bundle of His bundle branches

D Because no cardiac valve exists to separate the superior vena cava from the right atrium, the jugular veins give information about the activity on the right side of the heart. They reflect filling pressures and volume changes. Normal jugular venous pulsation is 2 cm or less above the sternal angle. Elevated pressure is more than 3 cm above the sternal angle at 45 degrees and occurs with heart failure.

The findings from an assessment of a 70-year-old patient with swelling in his ankles include jugular venous pulsations 5 cm above the sternal angle when the head of his bed is elevated 45 degrees. The nurse knows that this finding indicates: a. Decreased fluid volume. b. Increased cardiac output. c. Narrowing of jugular veins. d. Elevated pressure related to heart failure.

A The cause of these findings is tetralogy of Fallot. Its subjective findings include: (1) severe cyanosis, not in the first months of life but developing as the infant grows, and right ventricle outflow (i.e., pulmonic) stenosis that gets worse; (2) cyanosis with crying and exertion at first and then at rest; and (3) slowed development. Its objective findings include: (1) thrill palpable at the left lower sternal border; (2) the S1 is normal, the S2 has a loud A2, and the P2 is diminished or absent; and (3) the murmur is systolic, loud, and crescendo-decrescendo.

The mother of a 10-month-old infant tells the nurse that she has noticed that her son becomes blue when he is crying and that the frequency of this is increasing. He is also not crawling yet. During the examination the nurse palpates a thrill at the left lower sternal border and auscultates a loud systolic murmur in the same area. What would be the most likely cause of these findings? a. Tetralogy of Fallot b. Atrial septal defect c. Patent ductus arteriosus d. Ventricular septal defect

D With a very young child, the parent is asked, How do you know the child is in pain? A young child pulling at his or her ears should alert parents to the childs ear pain. Statements about teething and questioning whether the child is really having pain do not explore the symptoms, which should be done before a physical examination.

The mother of a 16-month-old toddler tells the nurse that her daughter has an earache. What would be an appropriate response? a. Maybe she is just teething. b. I will check her ear for an ear infection. c. Are you sure she is really having pain? d. Describe what she is doing to indicate she is having pain.

D The infants eustachian tube is relatively shorter and wider than the adults eustachian tube, and its position is more horizontal; consequently, pathogens from the nasopharynx can more easily migrate through to the middle ear. The other responses are not appropriate.

The mother of a 2-year-old is concerned because her son has had three ear infections in the past year. What would be an appropriate response by the nurse? a. It is unusual for a small child to have frequent ear infections unless something else is wrong. b. We need to check the immune system of your son to determine why he is having so many ear infections. c. Ear infections are not uncommon in infants and toddlers because they tend to have more cerumen in the external ear. d. Your sons eustachian tube is shorter and wider than yours because of his age, which allows for infections to develop more easily.

D Polyethylene tubes are surgically inserted into the eardrum to relieve middle ear pressure and to promote drainage of chronic or recurrent middle ear infections. Tubes spontaneously extrude in 6 months to 1 year.

The mother of a 2-year-old toddler is concerned about the upcoming placement of tympanostomy tubes in her sons ears. The nurse would include which of these statements in the teaching plan? a. The tubes are placed in the inner ear. b. The tubes are used in children with sensorineural loss. c. The tubes are permanently inserted during a surgical procedure. d. The purpose of the tubes is to decrease the pressure and allow for drainage.

A The normal pathway of hearing is air conduction, which starts when sound waves produce vibrations on the tympanic membrane. Conductive hearing loss results from a mechanical dysfunction of the external or middle ear. The other statements are not true concerning air conduction.

The nurse is assessing a patient who may have hearing loss. Which of these statements is true concerning air conduction? a. Air conduction is the normal pathway for hearing. b. Vibrations of the bones in the skull cause air conduction. c. Amplitude of sound determines the pitch that is heard. d. Loss of air conduction is called a conductive hearing loss.

D To screen for heart disease in an infant, the focus should be on feeding. Fatigue during feeding should be noted. An infant with heart failure takes fewer ounces each feeding, becomes dyspneic with sucking, may be diaphoretic, and then falls into exhausted sleep and awakens after a short time hungry again.

The mother of a 3-month-old infant states that her baby has not been gaining weight. With further questioning, the nurse finds that the infant falls asleep after nursing and wakes up after a short time, hungry again. What other information would the nurse want to have? a. Infants sleeping position b. Sibling history of eating disorders c. Amount of background noise when eating d. Presence of dyspnea or diaphoresis when sucking

D When gathering a complete history, the nurse should give the reason for the interview during the opening or introduction phase of the interview, not during or at the end of the interview.

The nurse asks, I would like to ask you some questions about your health and your usual daily activities so that we can better plan your stay here. This question is found at the __________ phase of the interview process. a. Summary b. Closing c. Body d. Opening or introduction

C Evaluation is the next step after the implementation phase of the nursing process. During this step, the nurse evaluates the individuals condition and compares the actual outcomes with expected outcomes

The nurse has implemented several planned interventions to address the nursing diagnosis of acute pain. Which would be the next appropriate action? a. Establish priorities. b. Identify expected outcomes. c. Evaluate the individuals condition, and compare actual outcomes with expected outcomes. d. Interpret data, and then identify clusters of cues and make inferences.

B Interpretation is not based on direct observation as is confrontation, but it is based on ones inference or conclusion. The nurse risks making the wrong inference. If this is the case, then the patient will correct it. However, even if the inference is correct, interpretation helps prompt further discussion of the topic.

The nurse has used interpretation regarding a patients statement or actions. After using this technique, it would be best for the nurse to: a. Apologize, because using interpretation can be demeaning for the patient. b. Allow time for the patient to confirm or correct the inference. c. Continue with the interview as though nothing has happened. d. Immediately restate the nurses conclusion on the basis of the patients nonverbal response.

A A symptom is a subjective sensation (e.g., chest pain) that a person feels from a disorder. A sign is an objective abnormality that the examiner can detect on physical examination or in laboratory reports, as illustrated by the other responses.

The nurse is asking a patient for his reason for seeking care and asks about the signs and symptoms he is experiencing. Which of these is an example of a symptom? a. Chest pain b. Clammy skin c. Serum potassium level at 4.2 mEq/L d. Body temperature of 100 F

B Frank blood or clear watery drainage (cerebrospinal leak) after a trauma suggests a basal skull fracture and warrants immediate referral. Purulent drainage indicates otitis externa or otitis media.

The nurse is assessing a 16-year-old patient who has suffered head injuries from a recent motor vehicle accident. Which of these statements indicates the most important reason for assessing for any drainage from the ear canal? a. If the drum has ruptured, then purulent drainage will result. b. Bloody or clear watery drainage can indicate a basal skull fracture. c. The auditory canal many be occluded from increased cerumen. d. Foreign bodies from the accident may cause occlusion of the canal.

C Children are prone to put an object up the nose, producing unilateral purulent drainage with a foul odor. Because some risk for aspiration exists, removal should be prompt.

The nurse is assessing a 3 year old for drainage from the nose. On assessment, a purulent drainage that has a very foul odor is noted from the left naris and no drainage is observed from the right naris. The child is afebrile with no other symptoms. What should the nurse do next? a. Refer to the physician for an antibiotic order. b. Have the mother bring the child back in 1 week. c. Perform an otoscopic examination of the left nares. d. Tell the mother that this drainage is normal for a child of this age.

B Biographic data, such as when the person entered the United States and from what country, are appropriate additions to the health history. The other answers do not reflect appropriate questions.

The nurse is assessing a new patient who has recently immigrated to the United States. Which question is appropriate to add to the health history? a. Why did you come to the United States? b. When did you come to the United States and from what country? c. What made you leave your native country? d. Are you planning to return to your home?

C A black, hairy tongue is not really hair but the elongation of filiform papillae and painless overgrowth of mycelial threads of fungus infection on the tongue. It occurs after the use of antibiotics, which inhibit normal bacteria and allow a proliferation of fungus.

The nurse is assessing a patient in the hospital who has received numerous antibiotics and notices that his tongue appears to be black and hairy. In response to his concern, what would the nurse say? a. We will need to get a biopsy to determine the cause. b. This is an overgrowth of hair and will go away in a few days. c. Black, hairy tongue is a fungal infection caused by all the antibiotics you have received. d. This is probably caused by the same bacteria you had in your lungs.

A Oral Kaposis sarcoma is a bruiselike, dark red or violet, confluent macule that usually occurs on the hard palate. It may appear on the soft palate or gingival margin. Oral lesions may be among the earliest lesions to develop with AIDS.

The nurse is assessing a patient with a history of intravenous drug abuse. In assessing his mouth, the nurse notices a dark red confluent macule on the hard palate. This could be an early sign of: a. Acquired immunodeficiency syndrome (AIDS). b. Measles. c. Leukemia. d. Carcinoma.

B When performing hepatojugular reflux, the jugular veins will rise for a few seconds and then recede back to the previous level if the heart is able to pump the additional volume created by the pushing. However, with heart failure, the jugular veins remain elevated as long as pressure on the abdomen is maintained.

The nurse is assessing a patient with possible cardiomyopathy and assesses the hepatojugular reflux. If heart failure is present, then the nurse should recognize which finding while pushing on the right upper quadrant of the patients abdomen, just below the rib cage? a. The jugular veins will rise for a few seconds and then recede back to the previous level if the heart is properly working. b. The jugular veins will remain elevated as long as pressure on the abdomen is maintained. c. An impulse will be visible at the fourth or fifth intercostal space at or inside the midclavicular line. d. The jugular veins will not be detected during this maneuver.

D The accommodation reaction includes pupillary constriction and convergence of the axes of the eyes. The other responses are not correct.

The nurse is assessing a patients eyes for the accommodation response and would expect to see which normal finding? a. Dilation of the pupils b. Consensual light reflex c. Conjugate movement of the eyes d. Convergence of the axes of the eyes

A, C, D, E The mnemonic PQRSTU may help the nurse remember to address the critical characteristics that need to be assessed: (1) P: provocative or palliative; (2) Q: quality or quantity; (3) R: region or radiation; (4) S: severity scale; (5) T: timing; and (6) U: understand the patients perception. Asking, Where is the pain? reflects region. Asking the patient to rate the pain on a 1 to 10 scale reflects severity. Asking How often reflects timing. Asking what makes the pain better reflects provocative. The other options reflect health history and family history.

The nurse is assessing a patients headache pain. Which questions reflect one or more of the critical characteristics of symptoms that should be assessed? Select all that apply. a. Where is the headache pain? b. Did you have these headaches as a child? c. On a scale of 1 to 10, how bad is the pain? d. How often do the headaches occur? e. What makes the headaches feel better? f. Do you have any family history of headaches?

C The sense of smell may be reduced because of a decrease in the number of olfactory nerve fibers. Nasal hairs grow coarser and stiffer with aging. The gums may recede with aging, not hypertrophy, and saliva production decreases.

The nurse is assessing an 80-year-old patient. Which of these findings would be expected for this patient? a. Hypertrophy of the gums b. Increased production of saliva c. Decreased ability to identify odors d. Finer and less prominent nasal hair

Test boys only once for color vision between the ages of 4 and 8 years. Color vision is not tested in girls because it is rare in girls. Testing is performed with the Ishihara test, which is a series of polychromatic cards.

The nurse is assessing color vision of a male child. Which statement is correct? The nurse should: a. Check color vision annually until the age of 18 years. b. Ask the child to identify the color of his or her clothing. c. Test for color vision once between the ages of 4 and 8 years. d. Begin color vision screening at the childs 2-year checkup.

A The heart rate may range from 100 to 180 beats per minute immediately after birth and then stabilize to an average of 120 to 140 beats per minute. Infants normally have wide fluctuations with activity, from 170 beats per minute or more with crying or being active to 70 to 90 beats per minute with sleeping. Persistent tachycardia is greater than 200 beats per minute in newborns or greater than 150 beats per minute in infants.

The nurse is assessing the apical pulse of a 3-month-old infant and finds that the heart rate is 135 beats per minute. The nurse interprets this result as: a. Normal for this age. b. Lower than expected. c. Higher than expected, probably as a result of crying. d. Higher than expected, reflecting persistent tachycardia.

D In the aging adult, the lungs are less elastic and distensible, which decreases their ability to collapse and recoil. Vital capacity is decreased, and a loss of intra-alveolar septa occurs, causing less surface area for gas exchange. The lung bases become less ventilated, and the older person is at risk for dyspnea with exertion beyond his or her usual workload.

The nurse is assessing the lungs of an older adult. Which of these changes are normal in the respiratory system of the older adult? a. Severe dyspnea is experienced on exertion, resulting from changes in the lungs. b. Respiratory muscle strength increases to compensate for a decreased vital capacity. c. Decrease in small airway closure occurs, leading to problems with atelectasis. d. Lungs are less elastic and distensible, which decreases their ability to collapse and recoil.

C A full, bounding pulse occurs with hyperkinetic states (e.g., exercise, anxiety, fever), anemia, and hyperthyroidism. An absent pulse occurs with occlusion. Weak, thready pulses occur with shock and peripheral artery disease.

The nurse is assessing the pulses of a patient who has been admitted for untreated hyperthyroidism. The nurse should expect to find a(n) _______ pulse. a. Normal b. Absent c. Bounding d. Weak, thready

A, C, D As a patient repeatedly says ninety-nine, normally the examiner hears voice sounds but cannot distinguish what is being said. If a clear ninety-nine is auscultated, then it could indicate increased lung density, which enhances the transmission of voice sounds, which is a measure of bronchophony. When a patient says a long ee-ee-ee sound, normally the examiner also hears a long ee-ee-ee sound through auscultation, which is a measure of egophony. If the examiner hears a long aaaaaa sound instead, this sound could indicate areas of consolidation or compression. With whispered pectoriloquy, as when a patient whispers a phrase such as one-two-three, the normal response when auscultating voice sounds is to hear sounds that are faint, muffled, and almost inaudible. If the examiner clearly hears the whispered voice, as if the patient is speaking through the stethoscope, then consolidation of the lung fields may exist

The nurse is assessing voice sounds during a respiratory assessment. Which of these findings indicates a normal assessment? Select all that apply. a. Voice sounds are faint, muffled, and almost inaudible when the patient whispers one, two, three in a very soft voice. b. As the patient repeatedly says ninety-nine, the examiner clearly hears the words ninety-nine. c. When the patient speaks in a normal voice, the examiner can hear a sound but cannot exactly distinguish what is being said. d. As the patient says a long ee-ee-ee sound, the examiner also hears a long ee-ee-ee sound. e. As the patient says a long ee-ee-ee sound, the examiner hears a long aaaaaa sound.

B To help expose the femoral area, particularly in obese people, the nurse should ask the person to bend his or her knees to the side in a froglike position.

The nurse is attempting to assess the femoral pulse in a patient who is obese. Which of these actions would be most appropriate? a. The patient is asked to assume a prone position. b. The patient is asked to bend his or her knees to the side in a froglike position. c. The nurse firmly presses against the bone with the patient in a semi-Fowler position. d. The nurse listens with a stethoscope for pulsations; palpating the pulse in an obese person is extremely difficult.

C Firmly holding the diaphragm of the stethoscope against the chest is the correct way to auscultate breath sounds. The patient should be instructed to breathe through his or her mouth, a little deeper than usual, but not to hyperventilate.

The nurse is auscultating the chest in an adult. Which technique is correct? a. Instructing the patient to take deep, rapid breaths b. Instructing the patient to breathe in and out through his or her nose c. Firmly holding the diaphragm of the stethoscope against the chest d. Lightly holding the bell of the stethoscope against the chest to avoid friction

A One type of adventitious sound, atelectatic crackles, does not have a pathologic cause. They are short, popping, crackling sounds that sound similar to fine crackles but do not last beyond a few breaths. When sections of alveoli are not fully aerated (as in people who are asleep or in older adults), they deflate slightly and accumulate secretions. Crackles are heard when these sections are expanded by a few deep breaths. Atelectatic crackles are heard only in the periphery, usually in dependent portions of the lungs, and disappear after the first few breaths or after a cough.

The nurse is auscultating the lungs of a patient who had been sleeping and notices short, popping, crackling sounds that stop after a few breaths. The nurse recognizes that these breath sounds are: a. Atelectatic crackles that do not have a pathologic cause. b. Fine crackles and may be a sign of pneumonia. c. Vesicular breath sounds. d. Fine wheezes.

D Exposure to passive and gestational smoke is a risk factor for ear infections in infants and children.

The nurse is conducting a child safety class for new mothers. Which factor places young children at risk for ear infections? a. Family history b. Air conditioning c. Excessive cerumen d. Passive cigarette smoke

B Novice nurses operate from a set of defined, structured rules. The expert practitioner uses intuitive links.

The nurse is conducting a class for new graduate nurses. During the teaching session, the nurse should keep in mind that novice nurses, without a background of skills and experience from which to draw, are more likely to make their decisions using: a. Intuition. b. A set of rules. c. Articles in journals. d. Advice from supervisors.

D First-level priority problems are those that are emergent, life threatening, and immediate (e.g., establishing an airway, supporting breathing, maintaining circulation, monitoring abnormal vital signs)

The nurse is conducting a class on priority setting for a group of new graduate nurses. Which is an example of a first-level priority problem? a. Patient with postoperative pain b. Newly diagnosed patient with diabetes who needs diabetic teaching c. Individual with a small laceration on the sole of the foot d. Individual with shortness of breath and respiratory distress

B, C, E Questions about tooth loss, ability to tell time, and ability to tie shoelaces are appropriate questions for a developmental assessment. Questions about junk food intake and vitamins are part of a nutritional history. Questions about food allergies are not part of a developmental history.

The nurse is conducting a developmental history on a 5-year-old child. Which questions are appropriate to ask the parents for this part of the assessment? Select all that apply. a. How much junk food does your child eat? b. How many teeth has he lost, and when did he lose them? c. Is he able to tie his shoelaces? d. Does he take a childrens vitamin? e. Can he tell time? f. Does he have any food allergies?

B The image formed on the retina is upside down and reversed from its actual appearance in the outside world. The light rays are refracted through the transparent media of the eye before striking the retina, and the nerve impulses are conducted through the optic nerve tract to the visual cortex of the occipital lobe of the brain. The left side of the brain interprets vision for the right eye.

The nurse is conducting a visual examination. Which of these statements regarding visual pathways and visual fields is true? a. The right side of the brain interprets the vision for the right eye. b. The image formed on the retina is upside down and reversed from its actual appearance in the outside world. c. Light rays are refracted through the transparent media of the eye before striking the pupil. d. Light impulses are conducted through the optic nerve to the temporal lobes of the brain.

A, D, E The use of a computer can become a barrier. The nurse should begin the interview as usual by greeting the patient, establishing rapport, and collecting the patients narrative story in a direct, face-to-face manner. Only after the narrative is fully explored should the nurse type data into the computer. When typing, the nurse should position the monitor so that the patient can see it.

The nurse is conducting an interview in an outpatient clinic and is using a computer to record data. Which are the best uses of the computer in this situation? Select all that apply. a. Collect the patients data in a direct, face-to-face manner. b. Enter all the data as the patient states them. c. Ask the patient to wait as the nurse enters the data. d. Type the data into the computer after the narrative is fully explored. e. Allow the patient to see the monitor during typing.

B Communication is all behaviors, conscious and unconscious, verbal and nonverbal. All behaviors have meaning. Her behavior does not imply that she is nervous about labor, upset by her husband, or worried about the nurses response.

The nurse is conducting an interview with a woman who has recently learned that she is pregnant and who has come to the clinic today to begin prenatal care. The woman states that she and her husband are excited about the pregnancy but have a few questions. She looks nervously at her hands during the interview and sighs loudly. Considering the concept of communication, which statement does the nurse know to be most accurate? The woman is: a. Excited about her pregnancy but nervous about the labor. b. Exhibiting verbal and nonverbal behaviors that do not match. c. Excited about her pregnancy, but her husband is not and this is upsetting to her. d. Not excited about her pregnancy but believes the nurse will negatively respond to her if she states this.

B, C, F Open-ended questions allow for self-expression, build and enhance rapport, and obtain narrative information. These features enhance communication during an interview. The other statements are appropriate for closed or direct questions.

The nurse is conducting an interview. Which of these statements is true regarding open-ended questions? Select all that apply. a. Open-ended questions elicit cold facts. b. They allow for self-expression. c. Open-ended questions build and enhance rapport. d. They leave interactions neutral. e. Open-ended questions call for short one- to two-word answers. f. They are used when narrative information is needed.

C A weak, thready pulse is hard to palpate, may fade in and out, and is easily obliterated by pressure. It is associated with decreased cardiac output and peripheral arterial disease.

The nurse is describing a weak, thready pulse on the documentation flow sheet. Which statement is correct? a. Is easily palpable; pounds under the fingertips. b. Has greater than normal force, then suddenly collapses. c. Is hard to palpate, may fade in and out, and is easily obliterated by pressure. d. Rhythm is regular, but force varies with alternating beats of large and small amplitude.

The nurse is describing how to perform a testicular self-examination to a patient. Which statement is most appropriate? a. A good time to examine your testicles is just before you take a shower. b. If you notice an enlarged testicle or a painless lump, call your health care provider. c. The testicle is egg shaped and movable. It feels firm and has a lumpy consistency. d. Perform a testicular examination at least once a week to detect the early stages of testicular cancer.

The nurse is describing how to perform a testicular self-examination to a patient. Which statement is most appropriate? a. A good time to examine your testicles is just before you take a shower. b. If you notice an enlarged testicle or a painless lump, call your health care provider. c. The testicle is egg shaped and movable. It feels firm and has a lumpy consistency. d. Perform a testicular examination at least once a week to detect the early stages of testicular cancer.

C Numerous comparison studies have shown that the percussed cardiac border correlates only moderately with the true cardiac border. Percussion is of limited usefulness with the female breast tissue, in a person who is obese, or in a person with a muscular chest wall. Chest x-ray images or echocardiographic examinations are significantly more accurate in detecting heart enlargement.

The nurse is examining a patient who has possible cardiac enlargement. Which statement about percussion of the heart is true? a. Percussion is a useful tool for outlining the hearts borders. b. Percussion is easier in patients who are obese. c. Studies show that percussed cardiac borders do not correlate well with the true cardiac border. d. Only expert health care providers should attempt percussion of the heart.

C The ear is lined with glands that secrete cerumen, which is a yellow waxy material that lubricates and protects the ear.

The nurse is examining a patients ears and notices cerumen in the external canal. Which of these statements about cerumen is correct? a. Sticky honey-colored cerumen is a sign of infection. b. The presence of cerumen is indicative of poor hygiene. c. The purpose of cerumen is to protect and lubricate the ear. d. Cerumen is necessary for transmitting sound through the auditory canal.

A The optic disc is located on the nasal side of the retina. Its color is a creamy yellow-orange to a pink, and the edges are distinct and sharply demarcated, not blurred. A pigmented crescent is black and is due to the accumulation of pigment in the choroid.

The nurse is examining a patients retina with an ophthalmoscope. Which finding is considered normal? a. Optic disc that is a yellow-orange color b. Optic disc margins that are blurred around the edges c. Presence of pigmented crescents in the macular area d. Presence of the macula located on the nasal side of the retina

B Lymph nodes are relatively large in children, and the superficial ones are often palpable even when the child is healthy.

The nurse is examining the lymphatic system of a healthy 3-year-old child. Which finding should the nurse expect? a. Excessive swelling of the lymph nodes b. Presence of palpable lymph nodes c. No palpable nodes because of the immature immune system of a child d. Fewer numbers and a smaller size of lymph nodes compared with those of an adult

B The community is assessed when the nurse asks whether a person is part of a religious or spiritual community or congregation. The other areas assessed are faith, influence, and addressing any religious or spiritual issues or concerns.

The nurse is incorporating a persons spiritual values into the health history. Which of these questions illustrates the community portion of the FICA (faith and belief, importance and influence, community, and addressing or applying in care) questions? a. Do you believe in God? b. Are you a part of any religious or spiritual congregation? c. Do you consider yourself to be a religious or spiritual person? d. How does your religious faith influence the way you think about your health?

A The nurse should ask the deaf person the preferred way to communicateby signing, lip reading, or writing. If the person prefers lip reading, then the nurse should be sure to face him squarely and have good lighting on the nurses face. The nurse should not exaggerate lip movements because this distorts words. Similarly, shouting distorts the reception of a hearing aid the person may wear. The nurse should speak slowly and supplement his or her voice with appropriate hand gestures or pantomime.

The nurse is interviewing a male patient who has a hearing impairment. What techniques would be most beneficial in communicating with this patient? a. Determine the communication method he prefers. b. Avoid using facial and hand gestures because most hearing-impaired people find this degrading. c. Request a sign language interpreter before meeting with him to help facilitate the communication. d. Speak loudly and with exaggerated facial movement when talking with him because doing so will help him lip read.

A Wheezes are caused by air squeezed or compressed through passageways narrowed almost to closure by collapsing, swelling, secretions, or tumors, such as with acute asthma or chronic emphysema.

The nurse is listening to the breath sounds of a patient with severe asthma. Air passing through narrowed bronchioles would produce which of these adventitious sounds? a. Wheezes b. Bronchial sounds c. Bronchophony d. Whispered pectoriloquy

B This question offers the person a final opportunity for self-expression. No new topic should be introduced. The other questions are not appropriate.

The nurse is nearing the end of an interview. Which statement is appropriate at this time? a. Did we forget something? b. Is there anything else you would like to mention? c. I need to go on to the next patient. Ill be back. d. While Im here, lets talk about your upcoming surgery.

D In the infant, salivation starts at 3 months. The baby will drool for a few months before learning to swallow the saliva. This drooling does not herald the eruption of the first tooth, although many parents think it does.

The nurse is obtaining a health history on a 3-month-old infant. During the interview, the mother states, I think she is getting her first tooth because she has started drooling a lot. The nurses best response would be: a. Youre right, drooling is usually a sign of the first tooth. b. It would be unusual for a 3 month old to be getting her first tooth. c. This could be the sign of a problem with the salivary glands. d. She is just starting to salivate and hasnt learned to swallow the saliva.

D It is important for the nurse to recognize positive health measures, such as what the person has been doing to help him or herself stay well and to live to an older age. The other responses are not pertinent to a patient of this age.

The nurse is obtaining a health history on an 87-year-old woman. Which of the following areas of questioning would be most useful at this time? a. Obstetric history b. Childhood illnesses c. General health for the past 20 years d. Current health promotion activities

A Health promotion for a man would include the performance of testicular self-examinations. The other questions are asking about possible disease or illness issues.

The nurse is obtaining a history from a 30-year-old male patient and is concerned about health promotion activities. Which of these questions would be appropriate to use to assess health promotion activities for this patient? a. Do you perform testicular self-examinations? b. Have you ever noticed any pain in your testicles? c. Have you had any problems with passing urine? d. Do you have any history of sexually transmitted diseases?

B The person should feel firm pressure but no pain. Sinus areas are tender to palpation in persons with chronic allergies or an acute infection (sinusitis).

The nurse is palpating the sinus areas. If the findings are normal, then the patient should report which sensation? a. No sensation b. Firm pressure c. Pain during palpation d. Pain sensation behind eyes

A A dull percussion note signals an abnormal density in the lungs, as with pneumonia, pleural effusion, atelectasis, or a tumor.

The nurse is percussing over the lungs of a patient with pneumonia. The nurse knows that percussion over an area of atelectasis in the lungs will reveal: a. Dullness. b. Tympany. c. Resonance. d. Hyperresonance.

C Inflammation of the precordium gives rise to a friction rub. The sound is high pitched and scratchy, similar to sandpaper being rubbed. A friction rub is best heard with the diaphragm of the stethoscope, with the person sitting up and leaning forward, and with the breath held in expiration. A friction rub can be heard any place on the precordium. Usually, however, the sound is best heard at the apex and left lower sternal border, which are places where the pericardium comes in close contact with the chest wall.

The nurse is performing a cardiac assessment on a 65-year-old patient 3 days after her myocardial infarction (MI). Heart sounds are normal when she is supine, but when she is sitting and leaning forward, the nurse hears a high-pitched, scratchy sound with the diaphragm of the stethoscope at the apex. It disappears on inspiration. The nurse suspects: a. Increased cardiac output. b. Another MI. c. Inflammation of the precordium. d. Ventricular hypertrophy resulting from muscle damage.

B Functional assessment measures how a person manages day-to-day activities. For the older person, the meaning of health becomes those activities that they can or cannot do. The other responses do not relate to functional assessment.

The nurse is performing a functional assessment on an 82-year-old patient who recently had a stroke. Which of these questions would be most important to ask? a. Do you wear glasses? b. Are you able to dress yourself? c. Do you have any thyroid problems? d. How many times a day do you have a bowel movement?

A In a situation during which a language barrier exists and no interpreter is available, simple words should be used, avoiding medical jargon. The use of contractions and pronouns should also be avoided. Nouns should be repeatedly used, and one topic at a time should be discussed.

The nurse is performing a health interview on a patient who has a language barrier, and no interpreter is available. Which is the best example of an appropriate question for the nurse to ask in this situation? a. Do you take medicine? b. Do you sterilize the bottles? c. Do you have nausea and vomiting? d. You have been taking your medicine, havent you?

: B Dense white patches on the tympanic membrane are sequelae of repeated ear infections. They do not necessarily affect hearing.

The nurse is performing a middle ear assessment on a 15-year-old patient who has had a history of chronic ear infections. When examining the right tympanic membrane, the nurse sees the presence of dense white patches. The tympanic membrane is otherwise unremarkable. It is pearly, with the light reflex at 5 oclock and landmarks visible. The nurse should: a. Refer the patient for the possibility of a fungal infection. b. Know that these are scars caused from frequent ear infections. c. Consider that these findings may represent the presence of blood in the middle ear. d. Be concerned about the ability to hear because of this abnormality on the tympanic membrane.

D Objective data are the patients record, laboratory studies, and condition that the health professional observes by inspecting, percussing, palpating, and auscultating during the physical examination. The other responses reflect subjective data.

The nurse is performing a physical assessment on a newly admitted patient. An example of objective information obtained during the physical assessment includes the: a. Patients history of allergies. b. Patients use of medications at home. c. Last menstrual period 1 month ago. d. 2 5 cm scar on the right lower forearm.

C The health history includes the same format as that described for the younger adult, as well as some additional questions. These additional questions address ways in which the activities of daily living may have been affected by the normal aging processes or by the effects of chronic illness or disability.

The nurse is performing a review of systems on a 76-year-old patient. Which of these statements is correct for this situation? a. The questions asked are identical for all ages. b. The interviewer will start incorporating different questions for patients 70 years of age and older. c. Questions that are reflective of the normal effects of aging are added. d. At this age, a review of systems is not necessarythe focus should be on current problems.

C Palpable lymph nodes are often normal in children and infants. They are small, firm, shotty, mobile, and nontender. Vaccinations can produce lymphadenopathy. Enlarged, warm, and tender nodes indicate a current infection.

The nurse is performing a well-child checkup on a 5-year-old boy. He has no current condition that would lead the nurse to suspect an illness. His health history is unremarkable, and he received immunizations 1 week ago. Which of these findings should be considered normal in this patient? a. Enlarged, warm, and tender nodes b. Lymphadenopathy of the cervical nodes c. Palpable firm, small, shotty, mobile, and nontender lymph nodes d. Firm, rubbery, and large nodes, somewhat fixed to the underlying tissue

A With chronic allergies, the mucosa looks swollen, boggy, pale, and gray. Elevated body temperature, colds, and nosebleeds do not cause these mucosal changes.

The nurse is performing an assessment on a 21-year-old patient and notices that his nasal mucosa appears pale, gray, and swollen. What would be the most appropriate question to ask the patient? a. Are you aware of having any allergies? b. Do you have an elevated temperature? c. Have you had any symptoms of a cold? d. Have you been having frequent nosebleeds?

C An ulcerated crusted nodule with an indurated base that fails to heal is characteristic of a carcinoma. These lesions fail to heal and intermittently bleed. Individuals with such symptoms should be referred for a biopsy (see Table 15-2). The other responses are not correct.

The nurse is performing an assessment on a 65-year-old man. He reports a crusty nodule behind the pinna. It intermittently bleeds and has not healed over the past 6 months. On physical assessment, the nurse finds an ulcerated crusted nodule with an indurated base. The preliminary analysis in this situation is that this: a. Is most likely a benign sebaceous cyst. b. Is most likely a keloid. c. Could be a potential carcinoma, and the patient should be referred for a biopsy. d. Is a tophus, which is common in the older adult and is a sign of gout.

C Normal capillary refill time is less than 1 to 2 seconds. The following conditions can skew the findings: a cool room, decreased body temperature, cigarette smoking, peripheral edema, and anemia.

The nurse is performing an assessment on an adult. The adults vital signs are normal, and capillary refill time is 5 seconds. What should the nurse do next? a. Ask the patient about a history of frostbite. b. Suspect that the patient has venous insufficiency. c. Consider this a delayed capillary refill time, and investigate further. d. Consider this a normal capillary refill time that requires no further assessment.

D Ulceration on the side or base of the tongue or under the tongue raises the suspicion of cancer and must be investigated. The risk of early metastasis is present because of rich lymphatic drainage. The vesicle may be an aphthous ulcer, which is painful but not dangerous. The other responses are normal findings.

The nurse is performing an assessment. Which of these findings would cause the greatest concern? a. Painful vesicle inside the cheek for 2 days b. Presence of moist, nontender Stensens ducts c. Stippled gingival margins that snugly adhere to the teeth d. Ulceration on the side of the tongue with rolled edges

A A high-tone frequency hearing loss is apparent for those affected with presbycusis, the hearing loss that occurs with aging. The pinna loses elasticity, causing earlobes to be pendulous. The eardrum may be whiter in color and more opaque and duller in the older person than in the younger adult.

The nurse is performing an ear examination of an 80-year-old patient. Which of these findings would be considered normal? a. High-tone frequency loss b. Increased elasticity of the pinna c. Thin, translucent membrane d. Shiny, pink tympanic membrane

A The cornea and the sclera make up the outer layer of the eye. The cornea is very sensitive to touch. The middle layer, the choroid, has dark pigmentation to prevent light from reflecting internally. The trigeminal nerve (CN V) and the facial nerve (CN VII) are stimulated when the outer surface of the eye is stimulated. The retina, in the inner layer of the eye, is where light waves are changed into nerve impulses.

The nurse is performing an external eye examination. Which statement regarding the outer layer of the eye is true? a. The outer layer of the eye is very sensitive to touch. b. The outer layer of the eye is darkly pigmented to prevent light from reflecting internally. c. The trigeminal nerve (CN V) and the trochlear nerve (CN IV) are stimulated when the outer surface of the eye is stimulated. d. The visual receptive layer of the eye in which light waves are changed into nerve impulses is located in the outer layer of the eye.

B Pupils are small in the older adult, and the pupillary light reflex may be slowed, but pupillary constriction should be symmetric. The assessment findings in the other responses are considered normal in older persons.

The nurse is performing an eye assessment on an 80-year-old patient. Which of these findings is considered abnormal? a. Decrease in tear production b. Unequal pupillary constriction in response to light c. Presence of arcus senilis observed around the cornea d. Loss of the outer hair on the eyebrows attributable to a decrease in hair follicles

D Testing for strabismus is done by performing the corneal light reflex test and the cover test. The Snellen eye chart and confrontation test are not used to test for strabismus.

The nurse is performing an eye-screening clinic at a daycare center. When examining a 2-year-old child, the nurse suspects that the child has a lazy eye and should: a. Examine the external structures of the eye. b. Assess visual acuity with the Snellen eye chart. c. Assess the childs visual fields with the confrontation test. d. Test for strabismus by performing the corneal light reflex test.

A Leukoedema, a grayish-white benign lesion occurring on the buccal mucosa, is most often observed in Blacks.

The nurse is performing an oral assessment on a 40-year-old Black patient and notices the presence of a 1 cm, nontender, grayish-white lesion on the left buccal mucosa. Which one of these statements is true? This lesion is: a. Leukoedema and is common in dark-pigmented persons. b. The result of hyperpigmentation and is normal. c. Torus palatinus and would normally be found only in smokers. d. Indicative of cancer and should be immediately tested.

C The pinna is pulled up and back on an adult or older child, which helps straighten the S-shape of the canal. Traction should not be released on the ear until the examination is completed and the otoscope is removed.

The nurse is performing an otoscopic examination on an adult. Which of these actions is correct? a. Tilting the persons head forward during the examination b. Once the speculum is in the ear, releasing the traction c. Pulling the pinna up and back before inserting the speculum d. Using the smallest speculum to decrease the amount of discomfort

D Use of the Doppler stethoscope is a noninvasive way to determine the extent of peripheral vascular disease. The normal ankle pressure is slightly greater than or equal to the brachial pressure. An ABI of 0.9 to 0.7 indicates the presence of peripheral vascular disease and mild claudication. The ABI is less reliable in patients with diabetes mellitus because of claudication, which makes the arteries noncompressible and may give a false high-ankle pressure.

The nurse is preparing to assess the ankle-brachial index (ABI) of a patient. Which statement about the ABI is true? a. Normal ABI indices are from 0.5 to 1.0. b. Normal ankle pressure is slightly lower than the brachial pressure. c. The ABI is a reliable measurement of peripheral vascular disease in individuals with diabetes. d. An ABI of 0.9 to 0.7 indicates the presence of peripheral vascular disease and mild claudication.

D The dorsalis pedis artery is located on the dorsum of the foot. The nurse should palpate just lateral to and parallel with the extensor tendon of the big toe. The popliteal artery is palpated behind the knee. The posterior tibial pulse is palpated in the groove between the malleolus and the Achilles tendon. No pulse is palpated at the lateral malleolus.

The nurse is preparing to assess the dorsalis pedis artery. Where is the correct location for palpation? a. Behind the knee b. Over the lateral malleolus c. In the groove behind the medial malleolus d. Lateral to the extensor tendon of the great toe

C The Snellen alphabet chart is the most commonly used and most accurate measure of visual acuity. The confrontation test is a gross measure of peripheral vision. The Jaeger card or newspaper tests are used to test near vision.

The nurse is preparing to assess the visual acuity of a 16-year-old patient. How should the nurse proceed? a. Perform the confrontation test. b. Ask the patient to read the print on a handheld Jaeger card. c. Use the Snellen chart positioned 20 feet away from the patient. d. Determine the patients ability to read newsprint at a distance of 12 to 14 inches

B Auscultation of breath sounds should not be limited to only four locations. Sounds produced by the valves may be heard all over the precordium. The stethoscope should be inched in a rough Z pattern from the base of the heart across and down, then over to the apex; or, starting at the apex, it should be slowly worked up (see Figure 19-22). Listening selectively to one sound at a time is best.

The nurse is preparing to auscultate for heart sounds. Which technique is correct? a. Listening to the sounds at the aortic, tricuspid, pulmonic, and mitral areas b. Listening by inching the stethoscope in a rough Z pattern, from the base of the heart across and down, then over to the apex c. Listening to the sounds only at the site where the apical pulse is felt to be the strongest d. Listening for all possible sounds at a time at each specified area

D The girl should be interviewed alone. The parents can wait outside and fill out the family health history questionnaires.

The nurse is preparing to complete a health assessment on a 16-year-old girl whose parents have brought her to the clinic. Which instruction would be appropriate for the parents before the interview begins? a. Please stay during the interview; you can answer for her if she does not know the answer. b. It would help to interview the three of you together. c. While I interview your daughter, will you please stay in the room and complete these family health history questionnaires? d. While I interview your daughter, will you step out to the waiting room and complete these family health history questionnaires?

D The purpose of the health history is to collect subjective data what the person says about him or herself. The other options are not correct.

The nurse is preparing to conduct a health history. Which of these statements best describes the purpose of a health history? a. To provide an opportunity for interaction between the patient and the nurse b. To provide a form for obtaining the patients biographic information c. To document the normal and abnormal findings of a physical assessment d. To provide a database of subjective information about the patients past and current health

D The functional assessment measures how a person manages day-to-day activities. The other answers do not reflect the purpose of a functional assessment.

The nurse is preparing to do a functional assessment. Which statement best describes the purpose of a functional assessment? a. The functional assessment assesses how the individual is coping with life at home. b. It determines how children are meeting developmental milestones. c. The functional assessment can identify any problems with memory the individual may be experiencing. d. It helps determine how a person is managing day-to-day activities.

A For an otoscopic examination on an infant or on a child under 3 years of age, the pinna is pulled down. The other responses are not part of the correct procedure.

The nurse is preparing to do an otoscopic examination on a 2-year-old child. Which one of these reflects the correct procedure? a. Pulling the pinna down b. Pulling the pinna up and back c. Slightly tilting the childs head toward the examiner d. Instructing the child to touch his chin to his chest

C A modified Allen test is used to evaluate the adequacy of collateral circulation before the radial artery is cannulated. The other responses are not reasons for a modified Allen test.

The nurse is preparing to perform a modified Allen test. Which is an appropriate reason for this test? a. To measure the rate of lymphatic drainage b. To evaluate the adequacy of capillary patency before venous blood draws c. To evaluate the adequacy of collateral circulation before cannulating the radial artery d. To evaluate the venous refill rate that occurs after the ulnar and radial arteries are temporarily occluded

C During the first few days after the birth, the tympanic membrane of a newborn often appears thickened and opaque. It may look injected and have a mild redness from increased vascularity. The other statements are not correct.

The nurse is preparing to perform an otoscopic examination of a newborn infant. Which statement is true regarding this examination? a. Immobility of the drum is a normal finding. b. An injected membrane would indicate an infection. c. The normal membrane may appear thick and opaque. d. The appearance of the membrane is identical to that of an adult.

B, C, E, F Nine modifiable risk factors for MI, as identified by a recent study, include abnormal lipids, smoking, hypertension, diabetes, abdominal obesity, psychosocial factors, consumption of fruits and vegetables, alcohol use, and regular physical activi

The nurse is presenting a class on risk factors for cardiovascular disease. Which of these are considered modifiable risk factors for MI? Select all that apply. a. Ethnicity b. Abnormal lipids c. Smoking d. Gender e. Hypertension f. Diabetes g. Family history

D Intraocular pressure is determined by a balance between the amount of aqueous produced and the resistance to its outflow at the angle of the anterior chamber. The other responses are incorrect.

The nurse is reviewing causes of increased intraocular pressure. Which of these factors determines intraocular pressure? a. Thickness or bulging of the lens b. Posterior chamber as it accommodates increased fluid c. Contraction of the ciliary body in response to the aqueous within the eye d. Amount of aqueous produced and resistance to its outflow at the angle of the anterior chamber

A, C, E, F Clustering related cues help the nurse recognize relationships among the data. The cues related to the patients respiratory status (e.g., wheezes, cough, report of dyspnea, respiration rate and rhythm) are all related. Cues related to bowels and peripheral edema are not related to the respiratory cues.

The nurse is reviewing data collected after an assessment. Of the data listed below, which would be considered related cues that would be clustered together during data analysis? Select all that apply. a. Inspiratory wheezes noted in left lower lobes b. Hypoactive bowel sounds c. Nonproductive cough d. Edema, +2, noted on left hand e. Patient reports dyspnea upon exertion f. Rate of respirations 16 breaths per minute

B The lens loses elasticity and decreases its ability to change shape to accommodate for near vision. This condition is called presbyopia.

The nurse is reviewing in age-related changes in the eye for a class. Which of these physiologic changes is responsible for presbyopia? a. Degeneration of the cornea b. Loss of lens elasticity c. Decreased adaptation to darkness d. Decreased distance vision abilities

C EBP is a systematic approach to practice that emphasizes the use of best evidence in combination with the clinicians experience, as well as patient preferences and values, when making decisions about care and treatment. EBP is more than simply using the best practice techniques to treat patients, and questioning tradition is important when no compelling and supportive research evidence exists.

The nurse is reviewing information about evidence-based practice (EBP). Which statement best reflects EBP? a. EBP relies on tradition for support of best practices. b. EBP is simply the use of best practice techniques for the treatment of patients. c. EBP emphasizes the use of best evidence with the clinicians experience. d. The patients own preferences are not important with EBP.

D Toward the end of diastole, the atria contract and push the last amount of blood (approximately 25% of stroke volume) into the ventricles. This active filling phase is called presystole, or atrial systole, or sometimes theatrial kick.

The nurse is reviewing the anatomy and physiologic functioning of the heart. Which statement best describes what is meant by atrial kick? a. The atria contract during systole and attempt to push against closed valves. b. Contraction of the atria at the beginning of diastole can be felt as a palpitation. c. Atrial kick is the pressure exerted against the atria as the ventricles contract during systole. d. The atria contract toward the end of diastole and push the remaining blood into the ventricles.

C The major artery supplying the arm is the brachial artery. The brachial artery bifurcates into the ulnar and radial arteries immediately below the elbow. In the hand, the ulnar and radial arteries form two arches known as the superficial and deep palmar arches.

The nurse is reviewing the blood supply to the arm. The major artery supplying the arm is the _____ artery. a. Ulnar b. Radial c. Brachial d. Deep palmar

C Bronchovesicular breath sounds are heard over major bronchi where fewer alveoli are located posteriorlybetween the scapulae, especially on the right; and anteriorly, around the upper sternum in the first and second intercostal spaces. The other responses are not correct.

The nurse is reviewing the characteristics of breath sounds. Which statement about bronchovesicular breath sounds is true? Bronchovesicular breath sounds are: a. Musical in quality. b. Usually caused by a pathologic disease. c. Expected near the major airways. d. Similar to bronchial sounds except shorter in duration.

D Only the maxillary and ethmoid sinuses are present at birth. The sphenoid sinuses are minute at birth and develop after puberty. The frontal sinuses are absent at birth, are fairly well developed at age 7 to 8 years, and reach full size after puberty.

The nurse is reviewing the development of the newborn infant. Regarding the sinuses, which statement is true in relation to a newborn infant? a. Sphenoid sinuses are full size at birth. b. Maxillary sinuses reach full size after puberty. c. Frontal sinuses are fairly well developed at birth. d. Maxillary and ethmoid sinuses are the only sinuses present at birth.

C The nerve impulses are conducted by the auditory portion of CN VIII to the brain.

The nurse is reviewing the function of the cranial nerves (CNs). Which CN is responsible for conducting nerve impulses to the brain from the organ of Corti? a. I b. III c. VIII d. XI

B People who undergo prolonged standing, sitting, or bed rest are at risk for venous disease. Hypercoagulable (not anticoagulated) states and vein-wall trauma also place the person at risk for venous disease. Obesity and the late months of pregnancy are also risk factors.

The nurse is reviewing the risk factors for venous disease. Which of these situations best describes a person at highest risk for the development of venous disease? a. Woman in her second month of pregnancy b. Person who has been on bed rest for 4 days c. Person with a 30-year, 1 pack per day smoking habit d. Older adult taking anticoagulant medication

D The eustachian tube allows an equalization of air pressure on each side of the tympanic membrane so that the membrane does not rupture during, for example, altitude changes in an airplane. The tube is normally closed, but it opens with swallowing or yawning.

The nurse is reviewing the structures of the ear. Which of these statements concerning the eustachian tube is true? a. The eustachian tube is responsible for the production of cerumen. b. It remains open except when swallowing or yawning. c. The eustachian tube allows passage of air between the middle and outer ear. d. It helps equalize air pressure on both sides of the tympanic membrane.

C Fremitus is a palpable vibration. Sounds generated from the larynx are transmitted through patent bronchi and the lung parenchyma to the chest wall where they are felt as vibrations. Crepitus is the term for air in the subcutaneous tissues.

The nurse is reviewing the technique of palpating for tactile fremitus with a new graduate. Which statement by the graduate nurse reflects a correct understanding of tactile fremitus? Tactile fremitus: a. Is caused by moisture in the alveoli. b. Indicates that air is present in the subcutaneous tissues. c. Is caused by sounds generated from the larynx. d. Reflects the blood flow through the pulmonary arteries.

A Blood moves through the veins by (1) contracting skeletal muscles that proximally milk the blood; (2) pressure gradients caused by breathing, during which inspiration makes the thoracic pressure decrease and the abdominal pressure increase; and (3) the intraluminal valves, which ensure unidirectional flow toward the heart.

The nurse is reviewing venous blood flow patterns. Which of these statements best describes the mechanism(s) by which venous blood returns to the heart? a. Intraluminal valves ensure unidirectional flow toward the heart. b. Contracting skeletal muscles milk blood distally toward the veins. c. High-pressure system of the heart helps facilitate venous return. d. Increased thoracic pressure and decreased abdominal pressure facilitate venous return to the heart.

C Questions concerning any family history of heart disease, high blood pressure, stroke, diabetes, obesity, blood disorders, breast and ovarian cancers, colon cancer, sickle cell anemia, arthritis, allergies, alcohol or drug addiction, mental illness, suicide, seizure disorder, kidney disease, and tuberculosis should be asked.

The nurse is taking a family history. Important diseases or problems about which the patient should be specifically asked include: a. Emphysema. b. Head trauma. c. Mental illness. d. Fractured bones.

D Typically with perforation, ear pain occurs first, stopping with a popping sensation, and then drainage occurs.

The nurse is taking the history of a patient who may have a perforated eardrum. What would be an important question in this situation? a. Do you ever notice ringing or crackling in your ears? b. When was the last time you had your hearing checked? c. Have you ever been told that you have any type of hearing loss? d. Is there any relationship between the ear pain and the discharge you mentioned?

B, D, E One pinch of SLT in the mouth for 30 minutes delivers the equivalent of three cigarettes. Pain is rarely an early sign of oral cancer. Many brands of SLT are sweetened with sugars, which promotes tooth decay. SLT is not considered a healthy alternative to smoking, and the use of SLT has been associated with a greater risk of oral cancer than smoking.

The nurse is teaching a health class to high-school boys. When discussing the topic of using smokeless tobacco (SLT), which of these statements are accurate? Select all that apply. a. One pinch of SLT in the mouth for 30 minutes delivers the equivalent of one cigarette. b. Using SLT has been associated with a greater risk of oral cancer than smoking. c. Pain is an early sign of oral cancer. d. Pain is rarely an early sign of oral cancer. e. Tooth decay is another risk of SLT because of the use of sugar as a sweetener. f. SLT is considered a healthy alternative to smoking.

B The flow of lymph is slow, compared with flow of the blood. Lymph flow is not propelled by the heart but rather by contracting skeletal muscles, pressure changes secondary to breathing, and contraction of the vessel walls. Lymph does not absorb lipids from the biliary tract. The vessels do have valves; therefore, flow is one way from the tissue spaces to the bloodstream.

The nurse is teaching a review class on the lymphatic system. A participant shows correct understanding of the material with which statement? a. Lymph flow is propelled by the contraction of the heart. b. The flow of lymph is slow, compared with that of the blood. c. One of the functions of the lymph is to absorb lipids from the biliary tract. d. Lymph vessels have no valves; therefore, lymph fluid flows freely from the tissue spaces into the bloodstream.

A The muscle fibers of the iris contract the pupil in bright light and accommodate for near vision, which also results in pupil constriction. The other responses are not correct.

The nurse is testing a patients visual accommodation, which refers to which action? a. Pupillary constriction when looking at a near object b. Pupillary dilation when looking at a far object c. Changes in peripheral vision in response to light d. Involuntary blinking in the presence of bright light

B, D, E Presbycusis is a type of hearing loss that occurs with aging and is found in 60% of those older than 65 years. It is a gradual sensorineural loss caused by nerve degeneration in the inner ear or auditory nerve, and it slowly progresses after the age of 50 years. The person first notices a high-frequency tone loss; it is harder to hear consonants (high-pitched components of speech) than vowels, which makes words sound garbled. The ability to localize sound is also impaired.

The nurse is testing the hearing of a 78-year-old man and is reminded of the changes in hearing that occur with aging that include which of the following? Select all that apply. a. Hearing loss related to aging begins in the mid 40s. b. Progression of hearing loss is slow. c. The aging person has low-frequency tone loss. d. The aging person may find it harder to hear consonants than vowels. e. Sounds may be garbled and difficult to localize. f. Hearing loss reflects nerve degeneration of the middle ear.

C Fine crackles are commonly heard in the immediate newborn period as a result of the opening of the airways and a clearing of fluid. Persistent fine crackles would be noticed with pneumonia, bronchiolitis, or atelectasis.

The nurse knows that auscultation of fine crackles would most likely be noticed in: a. A healthy 5-year-old child. b. A pregnant woman. c. The immediate newborn period. d. Association with a pneumothorax.

A An accurate nursing diagnosis provides the basis for the selection of nursing interventions to achieve outcomes for which the nurse is accountable. The other items do not contribute to the development of appropriate nursing interventions.

The nurse knows that developing appropriate nursing interventions for a patient relies on the appropriateness of the __________ diagnosis. a. Nursing b. Medical c. Admission d. Collaborative

C Using authority responses promotes dependency and inferiority. Avoiding the use of authority is best. Although the health care provider and patient do not have equal professional knowledge, both have equally worthy roles in the health process. The other statements are not correct.

The nurse makes this comment to a patient, I know it may be hard, but you should do what the doctor ordered because she is the expert in this field. Which statement is correct about the nurses comment? a. This comment is inappropriate because it shows the nurses bias. b. This comment is appropriate because members of the health care team are experts in their area of patient care. c. This type of comment promotes dependency and inferiority on the part of the patient and is best avoided in an interview situation. d. Using authority statements when dealing with patients, especially when they are undecided about an issue, is necessary at times.

Review of systems

The purpose is to evaluate the past and present health state of each body system, to double check in case any significant data were omitted in the Present Illness section, and to evaluate health promotion practices. Only the most common symptoms are listed. Pg. 50

A The nurse should reduce noise by turning off the television, radio, and other unnecessary equipment, because multiple stimuli are confusing. The interviewer and patient should be approximately 4 to 5 feet apart; the room should be well-lit, enabling the interviewer and patient to see each other clearly. Having a table or desk in between the two people creates the idea of a barrier; equal-status seating, at eye level, is better.

The nurse makes which adjustment in the physical environment to promote the success of an interview? a. Reduces noise by turning off televisions and radios b. Reduces the distance between the interviewer and the patient to 2 feet or less c. Provides a dim light that makes the room cozy and helps the patient relax d. Arranges seating across a desk or table to allow the patient some personal space

A The external ear is called the auricle or pinna and consists of movable cartilage and skin.

The nurse needs to pull the portion of the ear that consists of movable cartilage and skin down and back when administering eardrops. This portion of the ear is called the: a. Auricle. b. Concha. c. Outer meatus. d. Mastoid process.

C The nurse should notice any bruising or laceration on the buccal mucosa or gums of an infant or young child. Trauma may indicate child abuse from a forced feeding of a bottle or spoon.

The nurse notices that the mother of a 2-year-old boy brings him into the clinic quite frequently for various injuries and suspects there may be some child abuse involved. During an inspection of his mouth, the nurse should look for: a. Swollen, red tonsils. b. Ulcerations on the hard palate. c. Bruising on the buccal mucosa or gums. d. Small yellow papules along the hard palate.

C Periorbital edema occurs with local infections, crying, and systemic conditions such as heart failure, renal failure, allergy, and hypothyroidism. Periorbital edema is not associated with blepharitis.

The nurse notices the presence of periorbital edema when performing an eye assessment on a 70-year-old patient. The nurse should: a. Check for the presence of exophthalmos. b. Suspect that the patient has hyperthyroidism. c. Ask the patient if he or she has a history of heart failure. d. Assess for blepharitis, which is often associated with periorbital edema.

C A natural progression to prevention rounds out the present concept of health. Guidelines to prevention place the emphasis on the link between health and personal behavior

The nurse recognizes that the concept of prevention in describing health is essential because: a. Disease can be prevented by treating the external environment. b. The majority of deaths among Americans under age 65 years are not preventable. c. Prevention places the emphasis on the link between health and personal behavior. d. The means to prevention is through treatment provided by primary health care practitioners.

D The nursing process is a method of problem solving that includes assessment, diagnosis, outcome identification, planning, implementation, and evaluation.

The nursing process is a sequential method of problem solving that nurses use and includes which steps? a. Assessment, treatment, planning, evaluation, discharge, and follow-up b. Admission, assessment, diagnosis, treatment, and discharge planning c. Admission, diagnosis, treatment, evaluation, and discharge planning d. Assessment, diagnosis, outcome identification, planning, implementation, and evaluation

A Together with the patients record and laboratory studies, the objective and subjective data form the data base. The other items are not part of the patients record, laboratory studies, or data.

The patients record, laboratory studies, objective data, and subjective data combine to form the: a. Data base. b. Admitting data. c. Financial statement. d. Discharge summary.

A The major muscle of respiration is the diaphragm. The intercostal muscles lift the sternum and elevate the ribs during inspiration, increasing the anteroposterior diameter. Expiration is primarily passive. Forced inspiration involves the use of other muscles, such as the accessory neck musclessternomastoid, scaleni, and trapezii muscles. Forced expiration involves the abdominal muscles.

The primary muscles of respiration include the: a. Diaphragm and intercostals. b. Sternomastoids and scaleni. c. Trapezii and rectus abdominis. d. External obliques and pectoralis major.

B The nasal hairs filter the coarsest matter from inhaled air, whereas the mucous blanket filters out dust and bacteria. The rich blood supply of the nasal mucosa warms the inhaled air.

The primary purpose of the ciliated mucous membrane in the nose is to: a. Warm the inhaled air. b. Filter out dust and bacteria. c. Filter coarse particles from inhaled air. d. Facilitate the movement of air through the nares.

C The lateral walls of each nasal cavity contain three parallel bony projections: the superior, middle, and inferior turbinates. These increase the surface area, making more blood vessels and mucous membrane available to warm, humidify, and filter the inhaled air.

The projections in the nasal cavity that increase the surface area are called the: a. Meatus. b. Septum. c. Turbinates. d. Kiesselbach plexus.

B The purposes of the review of systems are to: (1) evaluate the past and current health state of each body system, (2) double check facts in case any significant data were omitted in the present illness section, and (3) evaluate health promotion practices.

The review of systems provides the nurse with: a. Physical findings related to each system. b. Information regarding health promotion practices. c. An opportunity to teach the patient medical terms. d. Information necessary for the nurse to diagnose the patients medical problem.

Describe the setting when testing the hearing acuity of infants and young children.

The room should be silent and the baby contented. Make a loud, sudden noise out of the baby's peripheral range of vision of about 12 in. You may need to repeat a few times, but you should note these responses: Newborn: startle (Moro) Reflex, acoustic blink reflex 3 to 4 months: acoustic blind reflex, infant stops movement and appears to "listen", stops sucking, quiets if crying, cries if quiet 6 to 8 months: infant turns head to localize sound, responds to own name Preschool and school-aged child: child must be screened with audiometry

Describe the structure and function of the throat.

The throat or pharynx is the area behind the mouth and nose. The oropharynx is separated from the mouth by a fold of tissue on each side, the anterior tonsillar pillar. Behind the folds are the tonsils, each a mass of lymphoid tissue. The tonsils are the same color as the surrounding mucous membrane, although they look more granular and their surface shows deep crypts. Tonsillar tissue enlarges during childhood until puberty and then involutes. The posterior pharyngeal wall is seen behind these structures. Some small blood vessels may show on it. The nasopharynx is continuous with the oropharynx, although it is above the oropharynx and behind the nasal cavity. The pharyngeal tonsils (adenoids) and eustachian tube openings are located here. The oral cavity and throat have a rick lymphatic network.

Marfan Syndrome

This inherited connective tissue disorder is characterized by tall, thin stature (<95th percentile), arachnodactyly (long, thin fingers), hyperextensible joints, arm span greater than height, pubis-to-sole measurement exceeding crown-to-pubis measurement, sternal deformity (note pectus excavatum), high-arched narrow palate, narrow face, and pes planus (flat feet). Early morbidity and mortality occur as a result of cardiovascular complications such as mitral regurgitation and aortic dissection.

C The presence of shadows in the anterior chamber may be a sign of acute angle-closure glaucoma. The normal iris is flat and creates no shadows. This method is not correct for the assessment of dacryocystitis, conjunctivitis, or cataracts.

When a light is directed across the iris of a patients eye from the temporal side, the nurse is assessing for: a. Drainage from dacryocystitis. b. Presence of conjunctivitis over the iris. c. Presence of shadows, which may indicate glaucoma. d. Scattered light reflex, which may be indicative of cataracts.

C First, approximately two thirds of the blood is shunted through an opening in the atrial septum, the foramen ovale, into the left side of the heart, where it is pumped out through the aorta. The foramen ovale closes within the first hour after birth because the pressure in the right side of the heart is now lower than in the left side.

When assessing a newborn infant who is 5 minutes old, the nurse knows which of these statements to be true? a. The left ventricle is larger and weighs more than the right ventricle. b. The circulation of a newborn is identical to that of an adult. c. Blood can flow into the left side of the heart through an opening in the atrial septum. d. The foramen ovale closes just minutes before birth, and the ductus arteriosus closes immediately after.

B If a pulse is weak or diminished at the femoral site, then the nurse should auscultate for a bruit. The presence of a bruit, or turbulent blood flow, indicates partial occlusion. The other responses are not correct.

When assessing a patient, the nurse notes that the left femoral pulse as diminished, 1+/4+. What should the nurse do next? a. Document the finding. b. Auscultate the site for a bruit. c. Check for calf pain. d. Check capillary refill in the toes.

B In the aging adult, the tongue looks smoother because of papillary atrophy. The teeth are slightly yellowed and appear longer because of the recession of gingival margins.

When examining the mouth of an older patient, the nurse recognizes which finding is due to the aging process? a. Teeth appearing shorter b. Tongue that looks smoother in appearance c. Buccal mucosa that is beefy red in appearance d. Small, painless lump on the dorsum of the tongue

A The left lung has two lobes, and the right lung has three lobes. The right lung is shorter than the left lung because of the underlying liver. The left lung is narrower than the right lung because the heart bulges to the left. The posterior chest is almost all lower lobes.

When assessing a patients lungs, the nurse recalls that the left lung: a. Consists of two lobes. b. Is divided by the horizontal fissure. c. Primarily consists of an upper lobe on the posterior chest. d. Is shorter than the right lung because of the underlying stomach.

D In pulsus paradoxus, beats have weaker amplitude with inspiration and stronger amplitude with expiration and is best determined during blood pressure measurement; reading decreases (>10 mm Hg) during inspiration and increases with expiration

When assessing a patients pulse, the nurse notes that the amplitude is weaker during inspiration and stronger during expiration. When the nurse measures the blood pressure, the reading decreases 20 mm Hg during inspiration and increases with expiration. This patient is experiencing pulsus: a. Alternans. b. Bisferiens. c. Bigeminus. d. Paradoxus.

A Normally, fremitus is most prominent between the scapulae and around the sternum. These sites are where the major bronchi are closest to the chest wall. Fremitus normally decreases as one progresses down the chest because more tissue impedes sound transmission.

When assessing tactile fremitus, the nurse recalls that it is normal to feel tactile fremitus most intensely over which location? a. Between the scapulae b. Third intercostal space, MCL c. Fifth intercostal space, midaxillary line (MAL) d. Over the lower lobes, posterior side

C To test the pupillary light reflex, the nurse should advance a light in from the side and note the direct and consensual pupillary constriction.

When assessing the pupillary light reflex, the nurse should use which technique? a. Shine a penlight from directly in front of the patient, and inspect for pupillary constriction. b. Ask the patient to follow the penlight in eight directions, and observe for bilateral pupil constriction. c. Shine a light across the pupil from the side, and observe for direct and consensual pupillary constriction. d. Ask the patient to focus on a distant object. Then ask the patient to follow the penlight to approximately 7 cm from the nose.

C Bronchovesicular breath sounds in the peripheral lung fields of the infant and young child up to age 5 or 6 years are normal findings. Their thin chest walls with underdeveloped musculature do not dampen the sound, as do the thicker chest walls of adults; therefore, breath sounds are loud and harsh.

When assessing the respiratory system of a 4-year-old child, which of these findings would the nurse expect? a. Crepitus palpated at the costochondral junctions b. No diaphragmatic excursion as a result of a childs decreased inspiratory volume c. Presence of bronchovesicular breath sounds in the peripheral lung fields d. Irregular respiratory pattern and a respiratory rate of 40 breaths per minute at rest

A The dorsal surface of the tongue is normally roughened from papillae. A thin white coating may be present. The ventral surface may show veins. Smooth, glossy areas may indicate atrophic glossitis

When assessing the tongue of an adult, the nurse knows that an abnormal finding would be: a. Smooth glossy dorsal surface. b. Thin white coating over the tongue. c. Raised papillae on the dorsal surface. d. Visible venous patterns on the ventral surface.

D A bruit occurs with turbulent blood flow and indicates partial occlusion of the artery. The other responses are not correct.

When auscultating over a patients femoral arteries, the nurse notices the presence of a bruit on the left side. The nurse knows that bruits: a. Are often associated with venous disease. b. Occur in the presence of lymphadenopathy. c. In the femoral arteries are caused by hypermetabolic states. d. Occur with turbulent blood flow, indicating partial occlusion.

C Vesicular breath sounds are low-pitched, soft sounds with inspiration being longer than expiration. These breath sounds are expected over the peripheral lung fields where air flows through smaller bronchioles and alveoli.

When auscultating the lungs of an adult patient, the nurse notes that low-pitched, soft breath sounds are heard over the posterior lower lobes, with inspiration being longer than expiration. The nurse interprets that these sounds are: a. Normally auscultated over the trachea. b. Bronchial breath sounds and normal in that location. c. Vesicular breath sounds and normal in that location. d. Bronchovesicular breath sounds and normal in that location.

C Second-level priority problems are those that require prompt intervention to forestall further deterioration (e.g., mental status change, acute pain, abnormal laboratory values, risks to safety or security)

When considering priority setting of problems, the nurse keeps in mind that second-level priority problems include which of these aspects? a. Low self-esteem b. Lack of knowledge c. Abnormal laboratory values d. Severely abnormal vital signs

C Stimulation of the sympathetic branch of the autonomic nervous system dilates the pupil and elevates the eyelid. Parasympathetic nervous system stimulation causes the pupil to constrict. The muscle fibers of the iris contract the pupil in bright light to accommodate for near vision. The ciliary body controls the thickness of the lens.

When examining a patients eyes, the nurse recalls that stimulation of the sympathetic branch of the autonomic nervous system: a. Causes pupillary constriction. b. Adjusts the eye for near vision. c. Elevates the eyelid and dilates the pupil. d. Causes contraction of the ciliary body.

B The tympanic membrane is a translucent membrane with a pearly gray color and a prominent cone of light in the anteroinferior quadrant, which is the reflection of the otoscope light. The tympanic membrane is oval and slightly concave, pulled in at its center by the malleus, which is one of the middle ear ossicles.

When examining the ear with an otoscope, the nurse notes that the tympanic membrane should appear: a. Light pink with a slight bulge. b. Pearly gray and slightly concave. c. Pulled in at the base of the cone of light. d. Whitish with a small fleck of light in the superior portion.

A The palpebral fissure is the elliptical open space between the eyelids, and, when closed, the lid margins approximate completely, which is a normal finding.

When examining the eye, the nurse notices that the patients eyelid margins approximate completely. The nurse recognizes that this assessment finding: a. Is expected. b. May indicate a problem with extraocular muscles. c. May result in problems with tearing. d. Indicates increased intraocular pressure.

C Rhinorrhea, itching of the nose and eyes, and sneezing are present with allergic rhinitis. On physical examination, serous edema is noted, and the turbinates usually appear pale with a smooth, glistening surface

When examining the nares of a 45-year-old patient who has complaints of rhinorrhea, itching of the nose and eyes, and sneezing, the nurse notices the following: pale turbinates, swelling of the turbinates, and clear rhinorrhea. Which of these conditions is most likely the cause? a. Nasal polyps b. Acute sinusitis c. Allergic rhinitis d. Acute rhinitis

D Inspection of the anterior chest includes shape and configuration of the chest wall; assessment of the patients level of consciousness and the patients skin color and condition; quality of respirations; presence or absence of retraction and bulging of the intercostal spaces; and use of accessory muscles. Symmetric chest expansion is assessed by palpation. Diaphragmatic excursion is assessed by percussion of the posterior chest. Breath sounds are assessed by auscultation.

When inspecting the anterior chest of an adult, the nurse should include which assessment? a. Diaphragmatic excursion b. Symmetric chest expansion c. Presence of breath sounds d. Shape and configuration of the chest wall

C When unsure of a sound heard while listening to a patients breath sounds, the nurse validates the data to ensure accuracy. If the nurse has less experience in an area, then he or she asks an expert to listen.

When listening to a patients breath sounds, the nurse is unsure of a sound that is heard. The nurses next action should be to: a. Immediately notify the patients physician. b. Document the sound exactly as it was heard. c. Validate the data by asking a coworker to listen to the breath sounds. d. Assess again in 20 minutes to note whether the sound is still present.

ANS: C The S1 coincides with the carotid artery pulse, is the start of systole, and is louder than the S2 at the apex of the heart; the S2 is louder than the S1 at the base. The nurse should gently feel the carotid artery pulse while auscultating at the apex; the sound heard as each pulse is felt is the S1.

When listening to heart sounds, the nurse knows that the S1: a. Is louder than the S2 at the base of the heart. b. Indicates the beginning of diastole. c. Coincides with the carotid artery pulse. d. Is caused by the closure of the semilunar valves.

C The second heart sound (S2) occurs with the closure of the semilunar (aortic and pulmonic) valves and signals the end of systole. Although it is heard over all the precordium, the S2 is loudest at the base of the heart.

When listening to heart sounds, the nurse knows the valve closures that can be heard best at the base of the heart are: a. Mitral and tricuspid. b. Tricuspid and aortic. c. Aortic and pulmonic. d. Mitral and pulmonic.

D When nonverbal and verbal messages are congruent, the verbal message is reinforced. When they are incongruent, the nonverbal message tends to be the true one because it is under less conscious control. Thus studying the nonverbal messages of the patients and examiners and understanding their meanings are important. The other statements are not true.

When observing a patients verbal and nonverbal communication, the nurse notices a discrepancy. Which statement is true regarding this situation? The nurse should: a. Ask someone who knows the patient well to help interpret this discrepancy. b. Focus on the patients verbal message, and try to ignore the nonverbal behaviors. c. Try to integrate the verbal and nonverbal messages and then interpret them as an average. d. Focus on the patients nonverbal behaviors, because these are often more reflective of a patients true feelings.

C Palpating the ulnar pulses is not usually necessary. The ulnar pulses are not often palpable in the normal person. The other responses are not correct.

When performing a peripheral vascular assessment on a patient, the nurse is unable to palpate the ulnar pulses. The patients skin is warm and capillary refill time is normal. Next, the nurse should: a. Check for the presence of claudication. b. Refer the individual for further evaluation. c. Consider this finding as normal, and proceed with the peripheral vascular evaluation. d. Ask the patient if he or she has experienced any unusual cramping or tingling in the arm.

C The right and left costal margins form an angle where they meet at the xiphoid process. Usually, this angle is 90 degrees or less. The angle increases when the rib cage is chronically overinflated, as in emphysema.

When performing a respiratory assessment on a patient, the nurse notices a costal angle of approximately 90 degrees. This characteristic is: a. Observed in patients with kyphosis. b. Indicative of pectus excavatum. c. A normal finding in a healthy adult. d. An expected finding in a patient with a barrel chest.

D The epitrochlear nodes are located in the antecubital fossa and drain the hand and lower arm. The other actions are not correct for this assessment finding.

When performing an assessment of a patient, the nurse notices the presence of an enlarged right epitrochlear lymph node. What should the nurse do next? a. Assess the patients abdomen, and notice any tenderness. b. Carefully assess the cervical lymph nodes, and check for any enlargement. c. Ask additional health history questions regarding any recent ear infections or sore throats. d. Examine the patients lower arm and hand, and check for the presence of infection or lesions.

A An amber-yellow color to the tympanic membrane suggests serum or pus in the middle ear. Air or fluid or bubbles behind the tympanic membrane are often visible. The patient may have feelings of fullness, transient hearing loss, and a popping sound with swallowing. These findings most likely suggest that the child has serous otitis media. The other responses are not correct.

When performing an otoscopic examination of a 5-year-old child with a history of chronic ear infections, the nurse sees that his right tympanic membrane is amber-yellow in color and that air bubbles are visible behind the tympanic membrane. The child reports occasional hearing loss and a popping sound with swallowing. The preliminary analysis based on this information is that the child: a. Most likely has serous otitis media. b. Has an acute purulent otitis media. c. Has evidence of a resolving cholesteatoma. d. Is experiencing the early stages of perforation.

C The history should be limited to patient statements or subjective data factors that the person says were or were not present.

Which of these statements represents subjective data the nurse obtained from the patient regarding the patients skin? a. Skin appears dry. b. No lesions are obvious. c. Patient denies any color change. d. Lesion is noted on the lateral aspect of the right arm.

D Consideration of the whole person is the essence of holistic health, which views the mind, body, and spirit as interdependent. The basis of disease originates from both the external environment and from within the person. Both the individual human and the external environment are open systems, continually changing and adapting, and each person is responsible for his or her own personal health state.

When reviewing the concepts of health, the nurse recalls that the components of holistic health include which of these? a. Disease originates from the external environment. b. The individual human is a closed system. c. Nurses are responsible for a patients health state. d. Holistic health views the mind, body, and spirit as interdependent.

A Silent attentiveness communicates that the person has time to think and to organize what he or she wishes to say without an interruption from the nurse. Health professionals most often interrupt this thinking silence. The other responses are not conducive to ideal communication.

When taking a history from a newly admitted patient, the nurse notices that he often pauses and expectantly looks at the nurse. What would be the nurses best response to this behavior? a. Be silent, and allow him to continue when he is ready. b. Smile at him and say, Dont worry about all of this. Im sure we can find out why youre having these pains. c. Lean back in the chair and ask, You are looking at me kind of funny; there isnt anything wrong, is there? d. Stand up and say, I can see that this interview is uncomfortable for you. We can continue it another time.

C Functional assessment includes interpersonal relationships and home environment. Family history includes illnesses in family members; a review of systems includes questions about the various body systems; and the reason for seeking care is the rationale for requesting health care.

When the nurse asks for a description of who lives with a child, the method of discipline, and the support system of the child, what part of the assessment is being performed? a. Family history b. Review of systems c. Functional assessment d. Reason for seeking care

C The correct technique for auscultating the carotid artery for bruits involves the nurse lightly applying the bell of the stethoscope over the carotid artery at three levels. While listening, the nurse asks the patient take a breath, exhale, and briefly hold it. Holding the breath on inhalation will also tense the levator scapulae muscles, which makes it hard to hear the carotid arteries. Examining only one carotid artery at a time will avoid compromising arterial blood flow to the brain. Pressure over the carotid sinus, which may lead to decreased heart rate, decreased blood pressure, and cerebral ischemia with syncope, should be avoided.

When the nurse is auscultating the carotid artery for bruits, which of these statements reflects the correct technique? a. While listening with the bell of the stethoscope, the patient is asked to take a deep breath and hold it. b. While auscultating one side with the bell of the stethoscope, the carotid artery is palpated on the other side to check pulsations. c. While lightly applying the bell of the stethoscope over the carotid artery and listening, the patient is asked to take a breath, exhale, and briefly hold it. d. While firmly placing the bell of the stethoscope over the carotid artery and listening, the patient is asked to take a breath, exhale, and briefly hold it.

B A reliable person always gives the same answers, even when questions are rephrased or are repeated later in the interview. The other statements are not correct.

When the nurse is evaluating the reliability of a patients responses, which of these statements would be correct? The patient: a. Has a history of drug abuse and therefore is not reliable. b. Provided consistent information and therefore is reliable. c. Smiled throughout interview and therefore is assumed reliable. d. Would not answer questions concerning stress and therefore is not reliable.

B Clustering related cues helps the nurse see relationships among the data.

Which critical thinking skill helps the nurse see relationships among the data? a. Validation b. Clustering related cues c. Identifying gaps in data d. Distinguishing relevant from irrelevant

C Some murmurs are common in healthy children or adolescents and are termed innocent or functional. The innocent murmur is heard at the second or third left intercostal space and disappears with sitting, and the young person has no associated signs of cardiac dysfunction

Which of these findings would the nurse expect to notice during a cardiac assessment on a 4-year-old child? a. S3 when sitting up b. Persistent tachycardia above 150 beats per minute c. Murmur at the second left intercostal space when supine d. Palpable apical impulse in the fifth left intercostal space lateral to midclavicular line

D Functional assessment measures a persons self-care ability in the areas of general physical health or absence of illness. The other statements concern health or illness issues.

Which of these responses might the nurse expect during a functional assessment of a patient whose leg is in a cast? a. I broke my right leg in a car accident 2 weeks ago. b. The pain is decreasing, but I still need to take acetaminophen. c. I check the color of my toes every evening just like I was taught. d. Im able to transfer myself from the wheelchair to the bed without help.

C Events occur just slightly later in the right side of the heart because of the route of myocardial depolarization. As a result, two distinct components to each of the heart sounds exist, and sometimes they can be heard separately. In the first heart sound, the mitral component (M1) closes just before the tricuspid component (T1).

Which of these statements describes the closure of the valves in a normal cardiac cycle? a. The aortic valve closes slightly before the tricuspid valve. b. The pulmonic valve closes slightly before the aortic valve. c. The tricuspid valve closes slightly later than the mitral valve. d. Both the tricuspid and pulmonic valves close at the same time.

A The spinous process of C7 is the vertebra prominens and is the most prominent bony spur protruding at the base of the neck. Counting ribs and intercostal spaces on the posterior thorax is difficult because of the muscles and soft tissue. The vertebra prominens is easier to identify and is used as a starting point in counting thoracic processes and identifying landmarks on the posterior chest.

Which of these statements is true regarding the vertebra prominens? The vertebra prominens is: a. The spinous process of C7. b. Usually nonpalpable in most individuals. c. Opposite the interior border of the scapula. d. Located next to the manubrium of the sternum.

C Diagnostic reasoning calls for the nurse to formulate a diagnostic hypothesis; the nursing process calls for a nursing diagnosis.

Which of these would be formulated by a nurse using diagnostic reasoning? a. Nursing diagnosis b. Medical diagnosis c. Diagnostic hypothesis d. Diagnostic assessment

D In a focused or problem-centered data base, the nurse collects a mini data base, which is smaller in scope than the completed data base. This mini data base primarily concerns one problem, one cue complex, or one body system.

Which situation is most appropriate during which the nurse performs a focused or problem-centered history? a. Patient is admitted to a long-term care facility. b. Patient has a sudden and severe shortness of breath. c. Patient is admitted to the hospital for surgery the following day. d. Patient in an outpatient clinic has cold and influenza-like symptoms.

B The apex of the lung on the anterior chest is 3 to 4 cm above the inner third of the clavicles. On the posterior chest, the apices are at the level of C7.

Which statement about the apices of the lungs is true? The apices of the lungs: a. Are at the level of the second rib anteriorly. b. Extend 3 to 4 cm above the inner third of the clavicles. c. Are located at the sixth rib anteriorly and the eighth rib laterally. d. Rest on the diaphragm at the fifth intercostal space in the midclavicular line (MCL).

B The pumping heart makes the arterial system a high-pressure system.

Which statement is true regarding the arterial system? a. Arteries are large-diameter vessels. b. The arterial system is a high-pressure system. c. The walls of arteries are thinner than those of the veins. d. Arteries can greatly expand to accommodate a large blood volume increase.

B A physiologic S3 is common in children. A venous hum, caused by turbulence of blood flow in the jugular venous system, is common in healthy children and has no pathologic significance. Heart murmurs that are innocent (or functional) in origin are very common through childhood.

While auscultating heart sounds on a 7-year-old child for a routine physical examination, the nurse hears an S3, a soft murmur at the left midsternal border, and a venous hum when the child is standing. What would be a correct interpretation of these findings? a. S3 is indicative of heart disease in children. b. These findings can all be normal in a child. c. These findings are indicative of congenital problems. d. The venous hum most likely indicates an aneurysm.

C The rhythm should be regular, although sinus arrhythmia occurs normally in young adults and children. With sinus arrhythmia, the rhythm varies with the persons breathing, increasing at the peak of inspiration and slowing with expiration.

While counting the apical pulse on a 16-year-old patient, the nurse notices an irregular rhythm. His rate speeds up on inspiration and slows on expiration. What would be the nurses response? a. Talk with the patient about his intake of caffeine. b. Perform an electrocardiogram after the examination. c. No further response is needed because sinus arrhythmia can occur normally. d. Refer the patient to a cardiologist for further testing.

A Children at risk for a hearing deficit include those exposed in utero to a variety of conditions, such as maternal rubella or to maternal ototoxic drugs.

While discussing the history of a 6-month-old infant, the mother tells the nurse that she took a significant amount of aspirin while she was pregnant. What question would the nurse want to include in the history? a. Does your baby seem to startle with loud noises? b. Has your baby had any surgeries on her ears? c. Have you noticed any drainage from her ears? d. How many ear infections has your baby had since birth?

B With a nosebleed, the person should sit up with the head tilted forward and pinch the nose between the thumb and forefinger for 5 to 15 minutes.

While obtaining a health history, a patient tells the nurse that he has frequent nosebleeds and asks the best way to get them to stop. What would be the nurses best response? a. While sitting up, place a cold compress over your nose. b. Sit up with your head tilted forward and pinch your nose. c. Just allow the bleeding to stop on its own, but dont blow your nose. d. Lie on your back with your head tilted back and pinch your nose.

B With carcinoma, the initial lesion is round and indurated, but then it becomes crusted and ulcerated with an elevated border. Most cancers occur between the outer and middle thirds of the lip. Any lesion that is still unhealed after 2 weeks should be referred.

While performing an assessment of the mouth, the nurse notices that the patient has a 1-cm ulceration that is crusted with an elevated border and located on the outer third of the lower lip. What other information would be most important for the nurse to assess? a. Nutritional status b. When the patient first noticed the lesion c. Whether the patient has had a recent cold d. Whether the patient has had any recent exposure to sick animals

B Absent or distorted light reflex and a bright red color of the eardrum are indicative of acute otitis media.

While performing the otoscopic examination of a 3-year-old boy who has been pulling on his left ear, the nurse finds that his left tympanic membrane is bright red and that the light reflex is not visible. The nurse interprets these findings to indicate a(n): a. Fungal infection. b. Acute otitis media. c. Perforation of the eardrum. d. Cholesteatoma.

When an otoscopic examination is performed on an older adult patient, the tympanic membrane may be

White than that of a younger child.

What term refers to a linear skin lesion that runs along a nerve route?

Zoeteriform Zosteriform describes a lesion that has a linear arrangement along a nerve root. Annular describes a lesion that is circular and begins in the center and spreads to the periphery. A dermatome is an area of skin that is mainly supplied by a single spinal nerve. Shingles (herpes zoster) are small grouped vesicles that emerge along the route of a cutaneous sensory nerve, followed by pustules, and then crusts; shingles is caused by the herpes zoster virus.

An ophthalmoscope is used for

a funduscopic examination, which is an examination of the internal structures of the eye. \An otoscope is used to visualize the ear canal and tympanic membrane. A flashlight or penlight and tongue depressor are used to examine the pharynx. An otoscope may also be used with a short, broad speculum to view the nasal turbinates and nares.

ecchymosis

a hemorrhagic spot or blotch, larger than petechial, in the skin or mucous membrane, forming a non-elevated, rounded or irregular blue or purplish match


Kaugnay na mga set ng pag-aaral

A - Min Net Capital Dollar Requirements

View Set

Behaviors- Interpersonal Violence

View Set